Top Banner
Directions for questions 11-20: Each problem contains a question and two statements giving certain data. You have to select the correct answer from (a) to (d) depending on the sufficiency of the data given in the statements to answer the question. Mark your answer as (a) If statement (I) alone is sufficient. (b) If statement (II) alone is sufficient. (c) If both (I) and (II) together are sufficient but neither of statements alone is sufficient. (d) Either of the statements (I) and (II) is sufficient. (e) If statements (I) and (II) together are not sufficient. 11. What is the distance from City A to City C in kms? (I) City A is 90 kms from City B. (II) City B is 30 kms from City C. 12. Is z less than w? z and w are real numbers. (I) z2 = 25 (II) w = 9 13. The value of an estate in January 1905 started gradually declining in such a way that at the end of each year it was worth only x times its value at the beginning of the year. What was its worth in end December 1910 ? (I) It was worth Rs.10,109 in the end of December 1906. (II)It was worth Rs.12,345 in the beginning of January 1905. 14. In an election, 3 candidates A,B and C were representing for a membership of parliament. How many votes did each receive? (I) A received 1006 votes more than B and 1213 more votes than C. (II) Total votes cast were 15,414. 15. John studies Chinese in a school. Which school does he attend? (I) All students in Jefferson High school take French. (II) Maysville High School offers only Chinese. 16. How many girls passed the entrance exam this year? (I) Last year 560 girls passed (II) This year there was a 10% decrease over last year in the number of failures. 17. What is Raju's age? (I) Raju, Vimala and Kishore are all of the same age. (II) Total age of Vimala, Kishore and Abishek is 32 and Abishek is as old
175

Quantitative

Nov 19, 2014

Download

Documents

KRISHNAVINOD
Welcome message from author
This document is posted to help you gain knowledge. Please leave a comment to let me know what you think about it! Share it to your friends and learn new things together.
Transcript
Page 1: Quantitative

Directions for questions 11-20: Each problem contains a question and two statements giving certain data. You have to select the correct answer from (a) to (d) depending on the sufficiency of the data given in the statements to answer the question. Mark your answer as (a) If statement (I) alone is sufficient.(b) If statement (II) alone is sufficient.(c) If both (I) and (II) together are sufficient but neither of statements alone is sufficient.(d) Either of the statements (I) and (II) is sufficient.(e) If statements (I) and (II) together are not sufficient.

11. What is the distance from City A to City C in kms?(I) City A is 90 kms from City B.(II) City B is 30 kms from City C.

12. Is z less than w? z and w are real numbers.(I) z2 = 25(II) w = 9

13. The value of an estate in January 1905 started gradually declining in such a way that at the end of each year it was worth only x times its value at the beginning of the year. What was its worth in end December 1910 ?(I) It was worth Rs.10,109 in the end of December 1906.(II)It was worth Rs.12,345 in the beginning of January 1905.

14. In an election, 3 candidates A,B and C were representing for a membership of parliament. How many votes did each receive?(I) A received 1006 votes more than B and 1213 more votes than C.(II) Total votes cast were 15,414.

15. John studies Chinese in a school. Which school does he attend?(I) All students in Jefferson High school take French.(II) Maysville High School offers only Chinese.

16. How many girls passed the entrance exam this year?(I) Last year 560 girls passed(II) This year there was a 10% decrease over last year in the number of failures.

17. What is Raju's age?(I) Raju, Vimala and Kishore are all of the same age.(II) Total age of Vimala, Kishore and Abishek is 32 and Abishek is as old as Vimala and Kishore together.

18. Is Sreedhar eligible for an entry pass to the company premisers?(I) The company does not allow strangers to enter the company.(II)All employees are elgible to get a pass.

19. Among five friends who is the tallest?(I) D is taller than A and C.(II)B is shorter than E but taller than D.

Page 2: Quantitative

20. Can a democratic system operate without effective opposition?(I) The opposition is indispensable.(II) A good statesman always learns more from his opponents than fromhis fervent supporters.

Directions for question 1-2 : Answer the questions based on the passage above them

A temple has 3 gateways, each of them is leading you into the temple, and at the end of each gateway there is an idol and as a devotee passes through the gateway with some flowers the number of flowers double. Ram enters the 1st gateway with some flowers and he puts same number of flowers at each idol and the end he is left with none.

21. How many flowers did Ram start with?(a) 4 (b) 5 (c) 3 (d) 7

22. How many flowers does he put at each idol?(a) 10 (b) 8 (c) 6 (d) 5

9)which is more economical of the followinga)2kg -- 30/- b)8kg -- 160/- c)5kg -- 80/-

11) bhanu spends 30% of his income on petrol on scooter. ¼ of the remaining on house rent and the balance on food. if he spends rs.300 on petrol then what is the expenditure on house rent? a) rs.525 b) rs.1000 c) rs.675 d) rs.175

12) a sporting goods store ordered an equal number of white and yellow balls. the tennis ball company delivered 45 extra white balls, making the ratio of white balls to yellow balls 1/5 : 1/6. how many white tennis balls did the store originally order for?a) 450 b) 270 c) 225 d)none of these

13) there is a circular pizza with negligible thickness that is cut into 'x' pieces by 4 straight line cuts. what is the maximum and minimum value of 'x' respectively? a) 12,6 b) 11,6 c) 12,5 d) 11,5

14) the total expense of a boarding house are partly fixed and partly variable with the number of boarders. the charge is rs.70 per head when there are 25 boarders and rs.60 when there are 50 boarders. find the charge per head when there are 100 boarders.a) 65 b) 55 c) 50 d) 45

counterfeit1)in 80 coins one coin is counterfiet what is minimum number of weighings to find out counterfiet coin

curves1)study the nature of the curves y==logx, xy==c etc

Page 3: Quantitative

2)curve was given & in option equation was given , you have to mark the correct equation(curve and equation was of general logx,ex ,sinx,cosx,tanx,x curves)

3)values of x &f(x) was given (like x=1000&f(x)=3) and equations was given

4)which shape will be obtained by using these values of x,yx y0 0.0000110 1.02100 1.721000 3.009999 4.72

4a)which shape will be obtained by using these values of x ,yxy00.00001101.021001.7210003.0099994.72

lcm1) lcm of x and y is 36 and hcf of x and y is 4. if x = 4 then which of the following is definitely not true?a)y is divisible by 3 b)y is divisible by 6 c)y is divisible by 9 d)x+y is divisible by

logarithms1) log .342==..., log . 343==.... then log .341==????? etc...

2) value of log3.142& log3.143 was given and log3,141 was asked so you have to only see on answer and mark

3) if log 0.317=……… and log 0.318=………. then find the value of log 0.319.

match1)match the following:1. male - boy ---> a. a type of

2. square - polygon ---> b. a part of3. roof - building ---> c. not a type of3. scooter --------- automobile a. a part of2. oxygen----------- water b. a type of3. shop staff------- fitters c. not a type of4. bug -------------reptile d. a superset of

matrix1)find the singularity matrix from a given set of matrices?(hint det(a)====0)

Page 4: Quantitative

memory1)if the vertex (5,7) is placed in the memory. first vertex (1,1) 's address is 1245 and then address of (5,7) is ----------1279

2)a 2d array is declared as a[9,7] and each element requires 2 byte. if a[ 1,1 ] is stored in 3000. find the memory of a[8,5] ? 3106

3)one circular array is given (means the memory allocation takes place like a circular fashion )dimension (9x7) .starting address is 3000.find the address of (2,3)..555

4)some has got the qs. as memory allocation takes place coloumnwise.u have to find out the address.

mixture1)16 litre can, 7 litre can,3 litre can,the customer has to be given 11 litres of milk using all the three cans only explain?

number system1) the number 362 in decimal system is given by (1362)x in the x system of numbers find the value of x

a)5 b) 6 c) 7 d) 8 e) 9

2) decimal no 319 is equal to 1234.this 1234 belongs to which number systems

3)194 base 10 = ____ base 5 (1234)

partnership2) a sum of money is divided among a, b and c such that for each rupee a gets, b gets 65paise and c gets 35paise. if c's share is rs.560, the sum is …a)2400 b)2800 c)3200 d)3800

percentage

2) a report consists of 20 sheets each of 55 lines and each such line consists of 65 characters. this report is retyped into sheets each of 65 lines such that each line consists of 70 characters. the % reduction in the number of sheets is closest toa)20 b)5 c)30 d)35

3) amal bought 5 pens, 7 pencils and 4 erasers. rajan bought 6 pens, 8 erasers and 14 pencils for an amount which was half more than what amal had paid. what % of the total amount paid by amal was paid for pens?a)37.5% b)62.5% c)50% d)none of these

permutations1)how many 3 digits with 3 distinct digits are there from 100-1000.

power of

Page 5: Quantitative

2) 8 to the power of x is 32,what is the value of x?

3)which is a/not a power of 2 or 3.

5)which one will be the exact power of 3(i) 2768 (ii)2678 (iii) 2187

prime numbers1)find the highest prime number that can be stored in an 8bit computer.

3)largest prime no. in a 6 bit,8 bit (ans 127),9 bit microprocessor

5)what is the largest prime number that can be stored in an 8-bit memory? 251 or 127

6)which is the largest prime number that can be stored in a 9-bit register?

probability1)in a company 30% are supervisors and 40% employees are male if 60% of supervisors are male. what is the probability that a randomly choosen employee is a male or female?

2) 12 blacksox and 12 whitesox mixed in a box,a pair of sox is picked at a time,in which pick/how many pick ,to get the right pair(black&black or white&white)?

3)two coins one with head in both sides and the other coin head in one side and tail in the other side is in a box,a coin is taken at random and found head in one side .what is the probability that the other side is head?

4) there are 5 distinct pairs of white socks and 5 pairs of black socks in a cupboard. in the dark, how many socks do i have to pull out to ensure that i have at least 1 correct pair of white socks?

a)3 b)11 c)12 d)16

problems on numbers

4) what is the number of zeros at the end of the product of the numbers from 1 to 100

5)9/10 or 10/11 which is greater.

6) (x-y/3)-(y-x/3)=?

7)x:y=3 and x+y=80 what is the value of y? answer y=20 m(893,10)+r( )+t( ) is asked

10)m(373, 7) + r(6. –t(3.4) + r( 3.4) 12)which is the perfect one among the 41. 2x +3y=4 2. x + y= -1 3. y=2x+ 3

Page 6: Quantitative

15) if the numerator of a fraction is increased by 25% and denominator decreased by 20%, the new value is 5/4. what is the original value?a) 3/5 b) 4/5 c) 7/8 d) 3/7

16)the value of ¾ + 5 / 36 + 7 / 144 + …….+17 / 5184 + 19 / 8100 isa)0.99 b)0.98 c)0.95 d)none of these

17) a student's grade in a course is determined by 6 quizzes and one examination. if the examination counts thrice as much as each of the quizzes, what fraction of final grade is determined by the examination?a)1/6 b)1/5 c)1/3 d)¼

18. when ¾ of a unit's digit is added to the ten's digit of a two number, the sum of the digits becomes 10. if ¼ of the ten's digit added to the unit's digit, then the sum of the digits is 1 less than the previous. find the number.a)94 b)84 c)48 d)88

19) a person had to multiply two numbers. instead of multiplying by 35, he multiplied by 53and the product went up by 540. what was the raised product?a)780 b)1040 c)1590 d)1720

rectangular1) which is not a side of a rectangular1. (2,3,4) 2.(3,4,7) 3. (3,5,9)

3) if the length of a rectangle is increased by 30% and the width is decreased by 20%, then the area is increased by...(a) 10% (b) 5% (c) 4% (d) 20% (e) 25%

4) the length of a rectangle is increased by 60%. by what % would the width have to be decreased to maintain the same area? a)30% b)60% c)75% d)37.5%

recursion

2)g (0) =1 g (1)= -1 if g (n)=2* (g (n-1))– 3(g (n-2)) then what is the value of g (4)?

4)if g(0)=g (1)=1 and g (n)= g (n-1) + g (n –2) find g (6);

temperature2)for temperature a function is given according to time : ((t**2)/6) + 4t +12 what is the temperature rise or fall between 4.am to 9 am

3)low temperature at the night in a city is 1/3 more than 1/2 high as higher temperature in a day. sum of the low tem.  and highest temp. is 100 degrees. then what is the low temp? (40 deg.)

time and distance

Page 7: Quantitative

3) falling height is proportional to square of the time. one object falls 64cm in 2sec than in 6sec from how much height the object will fall.

4)a car has run 10000 miles using 5 tyres interchangably,to have a equal wornout by all tyres how many miles each tyre should have run. answer 4000 miles/tyre

6)a ship started from port and moving with i miles per hour and another ship started from l and moving with h miles per hour. at which place these two ships meet?|----|----|----|----|----|----|port g h i j k l

7)a person was fined for exceeding the speed limit by 10mph. another person was also fined for exceeding the same speed limit by twice the same. if the second person was traveling at a speed of 35 mph, find the speed limit.sol: let ‘x’ be the speed limitperson ‘a’ was fined for exceeding the speed limit by = 10mphperson ‘b’ was fined for exceeding the speed limit by = twice of ‘a’ = 2*10mph=20mphgiven that the second person was traveling at the speed of 35mph=> 35mph – 20mph = 15mph

8)a bus started from bustand at 8.00am, and after 30 minutes staying at destination, it returned back to the busstand. the destination is 27 miles from the busstand. the speed of the bus is 18mph. In return journey bus travels with 50% fast speed. at what time it returns to the busstand? (11.00am).

9) wind flows 160 miles in 330 min, for 80 miles how much time required.

10)a storm will move with a velocity of towards the centre in hours, at the same rate how much far will it move in hrs.( but the answer is 8/3 or 2 2/3 )

11)if a is traveling at 72 km per hour on a highway. b is traveling at a speed of 25 meters per second on a highway. what is the difference in their speeds in meters per second?(a) 1/2 m/sec (b) 1 m/sec (c) 1 1/2 m/sec (d) 2 m/sec (e) 3 m/sec

12)a traveler walks a certain distance. had he gone half a kilometer an hour faster \, he would have walked it in 4/5 of the time, and had he gone half a kilometer an hour slower, he would have walked 2 ½ hr longer. what is the distance?a)10 km b)15 km c)20 km d)data insufficient

Page 8: Quantitative

13) a ship leaves on a long voyage. when it is 18 miles from the shore, a seaplane, whose speed is 10 times that of the ship is sent to deliver mail. how far from the shore does the seaplane catch upo with the ship?a)24 miles b)25 miles c)22 miles d)20 miles

14) in a circular race track of length 100 m, three persons a, b and c start together. a and b start in the same direction at speeds of 10 m/s and 8 m/s respectively. while c runs in the opposite at 15 m/s. when will all the three meet for the first time on the after the start?a)after 4s b)after 50s c)after 100s d)after 200s

15) if the distance traveled (s) in time (t) by a partile is given by the formula s = 1+ 2t+3t2+4t3 , then what is the distance travelled in the 4th second of its motion?a)141m b)171m c)243 m d)313 m

16) a non stop bus to amritsar overtakes an auto also moving towards amritsar at 10 am. the bus reaches amritsar at 12.30 pm and starts on the return journey after 1 hr. on the way back it meets the auto at 2 pm. at what time the auto will reach amritsar?a) 2.30pm b)3.00pm c)3.15pm d)3.30pm

time and work

9)a can do a piece of work in 20 days, which b can do in 12 days. in 9 days b does ¾ of the work. how many days will a take to finish the remaining work?

10)anand finishes a work in 7 days, bittu finishes the same job in 8 days and chandu in 6 days. they take turns to finish the work. anand on the first day, bittu on the second and chandu on the third day and then anand again and so on. on which day will the work get over? a)3rd b)6th c)9th d)7th

11) 3 men finish painting a wall in 8 days. four boys do the same job in 7 days. in how many days will 2 men and 2 boys working together paint two such walls of the same size?a) 6 6/13 days b)3 3/13 days c)9 2/5 days d) 12 12/13 days

trains1)two stations a & b are 110 km apart. one train starts from a at 7am, and travels towards b at 20kmph. another train starts from b at 8 am and travels towards a at 25kmph. at what time will they meet?a. 9 am b. 10 am c. 11 am d. 10.30 am

tree

1) 900 m wide 3000 m width something i can’t remember some values are given by air per m rs. 4 by ground per m rs. 5 then where u will cut

Page 9: Quantitative

2)two trees are there. one grows at 3/5 of the other in 4 years, total growth of trees is 8 ft. what growth will smaller tree will have in 2 years ( < 2 ft. ) ie 1 ½ feet

triangles1) given the length of the 3 sides of a triangle. find the one that is impossible? (hint : sum of smaller 2 sides is greater than the other one which is larger)

2)3 angles or 3 sides r given.which will form a triangle?

Q27. If the equation x2 - 3x + a = 0 has the roots (0,1) then value of a is ?

Q28. A and B's temperature are 10°c and 20°c having same surface , then their ratio of rate of emmisions is ?Q35. One U-230 nucleus is placed in a train moving by velocity emiting alpha rays .When the train is at rest the distance between nucleus and alpha particle is x . One passenger is observing the particle. When the train is moving what is the distance between particle and nucleus ?

(a) x (b) x + vt (c) x - vt1. Replace the letters with numbers and solve the equation 4(ABCDE) = EDCBA.

(A) 87978 (B) 98765 (C) 56789 (D) 87912 (E) None of these2. A certain number of workmen can do a piece of work in 25 days, in what time will another set of an equal number of men do a piece of work as great supposing that 2 men of the first set can do as much work in a hour as 3 men in the second set can do in an hour

(A) 60 days (B)75 days (C) 90 days (D) 105 days (E) None of these3. You are given two different length strings that have the characteristic that they both take exactly one hour to burn. However, neither string burns at a constant rate. Some sections of the strings burn very fast, other sections burn very slow. All you have to work with is a matches to calculate when exactly 45 minutes has elapsed.(A)1. Light both ends of the first string2. Light end of the second string at the same time.3. When the first string is finished burning,4. Light the unlit end of the second string.5. When the second string is finished burning exactly 45 minutes will have passed.(B)1. Light both ends of the second string2. Light beginning of the first string at the same time.3. when the first string is finished burning,4. Light the unlit end of the second string.5. When second string is finished burning exactly 45 minutes will have passed.(C)1. Light end of the first string2. Light end of the second string at the same time3. When the first string is finished burning,4. Light the unlit end of the second string.5. When the second string is finished burning exactly 45 minutes will have passed.(D) None of these8.Z4 X3 V9

Page 10: Quantitative

A6 C2 ?T5 R4 P15(A) E10 (B) S10 (C) E12 (D) S12 (E) None of these

9. There are 10 statements written on a piece of paper:1. At least one of statements 9 and 10 is ture.2. This either is the first true or the first false statement.3. There are three consecutive statements, which are false.4. The difference between the numbers of the last true and the first true statement divides the number, that is to be found.5. The sum of the numbers of the true statement divides the number, that is to be found.6. This is not the last true statement.7. The number of each true statement divides the number, that is to be found.8. The number that is to be found is the percentage of true statements.9. The number of divisors of the number, that is to be found,(apart from 1 and itself) is greater than the sum of the numbers of the true statements.10. There are no three consecutive true statements.

What is the number?(A) 420 (B) 520 (C) 415 (D) 515

10. “Lets have some!” The kids around Betty as she checked the candies. “ Okey, but I’ll have a few myself,” she told them. “It’s by age. A third of them for Bill, a quarter for Eve, a fifth for Linda, and a sixth for Bruce. That leaves just six for me.”

How many were there in all?(A) 200 (B) 180 (C) 120 (D) 90 (E) None of these

11. The question given below has problem and two statements numbered I and II givingcertain information. You have to decide if the information given in the statements is sufficient for answering the problem. Indicate your answer as(A) if the data in statement I alone is sufficient;(B) if the data in statement II alone is sufficient;(C) If the either in I and II alone is sufficient;(D) If the data given in I and II are not sufficient;(E) If the data given in I and II together are needed.

12. “At a party(1) There were 9 men and children(2) There were 2 more women than children(3) The number of different man-woman couples possible was 24.Of the three groups – men, women, and children(4) There were 4 of one group(5) There were 6 of one group(6) There were 8 of one group”

Exactly one of the speaker’s statement is false. Which of (1) through (6) is false(A) 3 (B) 5 (C) 4 (D)2 (E) none of these

Page 11: Quantitative

13. Your pockets are tearing from the weight of all the coins in them. After you unload them onto the kitchen table, you discover something surprising. You have exactly the same number of pennies, nickels, dimes, and quarters, totaling $6.15. How many of each coin do you have?

(A) 10 of each (B) 22 of each(C) 15 of each (D) 20 of each (E) None of these

14. I have ten boxes which I want to pack into crates. Each crate is capable of carrying a maximum of 25 kg. Unfortunately I only have three crates, and the total weight of the boxes is 75 kg: How can I pack the boxes into crates so that each one has exactly 25 kg?Crate 1 Crate 2 Crate 3

(A) 15, 10 11,13,1 9,8,4,2,2 (B) 15, 10 11,8,4,2 13,9,2,1(C) 15,9,1 11,8,4,2 13,10,2 (D) all of these(E) None of these

15. “What day do you go back to school, Henry?” asked his grandmother one day. “Well,” Henry replied, “Nine days ago, the day before yesterday was three weeks before the second day of term.” If Henry had this conversation on a Sunday, what day of the week did he start school?

(A) Monday (B) Wednesday (C) Tuesday (D) Thursday

Satyam Paper At Shri Vishnu Engineering College For Women 1. The mean of the students at one University is 2.4 and standard deviation is 0.8.Then what is the fraction of students who got grading more than 3.0 ? a. 0.75 b. 0.6 c. 0.2266 d. 0.2544 e. None 2. One has invested Rs. 6000 /- in two parts. One part is at 9 % and other part is at 6% to to get same amount . What are the amounts invested ?

a. 2400 3600 b. 3500 2400 c. 3650 2450 d. 2450 3650 e. None

3. There are number of lice dieing with a rate . The information of dieing of lies is given as the mean is 2.2 days and standard deviation is 0.4 days. Can you expect for how many days 90% lice will die?

a. 2.6 b. 3.9 c. 2.5 d. 2.7 e. None

12. 9% of stock yield 18.5%. the market value of the stock is a. 61.50 b. 47.55 c. 54.55 d. 53.55 d.None

6. A m_n borrowed a sum of money and agrees to payoff by paying Rs.3150 at the end of the first year and_RsA410 at the end of the second year. If the rate of compound interest is 5% p.a., find the sum borrowed.

A) Rs.7,000 B) Rs.6, 000 C) Rs.5000 D) Rs. 4000 E) None of these

7. A group consists of equal number of men and women. Of them 10% of men and 45% of women are unemployed. If a person is randomly selected from the group. Find the probability for the selected person to be an employee.

A) 29/40 B) 40/29 C)2 D) 4 E) None of these

Page 12: Quantitative

9. Mr. Srinivas Reddy Travel with bike at 30 km/hr. and reaches his college 7 minutes late. Next day he Travel with bike at 36 kmlhr. and start at the same time, reaches his school 10 min. early. How far is the school from his house?

A) 36 kms B) 30 kms C) 33 kms D) 40 kms E) None of these

10. On the first test of the semester, kiran scored a 60. On the last test of the semester, kiran scored 75. By what percent did kiran's score improve?

A) 12% B) 15% C) 20% D) 25% (E) None of these

11. Ram and Shankar walk up an escalator. The escalator moves at a constant speed, but Ram takes three upward steps for every two of Shankar's upward steps. Ram gets to the top of the escalator after having taken 25 steps, while Shankar takes 20 steps to reach the top. If the escalator were turned off, how many steps would they have to take to walk up?

A) 60 B) 70 C) 50 D) 40 E) None of these

12. Randy's chain of used car dealership sold 16,400 cars in 1998. If the chain sold 15,744 cars in 1999, by what percent did the number of cars sold decrease?

A) 1% B) 4% C) 11% D) 40% E) None of these

4. A man is having some money deposited in bank. In the 1st yr he gets 8% interest, 2nd yr -11 %, 3rd yr- 13%.If he gets Rs 12,400 at the end of three yr. How much he deposited.

5.12 man can complete a work on 12 days .For a work 12 men work for 6 days and then 4 more men joins them .How much more time will be required?

6. A man took Rs 1000 as loan on simple interest of 5%.But he pays Rs 200 annually as interest and amount. How much he will have to return after three years?

8. What are the no. of words that can be formed from 'COOLER' such that both the Os are not together.

9. (This type copied. .question was similar to this) the question is like this: if 1 can be written as # and 0 can be written as % and some data was given only to scared u --------- and after that they gave some examples to explain the theory as1 is written as #2 is written as #%3 is written as ##4 is written as #%%then what is the LCM of 20, 30, 36 5)  A Question on probability when a die and coin tossed what is the probability of getting a head and 6 on top.1. What is the day of 31st July 1957 the date (489)*4+1 year in that see all the odd days in that3. One question is on probability i.e.., 3 lawyers, a teacher, 2 doctors, 4 Advocates making a committee of 4 members what is the probability to choose a person that he is teacher.a) ½ b) ¾ c) 1/3 d) None of These

Page 13: Quantitative

4) A person purchases two items at same price. He sell one at 20% loss, Then how much price he have to sold the second item so for no profit no loss.a) 22% b) 24% c) 28% d) None of these

1. A person invests Rs2000/- on SI of 1 paisa and a rupee for a month. How much he'd get at the end of 4 months. (a) Rs. 2080  (b) Rs. 2070(c) Rs. 2060 - i think this should be the answer (d) Rs. 2000   (e) None of these.

3. Two trains A & B of some length (both different) lengths are traveling (sorry forgot the data). Find the time when the two trains will completely cross each other in opposite direction when the speed of the trains are 42kmph and 48kmph(not sure of the data) and find the time when the two trains will completely cross each other when the trains in the same speed are moving in the same direction.

1. Average weight of p, q, r is given (cannot remember 48 i think) the ratio of students   in class p & q is 3:4.no of students in r is 25% more than p. Find the average of all the 3 classes.

2. Odd man out.  1307, 8627, 6792, 7297. Numbers were not exact but in this model

4. Pie chart problem.  One chart was given on the expenditures of a person. His total income is 6000/- .Now his income is 1500/- we have to find the ratio of radii of two pie charts.       a) 2:1  b) sqrt (2):1  c) 1:sqrt(2)  etc. 5. A, B, C, D, E are 5 in family. A& D are unmarried women & don’t work. In family there are businessman, professor, and artist. B is the brother of a & is not artist or businessman. E is husband.    Who is professor? a) A b) B  c) D d) E  e) NONE OF THESE.

8.  Stamp combination problem.  There are 5, 10, 15 paisa stamps with a boys grandmother and the boy has to paste an equivalent of 45 paisa stamps to post a letter. The total combinations are?

a. 11  b. 12    etc. 9. Some long series of sentences were there I haven’t read them. Some reasoning type.

10.  3 work completion times were given regarding project preparation in hours by 3 teams and if each team person    is equicapable then find the time or days in which the complete some

12. Simple problem on Simple Interest.   If the person gets one fifth of the sum after lending it fo years find the rate of interest

1) Average weight of p, q, r is given (cannot remember) the ratio of students in class p&q is 3:4.no of students in r is 25% more than p. Find the average of all the 3 classes.

Page 14: Quantitative

3) A,B,C,D,E are 5 in family. A& D are unmarried women & don’t work. In family there are businessman, professor, and artist. B is the brother of a & is not artist or businessman. E is husband. Who is professor? a) A b) B c)D d) E e) NONE OF THESE.

4) PIE chart is given for a salary 6000 what will be the ratio of radius with a salary of 1500. a) 2:1 b) 1:2 c)3:2 d)none of these.

5) Sum in S.I. becomes 5 times its principal in 8 years what is the rate? a) 5/2 b) 6/3 c) d) none of these. 1. Find the area of the quadrilateral if the length of the diagonal is 30cms and the length of the lines intersecting the diagonal at right angles are 20cms and25cms?5. A question on compound interest. Something likeA man borrowed Rs X from 2 money lenders for 6 months with an interest of Rs Y and Rs Z. Finally he had to clear a debt which was double the amount that he borrowed. So finally what was the compound interest that has been gained by the lender?

6. An empty fuel tank of a car was filled with type petrol. When the tank was half empty, it was filled with B type petrol. Again when the tank was half-empty , it was filled with A type petrol. When the tank was half empty again, it was filled with B type petrol. What is the percentage of type petrol at present in the tank?

1) What is the angle diff between hr's mint's hands when clock is showing 19:55?a)92           b) 92.5           c)93.5           d)93            e) none

2) If sum has doubled with compound interest in 3 yrs, how many years it will take to sum to become timesa)10           b)9           c)15              d)8            e) none3) If u drive certain dist with 3kmph less speed then it takes 30 mints late . Where as.. You will be 30mints prior if ur speed is 4kmph more. Then whats the usual speed?a)24kmph   b)28kmph       c) 12kmph      d)14kmph      e) none

8) U purchased 2 articles for a total of 2500/- . And u sold one of them for 50% loss and other for 25% profit. How much is ur actual profit or loss %?

Page 15: Quantitative

9) Ten years ago average of a family of 4 members is 24. Now 3 Childs have born in that family out of them is twins and other is 2 yrs younger than these twins. In such case also. The average haven't changed. Now guess the child's ages?a)10,10,12             b)12,12,10            c) 9,9,11 ..... Like wise

1. The ratio of the ages of two persons three yrs ago were 3:4, and the ratio of their ages three yrs hence will be 5:4, what is the sum of their present ages.( numbers not exactly)

3. Cost of basmati rice per kg is Rs 24. Another rice of 25kgs costs 36rs/kg. How many kgs of basmati rice should he mix with that of rice so that after selling the mixture at 40rs/kg he gets a profit of 25%.

4. 1 pyramid problem         C      B    122   A   35    272   7     10    3    I hope u can do this...bottom row.. Its add two elements double them add1 u will get A, but for second row it is add them, double them sub 2 from them u get B, no need to calculate C..U will have unique options so u can tick. 

1. Average weight of p, q, r is given (cannot remember 48 I think) the ratio of students In class p &q is 3:4.no of students in r is 25% more than p. Find the average of all the 3 classes.

2. Odd man out. 1307, 8627, 6792, 7297. Numbers were not exact but in this model    4. Pie chart problem. One chart was given on the expenditures of a person. His total income Is 6000/- .Now his income is 1500/- we have to find the ratio of radii of two pie charts.      a. 2:1 b. sqrt(2):1 c.1:sqrt(2) etc.

5. A, B, C, D, E are 5 in family. A& D are unmarried women & don’t work in family there are Businessman, Professor, artist B is the brother of a & is not artist or businessman. E is husband.    Who is professor?       a) A  b)  B  c)D    d)E        e)NONE OF THIS

8. Stamp combination problem. There are 5,10, 15 paisa stamps with a boys grandmother and the boy has to paste an equivalent of 45 paisa stamps to post a letter. The total combinations are ?      a. 11 b. 12 etc.

Page 16: Quantitative

1. A company increases the bonus 3 times of its 50 employees. What is the % increase in Bonus?

2. A man swims in a river 5 min upstream and 5 min downstream. Starting at A and ending at B. What is the speed of the man?

3. There is enough material to fence 30m linear. Also there is old fence on oneside of the plot. Find the length and breadth of the plot.

4. In a game of Snooker A gives B 18 points for 90, A gives C 10 points for 60. How many can C give B in a game of 70?

5. Gold is 19 times heavier than water. Copper is 9 times heavier than water. In what ratio should they be mixed so that the alloy is 15 times heavier than water?

6. A sphere of radius r is placed inside a cylinder of height 2r which just fits. What is the Volume of the empty space?

7. A is 3 times as old as B. B is 4 years older than C. If C is Z years old express A in terms of Z.

15. Srikanth has borrowed Rs 3000 at 10% C.I.. He has to repay that after 3 years in 3 equal installments. How much does he have to pay as an installment?

16. Given a date and day, the question was to calculate the day of the date June 12, 1979.

17. The weight of a Bucket full of water is 17 kg, the weight of the same bucket with half full water is 13.5 kg. What is the weight of the Bucket?

18. In a party attended by boys and girls where the girls outnumbered the boys Rs 280 was distributed. Each boy was given Rs 10 and each girl Rs 20. How many boys are there?

19. 10% of an army died. 10% of the were ill. 12% of the rest met with an accident. What is the total number of the original army if 7 lakhs were finally left?

20. A started a business with Rs 1700 amount. B joined A after 3 months , C joined after 6 months. If the profit is Rs 1700 and they shared it in the Ratio of 2:3:5 how much did B and C invest?

21. A beats B by 24m. A beats C by 20m. C beats B by 1 sec. In how many seconds does A complete the race of distance 120m.

22. VIJAY is coded as YLMDB. STOP is coded as VWRS. A few questions were given based on this code.

23. 9 3 6 2 ? 1 4 7 8

24. 20 15 10 15 10 5 10 ? 10

25. Which is larger Rs 35 : Rs 1.40 or Rs 48 : Rs 1.44

26. A started a business with Rs 1700 amount. B joined A after 3 months , C joined after 6 months. If the profit is Rs 1700 and they shared it in the Ratio of 2:3:5 how much did B and C invest?

27. In a group of people 600 are Non-veg and 400 are Veg. Of these 150 are both Non-Veg and Veg. In the Non-Veg 300 eat Mutton and 400 eat Chicken.(i) Find the number of people who eat only Non-Veg Mutton.(ii) Who eat only Veg.

Page 17: Quantitative

28. In a wall clock the smaller needle is between 8 and 9 and the larger makes a right angle with it. Find the exact time.

29. Find the figure with the highest Area i)Circle with r=2. Ii)Equilateral Triangle with Side = 4. Iii)Square of diagonal =2. Iv)Triangle with a=8, b=5 and c=4.

30. A boat moves 10 km upstream and returns to the same point in 45minutes. The speed of the stream is 3km/hr. The speed of the boat is ?

31. Given an Equilateral triangle of side 10m. A cow is tied to one end. The length of the rope is 7 m. The area covered by the cow is ?

7 The average of scores of 40 innings of A is 50. The highest score is 178 more than the least score.. If those two are removed then the average of the remaining 38 innings is 40 Then the value of the least score is ..numbers r not same but the model is

8. The sum of the ages of sohan sunil and sanjay is 60 yrs .6 yrs before sohans age was two thirds of sunil and half of sanjay.find the age of sanjay

12. Mmatrix was given x+2 x+3 x+4 x+3 x+5 x+11 x+7 x+12 x+14..find the determinant of the matrix

13 - 16 One problem on arithmetic reasoning like A B C 6 * F E B C D G A F H H _ _ _ _ 8

The letters are not exactly the same but the numbers 6 and 8 are correct.. 4 questions based on this

17. What is the least number when subtracted frm 23 51 37 29 gives the result as 0 I dint understand this question.. But better go thru LCM problems

18 - 21 Some question was given on stocks and shares..it was very big and then 4 questions based on this.

22 One more on trigonometry consisting of some tan@

23. One more on complex numbers consisting of some alpha4beta4+alpha beta..find the complex root24. If X4+3X3+X2+1=0 then find the value of x3+2x2-1 question not exactly same but model is same25. Two points were given (1,0,0) and (1,1,0). Find the perpendicular distance

Page 18: Quantitative

26. Find the length of the wire required to tie around a square 15 times whose area is 30 hectares

27. Find the amount of water that can be filled in a pipe of diameter 8 cm and length some value

In the above two problems the units were a bit complicated like in hectares, acres, cubic meters.. Etc etc..so better be perfect with the conversions

29. There are 500 students out of which 320 eat fish and 250 eat eggs..and 150 eat both..find the number of students who neither eat fish nor eat eggs

3. What day of the week was 26 september 1599

G.D. Topics:

1) Roses are red.2) Genetic cloning.3) Arranged marriage and love marriage.4 dil chahatta hai or lajjathums up or coca colamercy killingxerox and photocopy etcetc were gd topics in our coll

SECTION A) APTITUTE1) day:night:: thena) man:womanb) spring:summerc) light : dark

11) what is the day on 12 january 1979 ?

2) a graph was given and three question was asked on it.

3) | z-2|square + |z-1| = constant . This represent what a) circle b) ellipse c) parabola d) none

4) (4+6w+4w*w)cube - (2+4w+2w*w)square = ? Where w is the complex root of unity

(prepare some question on complex root of 1 .There are 3 question on complex number)

6) there are two pipe ,one fill the tank in 12 hours and other fill in 32 hours .After how long the first pipe is to be closed so as to fill the tank in 16 hours.

7) A sphere is 3cm in radius. It was melted in three sphere out of  which one is 1.5 cm radius and other is 2 cm radius .Find the radius of third sphere

11) A train's to traverse between A &B. If it travels by 50Kmph it got late by 10min If it

Page 19: Quantitative

travels by 36Kmph it got late by 50 min. Find out the speed of the train.

12) I boy's to buy a bike whose cost is 75000. His fatherv gave him 1.5 times what hecan spend and his financial provider gave him 4 time of his father contribution. Find out the Rs. Given by his father.

2) A plane leaves a place A 30 mins late for B which is 1500 km from A. To make up the loss of time it increases the speed by 250 kms . What is its speed originally.(750km) 

3) 25% of alcohol is made 15% by adding water . How much of original solution is there in the final solution .(60%).. 

4) The average marks of 31 students is 40 and if 1 student is removed it gores down to 39. Find the mark of that student(70) . . 

5) A person bought two horses at Rs 240 each . If one was sold at profit of 15% and the other at a loss of 15% find the profit or loss(no profit no loss **) . 

6) If the radius of a circle is increased by 15% how will the area change (32.25**). .

7) If the length of a rectangle is increased by 1% and breadth is decreased by 36% find the change in the area (reduces by 35.36**). 

9) 16 men can do a work in 16 days . After the 4 days of starting the work, more persons joined them. What will be the total time taken?

11)Twoproblems on depriciation11) {x+ (...... ) }/3 =y .;y is the quotient, remainder is zero(all values given) 

14) Given a sum of Rs 4 .in 5 and 50 paise coins ,which combination is not possible ? 

15) If error in length is 10%,breadth is reduced 5%,find the increase or decrease in area? 

4)sum of father and sons age is 48 . 21 years later father will be 3times of sons age. Find their ages. 

5) A pipe A takes 4 hrs and B 6 hrs to fill a tank. At a leakage empties the tank at the rate of 6litres per hour . It takes 40 hrs to fill up the tank with all the pipes and the leakage working together. Find the volume of the tank (19.4594**) . . 

6) In an election 97210 votes were polled . 116 votes are invalid . One person gets 5 votes for every 4 votes the other person gets. What is the margin of win. 

7) One person bought horses for 100 each . He sells one at a profit of15% andother at a loss of 15%. Find the loss or gain he makes. 

8) There is a cone whose top portion is cut in such a way that the ratio of the volume of the cut portion o the left out portion is 1:8 Find the height of the cut

Page 20: Quantitative

portion(**2.08**) . . 

13)There is square ABCD inscribed in a circle of radius Y There is a circle inscribed within this square. Then a square EFGH is inscribed within the circle. Find the side of the innermost square(**y/2**) . . 

14) A sold adio to B at a profit of 25% .B sold it to C at a loss of 5% If C paid .... how much did A pay for it? 

15) Out of 90 persons x like tea ,y like coffee, some like both ...and there isno one who doesn't like none. Find the number of [reasons liking both tea and coffee. Use VENN diagram .(this s not the exact values) 

6.find the greatest no. (a)half 50%of 50(b)3times 40%of 40see the obtion & place 3times 40% of 40 blindly.

7.find the compound interest of 1000 rs. At the rate of 5% p.a. For 15 years.

8.find the greatest of 1000power of 1000,1001power of 999

11.if x is 90% of y then y is how much % of x.

12.the cost of 15 apples & 20 mangoes is much as equal to the 15 mangoes & 20 apple then what is the  relation between their cost..(a)apple is as much equal as mangoes.Place blindly (a) obtion.

16.if a certain money becomes twice in 5 years. then the 300 rs. Will become 2400 at the same rate in how many years........

17.if the average of three numbers is 135.and the difference between others is 25 then find the lowest no.

20.if the lenght of the rectangle is decreased by 4 and breath is increased by 3.then it becomes square whose areas is equal to that of rectangle.what is the perimeter of  the original rectangle.21.there is a hemisphere of radius of 2 cm how much litre will be occupied in the hemisphere.Given 1 litre= 1000 cubic cm.

22.there is a water tank which has enough water to be consumed in 60 days.if there is a leakage water stays there for 40 days.if there are more similar leakage then how lomg water will consumed. 

23.a man saves money 1000 in each year.and he gives this amount at the end of year for the compound interest at  the rate of 5% how much he will save after 25 years.

Page 21: Quantitative

26.the sides of a rectangle is 100 cm and 60 cm if there is a path of  5 cm around the rectanle.it is included in that given rectangle.find the area of that path.......

Q28.a train when travelling from 2 station A and B when travelling a speed of 50km/hr is late by 10min, and when travelling at 30km/hr arrives a late by 50min. Find the diatance.

30.a boat is going along the stream and returning in the opposite  direction of stream it travels  10 km along stream.the sppeed of stream is 3 km/hr,then find the speed of boat.

31.a boy leaves his home 15 minute before his shedule time.He takes 10 minutes to reach the bus stop.he arrive at the stop at  8:40 am when he leaves the home.

32.A and B runing around a circle of perimeter of 1200meter(may be differ).A is runing at 210 meter/minute and bb is running 190 metre/minute in oppsite direction to each other.at wich time they will meet.

39.Amit's D.O.B. is 3rd March 1980. Amit is four day's older than  Sumit. The Republic Day of 1980 is Thrusday. What is the day of Sumit's D.O.B.?

40.A complete a race of 120m taking 4sec less than B. B takes one sec less than C. How much time taken by A to complete the race?

15)If two days from tommorow is thursday then what will be 2 days before yesterday a)Friday b)Saturday c)Sunday d)none of these 

16)What is the day on 1978 jan 12 a)Monday b)Tuesday c)Friday d)wednesday 

In the below question the Facts are not Exactly the same but it's the same type a&b play hockey and football. d&c play football & tennis. B&c play Cricket & tennis c&a play hockey and tennis. Based on these 2 question like 

18) who play all the four games 

19)Who play hockey, tennis and football (i am not sure with the question) 

Here if a is true then answer is c. If both a&b are true then answer is b. If b is true then answer is d. If both are wrong then answer is a. 

1)An unbiased coin is tossed 8 times what is the probability of  getting at least 6 heads. 

3)there are two pipes p,q which can empty a container at 24hr,32hr respectively. both the pipes are opened simultaneously after what time should the first pipe be closed so  that the container is emptied in 16minutes. 

Page 22: Quantitative

5)x+y=40,(1/x)+(1/y)=60.what is the value of x,y 

7.The average of scores of 40 innings of A is 50. The highest score is 178 more than the least score .If those two are removed then the average of the remaining 38 innings is 40 Then the value of the least score is. numbers r not same but the model is

8.The sum of the ages of sohan sunil and sanjay is 60 yrs .6 yrs before sohans age was two thirds of sunil and half of sanjay.find the age of sanjay

12.matrix was given x+2 x+3 x+4x+3 x+5 x+11x+7 x+12 x+14 find the determinant of the matrix

14.What is the least number when subtraced frm 23 51 37 29 gives the result as 0

19.Two points were given (1,0,0) and (1,1,0). Find the perpendicular distance

20.Find the length of the wire required to tie around a square 15 times whose area is 30 hectares

7)if a sphere of dia 3 cm is melted & formed into 3 spheres ,  the diameter of 1st is 1.5cm and that of second is 2.0 cm , what is the  diameter of the third?? 9)there are 6 steps from 1st floor to 2nd floor  A si 2 stes below C  B is next to D  ONly one step is vacant   NO 2 people are on any step  Q!)if a is on the first step , which of the following are true    (( very very easy )) 10) there are 4 novels like thriller , mystery , romance , fiction  written by ,a,b,c,d not necessarily in the same order and published by e , f , g , h, not necessarily in the same order...and questions on this (( very very easy ))

11)6 people compete in a race A,b,c,d,e,f b is not in the 1st place there are 2 runners b/w d and e, a is ahead of d ( one more condition ) and 2 quesitions --very easy   12)if prizes are increased by 25% , by how much should i reduce the  consumption to keep the expenditure same??   13)how many factors ( or what nos) divide 6400   1)24 , 2) 25 3) ....   14) "COURTESY" - how many words can be constructed with C in the begining and Y at the end

Page 23: Quantitative

16)3 glasses containing mixture of water and alco in ratio 2:3 , 3:4 , 5:9 when all 3 are mixed what is the new ratio??

1. the is a meeting organised, every person shake hands with the other only once.if there are 60 shake hands,how many persons are there in the meeting?.2.A fater has 8 children ,he takes 3 at a time to a zoo.probability of a child going to the zoo.3. a father has six children .all the children are born at regular intervels.if the sum of their ages of all the children and father is 186. calculate the age of the elder son, when the younger sons age is 3. 1.(Page 126/ Q-21) THE SPEED OF THREE CAR ARE IN RATIO 2:3:4 THEN RATIO BETWEEN THE TIME TAKEN BY THESE CAR TO TRAVEL THE SAME DISTENCE IS....? 6:4:3

2.(Page 126/ Q-27) IF 30 OXCEN CAN PLOUGH 1/7TH OF A FIELD IN 2 DAYS THEN HOW MANY DAYS 18 OXCEN WILL TAKE TO DO THE REMAINING...?20 DAYS

3.(Page 127/ Q-48) A CASK CONTAIN A MIXTURE OF 49 LTRS OF WINE AND WATER IN RATIO 5:2. HOW MUCH WATER SHOULD BE ADDED TO MAKE WINE : WATER TO 7:4....?

4.WHICH OF THE FOLLOWING NEVER COMES AT THE END OF PERFECT SQUARE OF A NUMBER.. (a) 1 (B)000 (c)2 (D) 6

5.A TANK CAN BE FILL IN 6 HRS BUT DUE TI LIKAGE IT TAKE 4 HRS MORE. IF THE WHOLE TANK IS FULL THEN THE IN WHAT TIME IT WILL BECOME EMPTY?

6.(Page 257/ Q-22) 5 BOYS AND 3 GIRLS R SITTING IN A ROW OF 8 SEATS. FIND NO. OF WAYS SO THAT NOT ALL GIRLS SIT SIDE BY SIDE? 2400

7.(Page 273/ Q-15)TOW FAIR COINS R TOSSED. FINS THE PROBABLITY OF GETTING 1 HEAD AND 1 TAIL? 1/2

8.FIND SUM OF ALL ODD NOS. BETWEEN 100 AND 200.

9.WHAT IS 15TH TERM OF SERIES 20,15,10.....?

10.(Page 92/ Q-48) A MAN SELL A PLOT AT 6% PROFIT. IF HE HAD SOLD IT AT 10% PROFIT HE WOULD RECEIVED RS. 200 MORE. WHAT IS SELLING PRICE OF PLOT.? 5300

11.PROBABLITY OF A IS x (GIVEN) AND B IS y (GIVEN) OF TELLING TRUTH. WHAT IS PROBABLITY THAT BOTH ARE CONTRADICTORY.

12.HALF OF DISTANCE IS TRAVELLED AT 60 KM/HR. THE HALF OF REMAINING HALF AT 30 KM / HR AND REST AT 10 KM/HR. WHAT IS AVERAGE SPEED....?

14.A AND B R TWO INTEGER DIFFER BY 4. THE SUM OF THEIR RECIPROCAL IS 10/20. FIND THE NUMBER? 3

17. A WHEEL AFTER 5000 REVOLUTION COVER 11 KMS. WHAT IS RADIUS OF WHEEL? 35 CMS.

Page 24: Quantitative

3. How many nos start and end with 2 b/w 100 and 300?

4. If a sphere of dia 3 cm is melted & formed into 3 spheres , the diameter of 1st is 1.5cm and that of second is 2.0 cm , what is the diameter of the third?

5. If prizes are increased by 25% , by how much should i reduce the consumption to keep the expenditure same??

6. "COURTESY" - how many words can be constructed with C in the begining and Y at the end

7. My mother's husband's father in laws son's child->what is the relation?

8. 3 glasses containing mixture of water and alco in ratio 2:3 , 3:4 , 5:9 when all 3 are mixed what is the new ratio??

9. A father has 8 children ,he takes 3 at a time to a zoo. probability of a child going to the zoo.

10. A ball is dropped from a height of 10 feet.Every time it rebounces to half of the height. how many feet it travelled?

11. There are 3 jars. The ratio of spirit to water in each of these jars is 3:2,4:5,5:7. the three jars are mixed into a single jar. What is the ration of spirit to water in single jar.

23) there are 2 guards Bal and Pal walking on the side of a wall of a wearhouse (12m X 11m) in opposite directions. They meet at a point and Bal says to Pal "See you again in the other side". After a few moments of walking Bal decides to go back for a smoke but he changes his direction again to his previous one after 10 minutes of walking in the other(opposite) direction remembering that Pal will be waiting for to meet. If Bal and Pal walk 8 and 11 feet respectively, how much distance they would have travelled before meeting again.

24. 13 kigs and 6 libs can produce 510 tors in 10 hrs, 8 kigs and 14 libs can produce 484 tors in 12 hrs. Find the rate of production of tors for kigs and libs. Express the answer in tors/hr.

25) xxx)xxxxx(xxx 3xx ------- xxx x3x ------- xxx 3xx ------- Q) Find the 5 digit No. Hint: 5 is used atleast once in the calculation.

Page 25: Quantitative

26) A fly is there 1 feet below the ceiling right across a wall length is 30m at equal distance from both the ends. There is a spider 1 feet above floor right across the long wall eqidistant from both the ends. If the width of the room is 12m and 12m, what distance is to be travelled by the spider to catch the fly, if it takes the shortest path.

28)Question related to probabilities of removing the red ball from a basket, given that two balls are removed from the basket and the other ball is red. The basket contains blue,red,yellow balls.

29)Venkat has 1boy&2daughters.The product of these children age is 72.The sum of their ages give the door numberof Venkat.Boy is elder of three.Can you tell the ages of all the three.38. There is 66x33m rectangular area. Ram is 11/8 times faster than Krishna. Both of them started walking at opposite ends and they met at some point then, Ram said "See you in the other end" Then they continued walking. After some time Ram thought he will have tea so he turned back walked back 15 meters then he changed his mind again andcontinued walking. How much Krishna has traveled by the time they meet?

41. Write each statements true or false:- 1. The sum of the 1st three statements and the 2nd false statement gives the true statement. 2.The no. of true statements false statement. 3. The sum of 2nd true statement and 1st false statement gives the first true statement. 4. There are at most 3 false statements. 5.There is no two consecutive true statements.

42. There are twelve consecutive flags at an equal interval of distance. A man passes the 8th flag in 8 seconds. How many more seconds will he take to pass the remaining 4 flags?

Page 26: Quantitative

43. A person has to cover the fixed distance through his horses. There are five horses in the cart. They ran at the full potential for the 24 hours continuously at constant speed and then two of the horses ran away to some other direction. So he reached the destination 48 hours behind the schedule. If the five horses would have run 50 miles more, then the person would have been only 24 hours late. Find the distance of the destination.

44. A boat M leaves shore A and at the same time boat B leaves shore B. They move across the river. They met at 500 yards away from A and after that they met 300 yards away from shore B without halting at shores. Find the distance between the shore A & B.

45. A person was going through train from Bombay to Pune. After every five minutes he finds a train coming from opposite direction. Velocity of trains are equal of either direction. If the person reached Pune in one hour then how many trains he saw in the journey?

46. Food grains are to be sent to city from godown. Owner wants to reach the food grains at 11 O' Clock in the city. If a truck travels at a speed of 30km/hr then he will reach the city one hour earlier. If the truck travels at a speed of 20km/h then he will reach the city one hour late. Find the distance between the godown to city. Also with which speed the truck should travel in order to reach at exactly 11 'O clock.

58. Persons A and B. Person A picks a random no. from 1 to 1000. Then person B picks a random no. from 1 to 1000. What is the probability of B getting no. greater then what A has picked?

59. Three boys and three girls brought up together. Jim, Jane, Tom, Virgina, Dorthy, XXX. They marry among themselves to form three couples. Conditions are:- i) Sum of their ages would be the same.ii) Virgina was the oldest.iii) Jim was dorthy's brother.iv) Sum of ages Jane+Jim and Tom+dorthy is same. Give the three couples.

60. X^(1/3) - X^(1/9) =60. Solve for X.

61. X Z Y +X Y Z-------------- Y Z X. Find the three digits.

62. Two boats start from opposite banks of river perpendicular to the shore. One is faster then the other. They meet at 720 yards from one of the ends. After reaching opposite ends they rest for 10mins each. After that they start back. This time on the return journey they meet at 400yards from the other end of the river. Calculate the width of the river.

63. Basketball Tournament organizers decided that two consecutive defeats will knock out the team. There are 51 teams participating. What is the maximum no. of matches that can be played.

Page 27: Quantitative

64. The Master says to his grandmaster that me and my three cousins have ages in prime nos. only. Summation of our ages is 50. Grandmaster who knows the age of the master instantly tells the ages of the three cousins. Tell the ages of three cousins.( 1 is not considered as prime no.)

65. There are two families Alens and smiths. They have two children each. There names are A,B,C,D whose ages are different and ages are less then or equal to 11. The following conditions are given:- i) A's age is three years less then his brother's age . ii) B is eldest among the four. iii) C is half the age of the eldest in Alens family. iv) The difference in sum of the ages of Alens children and smiths children is same as that of five years ago. Find the ages of all the children.

66. a,b,c,d,e are having numerical values. There are some conditions given:- a) a=c <=== b!=eb) Difference between a and c as same as difference between c and b as same as difference between a and d.c) c<a and c>d. Then find a,b,c,d,e.

67. There are six cards in which it has two king cards. all cards are turned down and two cards are opened. a) What is the possibility to get at least one king.b) What is the possibility to get two kings.

68. There are 5 persons a,b,c,d,e and each is wearing a block or white cap on his head. A person can see the caps of the remaining four but can't see his own cap. A person wearing white says true and who wears block says false. i) a says i see 3 whites and 1 block.ii) b says i see 4 blocks.iii) e says i see 4 whites.iv) c says i see 3 blocks and 1 white. Now find the caps weared by a,b,c,d and e.

69. There are two women, kavitha and shamili and two males shyam, aravind who are musicians. Out of these four one is a pianist, one flutist, violinist and drummer. i) Across aravind beats pianist.ii) Across shyam is not a flutist.iii) Kavitha's left is a pianist.iv) Shamili's left is not a drummer. v) Flutist and drummer are married.

70. When Arthur is as old as his father Hailey is now, he shall be 5 times as old as his son Clarke is now. By then, Clarke will be 8 times older than Arthur is now. The combined ages of Hailey and Arthur are 100 years. How old is Clarke? 72. The Jones have named their four boys after favorite relatives; their friends, the Smiths, have done the same thing with their three boys. One of the families has twin

Page 28: Quantitative

boys. From the following clues, can you determine the families of all seven children and their ages? i) Valentine is 4 years older than his twin brothers.ii) Winston, who is 8, and Benedict are not brothers. They are each named after a grandfather.iii) Briscoe is two years younger than his brother Hamilton, But three years older than Dewey.iv) Decatur is 10 years old.v) Benedict is 3 years younger than Valentine; they are not related.vi) The twins are named for uncles.

73. Motorboat A leaves shore P as B leaves Q; they move across the lake at a constant speed. They meet first time 600 yards from P. Each returns from the opposite shore without halting, and they meet 200 yards from. How long is the lake?

74. On the Island of imperfection there is a special road, Logic Lane, on which the houses are usually reserved for the more mathematical inhabitants. Add, Divide and Even live in three different houses on this road (which has houses numbered from 1-50). One of them is a member of the Pukka Tribe, who always tell the truth. Another is a member of the Wotta Tribe, who never tell the truth and the third is a member of the Shalla Tribe, who make statements which are alternately true and false, or false and true. They make statements as follows:-

ADD:1. The number of my house is greater than that of Divide's.2. My number is divisible by 4.3. Even's number differs by 13 from that of one of the others. DIVIDE :1. Add's number is divisible by 12.2. My number is 37.3. Even's number is even. EVEN :1. No one's number is divisible by 10.2. My number is 30.3. Add's number is divisible by 3. Find to which tribe each of them belongs, and the number of each of their houses.

Page 29: Quantitative

75. The names of the inhabitants of Walkie Talkie Land sound strange to the visitors, and they find it difficult to pronounce them, due to their length and a few vowel sounds they contain. The Walkie Talkie guide is discussing the names of four inhabitants –

A,B,C and D. Their names each contain upto eight syllables, although none of the four names contain the same number. Two of the names contain no vowel sounds; one contains one vowel sound; and one contains two vowel sounds. From the Guide's statements below, determine the number of syllables and vowel sounds in each of the four Walkie Talkie names:- i) The one whose name contains two vowel sounds is not A.ii) C's name does not contain more than one vowel sound or fewer than seven syllables.iii) The name with seven syllables does not contain exactly one vowel sound. iv) B and C do not have names with the same number of vowel sounds.v) Neither the name with five syllables nor the name with seven syllables contains more than one vowel sound.vi) Neither the name with six syllables, nor the B's name, contains two vowel sounds.

76. Two identical twins have a very unusual characteristic. One tells nothing but lies on Mondays, Wednesdays and Fridays, and tells nothing but the truth all other days. The other tells nothing but lies on Tuesdays, Thursdays and Saturdays, and tells nothing but the truth all other days. On Sundays both children speak the truth.

77. According to the information presented, which of the following conversations will be impossible. a)Twin A : "Today you are a lier"Twin B : "You are telling the truth" b)Twin A : "Today you are a lier" Twin B : "Today I am a truth teller" c)Twin A : "Tommorow I shall be a lier" Twin B : "That's correct" d)Twin A : "Tommorow you will be a lier" Twin B : "Today you are a truthteller" e)Twin A : "Yesterday we were both truthtellers" Twin B : "You are lying".

78. Assume that the twins followed a different set of rules, so that on a given day both told only the truth while next day both only lied, alternating days of truth telling and lying. Under these rules,which of the following conversations would be possible? a) Twin A : "Today you are a lier" Twin B : "That is correct" b) Twin A : "Today you are a lier" Twin B : "That is not so" c) Twin A : "Tommorow we will be liers" Twin B : "Yesterday we were truthtellers" d) Twin A : "Tommorow we will be liers" Twin B : "You are 1 year older than I am" e) Twin A : "We always tell the truth" Twin B : "We some times tell the truth".

79. If the twins are heard saying the following on the same day, which choice presents a correct statement ?

Page 30: Quantitative

Twin A : "It is Sunday Today"Twin B : "Yesterday was Sunday"Twin A : "it is summer season now" a) it is a summer sunday.b) it is a summer monday.c) it is Monday but not summer.d) it is Sunday but not summer.e) it is impossible to determine whether it is Sunday or Monday.

93) H starts running after T reaches 1/5th they must when H reach 1/6th, if H wants win at what speed H should be run? Note: One circle is there, you show this type of problem.

94) There are 4 mothers, 4 daughters and the colour of their dresses, and they are aged 1, 2, 3 & 4. Details of the dresses are given & then it asked about the remaining dresses.

95) There are 5 levels of dolls and each of different colors & condition are given. Note: This type of problem also refer.

96) 5 student A, B, C, D, E. One student knows 5 languages. Like that up to one langauge. Conditions:- *) Spanish is most popular langauge.*) 3 persons knows Porchigese.*) B & C normally speak English, but when D gathered, they switched to Spanish because that is only common between the three.*) Only langauge common between A, B, E is French.*) Only langauge common between C & E is Italian.

98. Every day a cyclist meets a train at a particular crossing. The road is straignt before the crossing and both are travelling in the same direction. Cyclist travels with a speed of 10 Kmph. One day the cyclist comes late by 25 min. and meets the train 5km before the crossing. What is the speed of the train. 60 kmph. 99. Five persons muckerjee, misra, iyer, patil and sharma, all take then first or middle names in the full names. There are 4 persons having first or middle name of kumar, 3 persons with mohan, 2 persons with dev and 1 anil. -- Either mukherjee and patil have a first or middle name of dev or misra and iyer have their first or middle name of dev.-- Of mukherkjee and misre, either both of them have a first or middle name of mohan or neither have a first or middle name of mohan.-- Either iyer of sharma has a first or middle name of kumar but not both. Who has the first or middle name of anil? Today is Mukherjee.

Page 31: Quantitative

101. Two turns have vertain peculiar characteristics. One of them always lies on Monday, Wednesday, Friday. The other always lies on Tuesdays, thursdays and saturdays. On the other days they tell the truth. You are given a conversation. Person A -- Today is sunday and my name is anil. Person B -- Today is tuesday and my name is Bill. What is today? Today is tuesday.

102) Which of the following statements can be deduced from the information presented? i) If it is Sunday, the twins will both say so.ii) If it is not Sunday, one twin will give the correct day and the other will lie about everything.iii) On any given day, only one twin will give his correct name. a) i only. b) i and ii only. c) i and iii only. d) ii and iii only. e) i,ii and iii.

103) If the twins are heard saying the following on the same day, which choice presents a correct statement? Twin A : "It is Sunday Today".Twin B : "Yesterday was Sunday".Twin A : "it is summer season now". a) It is a summer sunday.b) It is a summer monday.c) It is Monday but not summer. d) It is Sunday but not summer.e) It is impossible to determine whether it is Sunday or Monday.

105. There is a robery and four persons are suspected out of them one is actual thief, these are the sentences said by each one of them!

A says D had done B says A had doneC says i dddnt done D B lied when he said that i am thief

108. Conversation between two employees is as follows:- EMPLOYEE-1: Hello! Now your experience is twice the my experience.EMPLOYEE-2: Exactly two times.EMPLOYEE-1: But at the last meet, you said that your experience is thrice of my experience.EMPLOYEE-2: That is when we met at 2 years back, your experience is thrice that of yours.

110.There are 3 women ,they having three jewells, named diamond emerald, ruby 3 women A,B,C 3 thiefs D,E,F each they had taken one jewel from each of the women following conditions one who had taken diamond is the bachelor and most dangerous D 's brother in law E who is less dangerous than the thief who had stolen emerald (this is the key from this e had stolen ruby D did nt stolen from B one more condition is there

111. there were three suspects for a robbery that happend in a bank, tommy, joy and bruceEach of them were saying that I haven't done anything and the other two has done it.police found that tommy was lying .who is the thief. 3M (MARKS).

112. Joe started from bombay towards pune and her friend julie in opposite direction. they meet at a point .distance travelled by joe was 1.8 miles more than that of julie.after

Page 32: Quantitative

spending some both started there way. joe reaches in 2 hours while julie in 3.5 ours. Assuming both were travelling with constant speed.Wath is the distance between the two cities.

115. Imagine a rectangle. Its length = 2*width. A square of 1 inch is cut on all corners so that the remaining portion forms a box when folded. The volume of the box is _____ cubic inches. Find the original dimensions of the box.

116. 2 persons are doing part time job in a company say A and B. THe company is open for all the 7 days of the week. 'A' works every second day. 'B' works every 3rd day. If 'A'works on first june and 'B' works on second june. Find out the date on which both 'A' and 'B' will work together.

117. Consider a pile of Diamonds on a table. A thief enters and steals 1/2 of th e total quanity and then again 2 extra from the remaining. After some time a second thief enters and steals 1/2 of the remaining+2. Then 3rd thief enters and steals 1/2 of the remaining+2. Then 4th thief enters and steals 1/2 of the remaining+2. When the 5th one enters he finds 1 diamond on the table. Find out the total no. of diamonds originally on the table before the 1st thief entered.

128.There are some chicken in a poultry. They are fed with corn One sack of corn will come for 9 days. The farmer decides to sell some chicken and wanted to hold 12 chicken with him. He cuts the feed by 10% and sack of corn comes for 30 days. So initially how many chicken are there? 129.Two people X & Y walk on the wall of a godown in opposite direction. They meet at a point on one side and then go ahead. X after walking for some time, walks in opposite direction for 15 mtrs. Then again he turns back and walks in the original direction. What distance did Y walk before they met again, if X walks 11 mtrs by the time Y walks 8 mtrs.

135. 4 persons are there caledd JOHN, JACOB, PITER, and WILLIAMS. 4 laungages are there named ENGLISH, ITALIAN, GERMAN, FRENCH. Conditions:- a. There is no common language for all.b. Except one language, no language is spoken by more than two.c. One can know either German or FRENCH but not both.d. John can't speak ENGLISH But John can act as interpreter between JACOB and PITER. Jacob knows GERMAN but he can tailk with WILLIAM who doesn't know a word of GERMAN.e. No common language between JOHN, PITER, and WILLIAMS. Which two languages does each person speaks?

Hint: ITALIAN IS SPOKEN BY THREE PERSONS (This hint is given in Question paper)

137. I participated in a race.1/5th of those who are before me are equal to 5/6th of those behind me. What were the total number of contestants in the race?

138. Find the 3 digit number. Third digit is square root of first digit. Second digit is sum of first and third digits. Find the number.

Page 33: Quantitative

139. This problem is of time and work type. Some A and some B are able to produce so many tors in so many hours.(for example 10 A and 20 B are able to produce 30 tors per hour). Like this one more sentence was given. We have to find out the rate of working of A and B in tors/hour.

140. A and B play a game of dice between them. The dice consists of colors on their faces instead of numbers. A wins if both dice show same color. B wins if both dice show different colors. One dice consists of 1 red and 5 blue. What must be the color in the faces of other dice.(i.e how many blue and how many red?). Chances of winning for A and B are even.

141. A girl has 55 marbles. She arranges them in n rows. The nth row consists of n marbles, the (n-1)th row consists of (n-1) marbles and so on. What are the number of marbles in nth row?144) Every station in N railroad issues every other station's ticket. Some stations are added. Now they have to issue 46 more tickets. Give the No. of stations after and before added.

145) There was a race between 3 people. Me, Doug and Anne. When I take 21 steps the distance covered is equal to Doug's 24 steps and Anne's 28 steps. I take 6 steps to every 7 steps of Doug and 8 steps of Anne. Who won the race?

146)How many bearers will an explorer need for a 6 day march if each man can carry the food stuff of one man for 4 days.

147) Consider the following statements:Albert: Dave did it.Dave: Tony did it.Gug: I did not do it.Tony: Dave lied when he said that i did it.(a)If only one out of all above statements is true, who did it?(b)If only one out of all above statements is false, who did it?

148) A contribution of Rs. 500 was raisedfrom 500 people. The fee was as follows: Men: Rs.3.00 eachWomen: Rs. 2.00 eachChildern: 0.48 eachIf number of women is more than number of men, how many childern are there?

149) Alice and Liu had some berries. The total of Alice's berries and square of number of berries with Liu is 62. The total of Liu's berries and square of number of berries with Alice is 176. How many berries does each of them have?

150) A rope ladder was left down from a ship. 12 steps of the ladder were exposed at 10:00 am. The queen who was going to visit the ship, said she would visit at 1:00 pm as she would have to climb lesser number of steps then. The tide in the sea increases from morning to afternoon at the rate of 1.2 meters per hour. The distance between any 2 steps of the ladder is 0.4 mts. How many steps will the queen have to climb?

Page 34: Quantitative

151) 5 hunters Doe, Deer, Hare, Boarand Row kill 5 animals. Each hunter kills an animal that does not correspond to his name. Also each hunter misses a different animal which again does not correspond to his name.a) The Deer is killed by the hunter, known by the name of the animal killed by Boar.b) Doe is killed by the hunter, known by name of animal missed by Hare.c) The Deer was disappointed to kill only a Hare and missed the Roe.

154) A Man is sitting in the last coach of train could not find a seat, so he starts walking to the front coach ,he walks for 5 min and reaches front coach. Not finding a seat he walks back to last coach and when he reaches there, train had completed 5 miles. What is the speed of the train. 155) The Old car of Mary requires tyres to be changed after each 24000 km. If she wants to go for 42000 km journey then how many minimum number of tyres she will need.

156) A coin is so unbalanced that it may come both heads in 2 tosses as it may come tails in a single toss. What is the probabality of getting a head in a single toss.

157) A pen, pencil and eraser together cost $1.00. if 2E<N , if N>2P, and 3P>4E then what a single pen will cost?

158) A local forecast service has accuracy of 2/3 says No rain , and Meteriological service having accuracy of 4/3 says Rain. if Preference is as no rain what is the chance of rain?

159) Sherlock holmes thrwated the plan to kidnapp Mrs mary when they were questioned Mercy and his two associated shipy and rany.when they were telling the story one of them told one thing wrong and other true, the other told both true, and the last told both false. examining the following tell the roles played by eachMercy:: 1) i wrote the ransome note 2) shipy broke into the window rany 1) shipy wrote the ransome note 2) mercy ran away with the lady shipy 1)i broke into the window 2)rany wrote the ran some note.

160) Tom asked kim did you like the stamps? She said yes ,me and rob too liked them. Kim again said that rob got 3 more than he would have got, if i would have kept 2 more than, what he got. Tom asked how many u gave Rob? She replied 2 more than what I got. Tell, how many stamps each rob and kim got?

Page 35: Quantitative

161) The virgo club members used to meet every week to play cards. Each time they used to seat around a round table and for their memory they used all the possible combinations of postions each for a single time only. Can you tell for how many times they met? 173. Ram Singh goes to his office in the city, every day from his suburban house. His driver Gangaram drops him at the railway station in the morning and picks him up in the evening. Every evening Ram Singh reaches the station at 5 O' Clock. Gangaram also reaches at the same time. One day Ram Singh started early from his office and came to the station at 4 O' Clock. Not wanting to wait for the car he starts walking home. Mangaram starts at normal time, picks him up on the way and takes him back house, half an hour early. How much time did Ram Singh walk? 176. Albert and Fernandes have two leg swimming race. Both start from opposite ends of the pool. On the first leg, the boys pass each other at 18 m from the deep end of the pool. During the second leg they pass at 10 m from the shallow end of the pool. Both go at constant speed but one of them is faster. Each boy rests for 4 seconds at the end of the first leg. What is the length of the pool? 177. Each alphabet stands for one digit in the following multiplication. T H I S x I S --------- X F X X X X U X ------------ X X N X X ------------ What is the maximum value T can take?

178. If 1/4 of the time from midnight plus 1/2 of the time from now to midnight is the present time, then What is the present time? 2. In a 10 digit number, if the 1st digit number is the number of ones,2nd digit number is the number of twos, and ... so on. 10th digit is the number of zeroes, then find the number.

179. A train blows a siren one hour after starting from the station. After that it travels at 3/5th of its speed it reaches the next station 2 hours behind schedule. If it had a problem 50 miles farther from the previous case,it would have reached 40 minutes sooner. Find the distance between the two stations.

180. An army 50 miles long marches at a constant rate. A courier standing at the rear moves forward and delivers the message to the first person and then turns back and reaches the rear of the army as the army completes 50 miles. Find the distance travelled by the courier.

181. Olympic race : 4 contestants : Alan,charlie, Darren ,Brain. There are two races and average is taken to decide the winner. One person comes at the same position in both the race. Charlie always come before Darren. Brian comes first once. Alan comes third atleast once. Find the positions. Alan never comes last. Charlie & Darren comes 2nd atleast once. 184) There were 50 players playing a game among themselves. Each player is out of the game when lose 3 matches. What is the number of matches should be played in order to get the winner.

Page 36: Quantitative

185) A & B two places. C & D are two people. C started from A and D started from B. When they meet each other in the way C traveled 18 m more than D. Then C takes 13 and half a minute and D takes 24 minutes to reach the other end. What was the distance between A & B.

186) I have been hearing a girl singing a song for last two score. Song: If seven times five and three times seven is added to my age it would be as far above six nines and four as the difference between twice of my age and a score. Given-A score is 20 yrs.

187) A tourist wants to go from A to B. There are four ways to do this:- 1. To take a wagon. The wagon stops for half an hour at a station in between a & b and then goes to b. 2. To walk to B. If he leavs A at the same time the wagon leaves, he will be between by the wagon by 1 mile to reach B. 3. To walk from A at the same time the wagon leaves from A. He will arrive at the mid station at the time when the wagon is prepared to leave. He can take the wagon from there. This will take shortest time. 4. To go on upto the mid station & to walk from there. He will reach at B 15 minutes before the wagon. What is the distance between A & B?.

190) 4 ladis, Mrs Margarat, Mrs Price, Mrs Winter & Mrs Ellen went for marketing. Each went for 2 shops only. Their surnames are lorret, torrey, doris and marshall. One went to a hardwares shop. Two went to bank. Two went to buchers. All but dorris went to grocery etc. Who went where?

191). A software engineer starts from home at 3 pm for evening walk. He walks at a speed of 4 kmph on level ground and then at a speed of 3 kmph on the uphill and then down the hill at a speed of 6 kmph to the level ground and then at a speed of 4 kmph to the home at 9 pm. What is the distance on one way?

192). A bag contains certain number of files. Each file is numbered with one digit of 0 to 9. Suppose the person want to get the number between 1 to 2000 (or 7000 check ). How many minimum number of files should be present in the bag.

198. Answer the questions from facts:- The members of certain tribe are divided into 3 casts abhor, dravid amp; magar. 1. An abhor woman can't marry dravid man.2. A magar woman can't marry a dravid man.3. A son takes the caste of his father and a daughter takes caste of her mother.5. All marriages except those mentioned, are not permitted.6. There are no children born out of a wedlock.

155) The Old car of Mary requires tyres to be changed after each 24000 km. If she wants to go for 42000 km journey then how many minimum number of tyres she will need.

156) A coin is so unbalanced that it may come both heads in 2 tosses as it may come tails in a single toss. What is the probabality of getting a head in a single toss.

Page 37: Quantitative

157) A pen, pencil and eraser together cost $1.00. if 2E<N , if N>2P, and 3P>4E then what a single pen will cost?

158) A local forecast service has accuracy of 2/3 says No rain , and Meteriological service having accuracy of 4/3 says Rain. if Preference is as no rain what is the chance of rain?

159) Sherlock holmes thrwated the plan to kidnapp Mrs mary when they were questioned Mercy and his two associated shipy and rany.when they were telling the story one of them told one thing wrong and other true, the other told both true, and the last told both false. examining the following tell the roles played by eachMercy:: 1) i wrote the ransome note 2) shipy broke into the window rany 1) shipy wrote the ransome note 2) mercy ran away with the lady shipy 1)i broke into the window 2)rany wrote the ran some note.

160) Tom asked kim did you like the stamps? She said yes ,me and rob too liked them. Kim again said that rob got 3 more than he would have got, if i would have kept 2 more than, what he got. Tom asked how many u gave Rob? She replied 2 more than what I got. Tell, how many stamps each rob and kim got? 161) The virgo club members used to meet every week to play cards. Each time they used to seat around a round table and for their memory they used all the possible combinations of postions each for a single time only. Can you tell for how many times they met? 173. Ram Singh goes to his office in the city, every day from his suburban house. His driver Gangaram drops him at the railway station in the morning and picks him up in the evening. Every evening Ram Singh reaches the station at 5 O' Clock. Gangaram also reaches at the same time. One day Ram Singh started early from his office and came to the station at 4 O' Clock. Not wanting to wait for the car he starts walking home. Mangaram starts at normal time, picks him up on the way and takes him back house, half an hour early. How much time did Ram Singh walk? 176. Albert and Fernandes have two leg swimming race. Both start from opposite ends of the pool. On the first leg, the boys pass each other at 18 m from the deep end of the pool. During the second leg they pass at 10 m from the shallow end of the pool. Both go at constant speed but one of them is faster. Each boy rests for 4 seconds at the end of the first leg. What is the length of the pool?

177. Each alphabet stands for one digit in the following multiplication. T H I S x I S --------- X F X X X X U X ------------ X X N X X ------------ What is the maximum value T can take?

Page 38: Quantitative

178. If 1/4 of the time from midnight plus 1/2 of the time from now to midnight is the present time, then What is the present time? 2. In a 10 digit number, if the 1st digit number is the number of ones,2nd digit number is the number of twos, and ... so on. 10th digit is the number of zeroes, then find the number.

179. A train blows a siren one hour after starting from the station. After that it travels at 3/5th of its speed it reaches the next station 2 hours behind schedule. If it had a problem 50 miles farther from the previous case,it would have reached 40 minutes sooner. Find the distance between the two stations.

180. An army 50 miles long marches at a constant rate. A courier standing at the rear moves forward and delivers the message to the first person and then turns back and reaches the rear of the army as the army completes 50 miles. Find the distance travelled by the courier.

181. Olympic race : 4 contestants : Alan, charlie, Darren ,Brain. There are two races and average is taken to decide the winner. One person comes at the same position in both the race. Charlie always come before Darren. Brian comes first once. Alan comes third atleast once. Find the positions. Alan never comes last. Charlie & Darren comes 2nd atleast once. 184) There were 50 players playing a game among themselves. Each player is out of the game when lose 3 matches. What is the number of matches should be played in order to get the winner.

185) A & B two places. C & D are two people. C started from A and D started from B. When they meet each other in the way C traveled 18 m more than D. Then C takes 13 and half a minute and D takes 24 minutes to reach the other end. What was the distance between A & B.

186) I have been hearing a girl singing a song for last two score. Song: If seven times five and three times seven is added to my age it would be as far above six nines and four as the difference between twice of my age and a score. Given-A score is 20 yrs.

Page 39: Quantitative

187) A tourist wants to go from A to B. There are four ways to do this:- 1. To take a wagon. The wagon stops for half an hour at a station in between a & b and then goes to b. 2. To walk to B. If he leavs A at the same time the wagon leaves, he will be between by the wagon by 1 mile to reach B. 3. To walk from A at the same time the wagon leaves from A. He will arrive at the mid station at the time when the wagon is prepared to leave. He can take the wagon from there. This will take shortest time. 4. To go on upto the mid station & to walk from there. He will reach at B 15 minutes before the wagon. What is the distance between A & B?.

190) 4 ladis, Mrs Margarat, Mrs Price, Mrs Winter & Mrs Ellen went for marketing. Each went for 2 shops only. Their surnames are lorret, torrey, doris and marshall. One went to a hardwares shop. Two went to bank. Two went to buchers. All but dorris went to grocery etc. Who went where?

191). A software engineer starts from home at 3 pm for evening walk. He walks at a speed of 4 kmph on level ground and then at a speed of 3 kmph on the uphill and then down the hill at a speed of 6 kmph to the level ground and then at a speed of 4 kmph to the home at 9 pm. What is the distance on one way?

192). A bag contains certain number of files. Each file is numbered with one digit of 0 to 9. Suppose the person want to get the number between 1 to 2000 (or 7000 check ). How many minimum number of files should be present in the bag.

198. Answer the questions from facts:- The members of certain tribe are divided into 3 casts abhor, dravid amp; magar. 1. An abhor woman can't marry dravid man.2. A magar woman can't marry a dravid man.3. A son takes the caste of his father and a daughter takes caste of her mother.5. All marriages except those mentioned, are not permitted.6. There are no children born out of a wedlock.

iGATE

a. what is the sum of the nos b/w 100 to 1000 that  are divisible by 14?b. if a boy travels 5km at 30kmph and 6km at 24kmph what is its avg speed?c. what is the value,by which radius and height hould be increased so that volume of the cylinder remains the same,given that radius is 3cm and height is 5cm?d. if in 49litre of mixture contais water and milk in the ratio2:7 ,and after that 14litre of pure milk was added ,what is the new ratio?if day before yesterday was sunday then what will be the day after tomorrow?e. if a can do work in 15 days ,b can co work in 10 days ,if a,b and c started doin work, and they completed it in 5 days ,there master paid them 720 rs find the share of c?f. if in a hostel there are 36 boys and thery have some expenditure spent over them,if no of boys increased by 4 and expediture is increased by 4 and then expeniture per head is decreased by 1,then find the original expenditure?g. if  a wheel traveled 1.1 km in 560 revolutions then what is the area of the wheel?

Page 40: Quantitative

h. if a train 1km long crossed aplatform 3km long in 10min then what is the speed of that train?

4.aa person sold two articles for 80 /- each. with 20% profit on one article and 20% loss on another article, what is the loss / prifit he will gain on both.

9) on eman walked 1/4,3 1/4,7 1/4,9 1/4 and other man walked 2/4,2 1/4,  2 2/3,3 1/4 kms. Who walked  more and by how much

1. It was based on hour and minute hand. If the diff. between hour and minute hand is 6 minute then after how much time the diff. will be 7 min. It was quite tricky.

2. If 6 weights and one pan is given then what are the weights exactly to measure 1-364 candies. 3. Divide 170 in 3 parts such a way that first part is 10 more than second part. 4. There is a 4 digit no. in which 3rd digit is twice of first one. Second digit is the sum of first and third. Fourth digit is twice of third. What is the no.? 5. A circle is touched by 34 tangents then what is the total no. of unclosed regions? 6. A room having floor dimension of 48 feet by 24 feet has cross diagonal length of 54 feet. What is the height of the room? 7. One year ago pandit was 3 times older than a girl and after one year he is 2 times older than the girl. What is the age of pandit after 5 years. 8. A brick weighs 4kg. How many bricks will be formed by reducing its dimension to one-fourth? 9. In a family every child have 3 brother and 3 sister, then how many children are all there? 10. A dog says that there are 2 dogs in front of him, other dog says that there 2 dogs behind him. How many dogs are all there? 11. In a telephone exchange if area code is of three digits, then how area codes are possible for that exchange from a-z alphabets only?

12. If a girl has 10z more chocolates then she will have 5y+1 times chocolates than she originally had. Find the no. of chocolates the girl had originally in terms of y and z.

2. A man took 5000 rs as loan for 10 years. for first 3 yrs he has to pay 3 %,next 2yrs 3.5%, for rest 4 % as SI. What amount is he supoosed to give back at the completion of 10 years? 5. a club has 4 couples as members.Form a group of 3 members such that no two couple members be on that group? 3. A train is coming at some speed and the other train is coming in opposite direction with some other speed, a bird is flying in between the trains touching each end and the bird is also flying at some speed, what is the distance covered by the bird, when both trains meet each other?( I have forgetted the data ) 4. A man has two children and atleast 1 girl child, what is the probability that both his children are girls?

5. A profit and Loss problem where the gain percentage with respect to Selling price is given we have to calculate the gain percentage with respect to the Cost price

Page 41: Quantitative

6. There were some red balls and blue and green balls what percent of probability of selecting  6 balls out of which we have 2 balls of the  same color.

3.    The angle in a triangle r in the ration ratio of 2:3:4. Can u find out the smallest angle of the triangle. (40)  4.   4 white balls & 8 blue ball in the bucket 1 and in bucket 2 there were 4 white and 8 black balls. Find the probability of taking the white balls when one ball is drawn from each bucket. (either 2/3 or 1/3)  5.    Mother says to a boy to bring 9 marble from the market. But boy can bring only 2 marble at a time. Can you say how many trip to market the boy had to go to bring 9 marbles. (5)  6.    One fish tank is half full. After adding 10 gallons the tank is 7/8 of the tank. Calculate the capacity of the fish tank.(26.66)  7.    Mr. X, Mr. Y, Mr. Z collect Rs. 1,00,000 in the ratio of 2:3:5. Can U say how much extra money is given by Mr. Z than Mr. X? (30,000)  8.)    One boy is having some coins and after adding 10z coin the boy is now having (5y-1) times of the coin he had at the beginning. Calculate the coins the boy had initially in terms of y & z. (2z/y)  9.)    A shopkeeper gives a discount of 20% on the sale. By what percent he had to increase the S.P of the item so that after giving discount he got the price of the its C.P.  11.)  A fashion designer and sold it to retailer at the profit of 40% and retailer sold it customer at a profit of 25%.Can u say how much extra money paid by the customer from the cost manufacturing. Calculate the price in percentage.  12.)  A taxi driver charges 1.75$ for first quarter of mile and then charges 15 cents for each quarter. How much distance can be covered in 4.9$. (5.4 mile)  13.) Out of 170 baseball player 95 were right handed. Can u calculate the percent of players who were not right handed.  14.) 8 eggs were identical except one is lighter. How many times we have to weight to find out to find out the lighter. (3)  15.) Out of 35 students if the ratio of the girls to boys is 4:3. Then by what number the girls are more than boys. (5)

APTITUDE QUESTIONS:-

1. Last month of an year (a) January (b) February  (c) December (d) November 

Page 42: Quantitative

2. Select the odd one  (a) January (b) February (c) Wednesday  (d) November 

12. Select the odd one out. (a) 1/4 (b) 1/3 (c) 1/6  (d) 1/18 

13. Select the least from the following. (a) 0.99 (b) 1  (c) 81  (d) 0.333 

1. One dollar is saved in one month. Then how much dollar is saved in one day? A.   ?    0.0333$   ?    0.333$ B.   ?    0.00333$   ?    0.3333$2. If a train covers 600m in 0.5 seconds, how long it will cover in 10 seconds? A.   ?    3000 m  ?    7600 m B.   ?    4000 m   ?    3500 m3. Y catches 5 times more fishes than X. If total number of fishes caught by X and Y is 42, then number of fishes caught by X? A.   ?    8   ?    7 B.   ?    10   ?    94. One skirt requires 3.75 yards of cloth. How many Skirts you can make from 45 yards?A.   ?    14 skirts   ?    16 skirts B.   ?    None of the Given Answers   ?    12 skirts5. Sister's age is twice than that of the brother. If the brother's age is six, what is the sister's age after two years? A.   ?    18 years  ?    21 years   ?    20 years ?    14 years6. A clock is late by 1 minute 27 seconds in a month. Then how much will it be late in 1 day? A.   ?    2.9 seconds   ?    3.0 seconds B.   ?    270 seconds   ?    2.0 seconds7. The girl's age is twice that of boy, if the boy is four years old. After four years the age

of the girl is A.   ?    14 years ?    12 years B.   ?    16 years   ?    None of the Given Answers8. Two lemons cost 10 cents. Then one and a half dozen cost A.   ?    190 cents ?    90 cents   ?    20 cents  ?    198 cents

1.In a right angled isosceles triangle ABC,where angle B = 90 degrees,BM is the median on AC,then 1.AB2+BC2=4BM2 2.AB2+BC2=2BM2 3.AB2+BC2=BM2 4.AB2+BC2=7BM2

2.A sum of money put at compound interest amounts to Rs.2205 in 2 years and to Rs.2315.25 in 3 years.Find the rate of interest per annum is : 1.10% 2.5% 3.8% 4.6% 5.None of these

3.If A:B=2:3 and B:C=5:6 then A:B:C is : 1. 15:10:18 2. 10:18:15 3. 18:15:10 4. 10:15:18

Page 43: Quantitative

4.The difference between simple interest and compound interest on Rs.P at R% p.a in 2 years is : 1. PR/100 2. 2PR/100 3. PR2/100 4. PR2/(100)2

5.The rate at which Rs.64 will amount to Rs.125 in 3 years at compound interest will be: 1. 15% 2. 25% 3. 20% 4. 50%

8. A shopkeeper marks up his goods to gain 35%.But he allows 10% discount for cash payment. His profit on the cash transaction therefore in percentage is: 1.13 ½ 2.21 ½ 3.25 4.31 ½ 5.none

9.Fine the positive value of n when the sum of 5+7+9+. . . .=140. 1. 10 2. 8 3. 6 4. 510. The average age of an adult class is 30 years. 12 new students with an average age of 32 years join the class, thereby increasing the average age of the class by one year. The original strength of the class was:1. 8 2. 10 3. 12 4. 30

11.The mean of 100 observations was calculated as 40. If it was found later that one of the observations was mis-read as 83 instead of 53, the correct mean is : 1. 39.0 2. 39.7 3. 40.3 4. 42.7

12. The average of the reciprocals of x and y is : 1. 2xy/(x+y) 2. 2(x+y)/xy 3. (x+y)/(2xy) 4. (x+y)/(x-y)

13. If the length of a rectangle is increased by 50% and its breadth is decreased by 25% what is the percentage change in its area: 1. 12.5% increase 2. 10% increase 3. 25% increase 4. 20% decrease14. A cone and a sphere have equal radii and equal volumes. The ratio of the diameter of the sphere to the height of the cone is : 1. 3:1 2. 1:3 3. 6:1 4. 1:2

15. Three equal circles are described with vertices of the triangle as centers. If the radius of each circle is r, the sum of areas of the portions of the circles intercepted in a triangle is : 1. 2*pi*r*r 2. (3/2)*pi*r*r 3. pi*r*r 4. (1/2)*pi*r*r

16. A candidate was asked to find 7/8 of a positive number. He found 7/18th of the same by mistake. If his answer was 770 less than the correct one, then the original given number was: 1. 1260 2. 6160 3. 1548 4. None17. The remainder obtained when any prime number, greater than 6, is divided by 6 must be: 1. either 1 or 2 2. either 1 or 3 3. either 1 or 5 4. either 3 or 5

18. A certain number of 2 digits, is four times the sum of its digits. If 9 were added to the number, the digits are reversed. Find the number: 1. 15 2. 12 3. 21 4. 51

Page 44: Quantitative

19. The majority against a certain motion is equal to 8% of the total number of voting. If 14 of those who voted against it had voted for it, the motion would have been carried by 4 votes. Find the number voting 'for' and 'against':1. for:138,against:162 2. for:160,against:185 3. for:130,against:170 4. for:162,against:138

20. A bus traveled from station A to station B at a speed of u/1 kmph and traveled back from B to A at u/2 kmph. What is the average speed of the bus: 1. (1/u/1 + 1/u/2)/2 2. (u/1 + u/2)/2 3. 2u/1.u/2/(u/1+u/2) 4. 2u/1.u/2 /(u/1-u/2)

21. The cost price of an article, which on being sold at a gain of 12% yield RS.6 more than when it is sold at a loss of 12%, is:1. Rs:24 2. Rs:25 3. Rs:30 4. Rs:20

22. The price of sugar increases by 32%. A family reduces its consumption so that the expenditure of the sugar is up only by 10%. If the total consumption of the sugar before the price-rise was 10 Kg per month, then the consumption of sugar per month at present (in Kg) is: 1. 8 1/3 2. 8 ½ 3. 8 ¾ 4. 9

23. An article is sold at 12.5% profit. If it had been sold at 5% loss, the selling price would have been Rs.9.80 less. The cost price of the article is:1. Rs.58 2. Rs.56 3. Rs.54 4. Rs.46

24. Two friends invested Rs.1500 and Rs.2500 in a business. They earned a profit of Rs.800. one half of the profit was divided equally between them and other half was divided in proportion of their capitals. How much did each of them receive? 1. Rs.370 and Rs.430 2. Rs.350 and Rs.4503. Rs.360 and Rs.440 4. Rs.375 and Rs.42525. ABCD is a squared, E is the mid-point of BC and F is the mid-point of CD. The ratio of the area of triangle AEF to the area of the square ABCD is:1. 1:4 2. 1:3 3. 3:8 4. 1:2

26. ABC is an equilateral triangle inscribed in a circle. P is any point on the minor arc BC. Then: 1. PA < (PB+PC) 2. PA > (PB+PC) 3. PA = PB+PC 4. PA = 1/2 (PB+PC)

27. In a triangle ABC , a line XY is drawn parallel to BC meeting AB in X and AC in Y. The area of the triangle AXY is half of the area of the triangle ABC.XY divides AB in the ratio of : 1. 1 : root2 2. root2 : (root2-1) 3. 1 : (root2-1) 4. root2 : root3

28.Five mangoes and four oranges cost as much as three mangoes and seven oranges.What is the ratio of the cost of one mango to the cost of one orange? 1. 4:3 2. 1:3 3. 3:2 4. 5:2

29.If x men working x hours a day for x days produce x articles, then the number of articles produced by y men working y hours a day for y days is : 1.(x*x*x)/(y*y*y) 2. (x*x)/(y*y*y) 3. (y*y*y)/(x*x) 4. (x*x)/(y*y)

Page 45: Quantitative

30. x,y,z,u are real numbers such that x:y = y:z=z:u and x:u =64:27.The value of x:z is: 1. 64:27 2. 16:9 3. 4:3 4. 3:4

31.A man borrowed Rs.1000 at 5% per annum at simple interest.Each year he pays Rs.200 for interest and part of debt.The amount that is left to be paid back in the end of 3 years is :1.Rs.850.00 2. Rs.826.13 3. Rs.692.50 4. Rs.527.13

32.A monkey climbing up a pole ascends 6 meters and slips 3 meters in alternate minutes.This way he continues climbing the pole.If the pole is 60meters height,how long will it take the monkey to reach the top: 1. 31 minutes 2. 33 minutes 3. 35 minutes 4. 37 minutes.

33.Two trains start from stations A and B and travels towards each other at speeds of 50km/hour and 60km/hour respectively.At time of their meeting the second train has traveled 120km more than the first.The distance between A and B is: 1. 990km 2. 1200km 3. 1320km 4. 1440km

34.A and B can do a piece of work in 6 2/3 and 5 days respectively.they work together for 2 days and then A leaves.In how many days after that will B complete the work alone? 1. 1 ½ 2. 2 ¼ 3. 3 days 4. 7 days

35.Two pipes A and B can fill a tank in 4 hours and 5 hours respectively.If they are turned up alternatively for one hour each,then time taken to fill the tank is: 1. 2 hrs 15mins 2. 4 hrs 24mins 3. 5 hrs 4. 3 hrs

36.An electric device makes a 'beep' after 60 seconds and an another one after 62 seconds.They beeped together at 10.00a.m.Again they will beep at 1. 10.30 hrs 2. 10.31 hrs 3. 10.59 hrs 4. 11.00 hrs

37. A man has Rs.210 in coins alone.The coins consist of one rupee. 50p and 25p whose numbers from a ratio of 5 : 6 : 8 respectively.How many coins are there in the 50pdenomination? 1. 210 2. 126 3. 105 4. 16838.The L.C.M of three different numbers is 120. Which of the following cannot be their H.C.F? 1. 8 2. 12 3. 24 4. 30 and 24.

39.A sphere of radius 2 cm is put into water contained in a cylinder of radius 4 cm.If the sphere is completely immersed in the water, the water level in the cylinder rises by : 1. 2cm 2. 1/3 cm 3. 1/2 cm 4. 2/3 cm

40.The number of students in a school kept changing every year.In the year 1999, the total number of students in the school was 1000.In the year 2000,it is increased by 20%.In 2001,it again increased by 20%, but in 2002,it decreased by 5%.Find the number of students in the school at the end of year 2002 : 1. 1300 2.1368 3. 1400 4. 1638

Page 46: Quantitative

41.A stock of rice is enough for 100 men for 60 days.How long will the same stocks of rice last for 500 men: 1. 9 2. 5 3. 6 4. 10

42.If on burning 6 gas ovens for 6 hrs per day, an amount of Rs.45 is spent for 8 days, then by spending Rs.60 how many gas ovens can be burnt 5 hrs per day for 10 days? 1. 8 2. 16 3. 12 4. 6 5. 18

43.The cistern is filled in 9 hours and it takes 10 hours when there is a leak in its bottom.If the cistern is full,in what time shall the leak empty it? 1.90 hours 2.94 hours 3.92 hours 4.91 hours 5.none of these.

44.A sum of money yields at compound interest Rs.200 and Rs.220 at the end of first and second year respectively.the rate % is : 1. 20 2. 15 3. 10 4. 5 5. 11

45.In what time will a man receive Rs.85 as compound interest on Rs.320 at 12 1/2 % per annum , compounded annually? 1. 4 1/2 years 2. 2 1/2 years 3. 2 years 4. 5 years 5. 3 1/2 years

2.) Find the number of sides of a regular convex polygon whose angle is 40 degrees.

3.) a+b+c=0, then roots of ax^2+bx+c=0 is 1.imag 2.real 3.coincidental 4.zero

4.) Difference b/w the compound interest and simple interest for Rs.2500 for 2 years is given-----. find the rate of interest.

5.) there was one more question on S.I and C.I

10.) the largest no: which is a factor of 1080 and 729

11.) No: of spheres of radius 1 that can be got from sphere of radius (or diameter don't recall) 8

12.) Travelling at 3/4th the speed a man is 20 minutes later then speed is??

13.) there are 6 keys and 6 locks. then number of combinations to be tried out to get the actual solution a. 5^6 b.6^5 (don't remember the rest)

14.) choosing 2 people out of 10 in how many combinations can a particular person(some name) be always included..

18.) a man gets a gain of x%. but if he had sold at twice the cost price, what will be his gain?(question not sure) a. 2x b.200-2x c.100+x (not sure of the options)

Page 47: Quantitative

19.) a clock was 7mts behind the actual time on 3 p.m. on wednesday and 8 mts ahead of actual time on (not sure) 4 p.m. friday. when will it show the correct time?

20.) boat moves upstream in 6 hrs and covers the same distance downstream in 5 hrs. then speed of a raft floating?(accuracy of question not sure)

1. There are 4 buses - A,B,C,D. There are 220 students in a school. A can carry 60 students. B can carry 50 students. C can carry 40 students. D can carry 35 students Cost of travelling in the 4 buses were given, A - 160 , B- 140, C- 125 , D- 95 (not exact values) a) Find the bus combinations, so that all the students can be carried in the minimum cost (One can use any no. of buses of a particular type) b) Find the min. no. of buses required to carry all students etc.

12. A sum of money doubles in 5 years by simple interest. how much time it will take for 300 to become 2400.a>40 years b>25 years c>35 years d> 20 years.

13. If length of a rectangle is made 4 less and breadth is increased by 3 then the resultant square ?s area is equal to the rectangle. What is the perimeter of the rectangle. ( options were given)14. What will be the no. at ? position below 51 11 61 64 30 32 35 ? 43

16. Find the volume of the hemisphere having radius 2m .

18. A train goes from a to b. if it travels with 50 km/s then it is late by 10 mins. When it travels with30km/s then it bis late by 50 mins. Fi nd the distance between a and b.

19. A river is 8 m deep and 150 m wide.river?s speed is 5 km/s. find the volume of water passed in one min.(1 m cube =1000 c.c.)

20. Fin dthe equation of the st. line passing thru the intersection of the two lines and two other lines.(equ. Were given)

21. A man sells 20 mangoes + 15 oranges ath the same price as 15 mangoes+20 oranges. Ho would judge which is costlier . ( four options were given . I didn?t remember those)

24. In the word ?DISTURBANCE? 1st is replaced by last 2nd is replced by 10th , 3rd is by 9th and so on. Which will be the next to ?T?.

25. GIVEN if x/y==3/5. if 1 is added to num . and 1 is subtracted from denominator then ratio ==5/7. find the number

26. There are 5 black and 9 white balls .if one ball is drawn then find the probability of being white. 27. Avg. age of 5 children =.a child of 5 year age dies.after 4 years what will be the avg. age.

Page 48: Quantitative

29. A rectangle was having length 100 m breadth 60 m. there is a road of 5m wide on each side of therectangle . find the area of the road.

32. A person first goes upstream then down stream .river?s speed was given .also the diff. Of time was given . find the speed of the man.

33. Ravana speed is 5000 km/s when going towards the heaven. The distance == 75000. ravana has traveled 2 mins.When god?s messenger reaches the earth . Rama told him to go back. The speed of the god?s messenger =`00 km/s. By how much he will increase or decrease his speed so that he and Ravana reaches exactly at the same time.

34. A man sells a product by giving 10% reduction on it .in spite of this he gets 10% profit. on a product of 330 rs. Original price What will be the buying price.

1. one pipe fills in 4 hrs and another in 5hrs when they both work alternately how much time will be taken to fill the tank.

2. LCM of 3 nos is 120 which of the following no must not be their HCFa. 8 , b.24 c.12 d.24 and 30

3.two trains from the points A and B moving in opposite direction , at the point they meet the second train travels 120 kms more than the first. The speeds are 50kmph and 60kmph respectively find the distance between A and B?

5. a number when multiplied by 7/18 instead of 7/8 and got the result 770 less than the actual result , find the original number?

6. The volume and the radius of both cone and sphere are equal , then find the ratio of height of the cone to the diameter of the sphere?1. one pipe fills in 4 hrs and another in 5hrs when they both work alternately how much time will be taken to fill the tank.

2. LCM of 3 nos is 120 which of the following no must not be their HCFa. 8 , b.24 c.12 d.24 and 30

3.two trains from the points A and B moving in opposite direction , at the point they meet the second train travels 120 kms more than the first. The speeds are 50kmph and 60kmph respectively find the distance between A and B?

5. a number when multiplied by 7/18 instead of 7/8 and got the result 770 less than the actual result , find the original number?

6. The volume and the radius of both cone and sphere are equal , then find the ratio of height of the cone to the diameter of the sphere?

7. A and B started a business with 1500 and 2500 and got a profit 800 rs. Half of the profit is shared equally the remaining is shared according to their investment. Find their profits.

8. The difference between the simple interest and compound interest for 2 years?

Page 49: Quantitative

A:B =2:3 and B:C=5:6 then find A:B:C

9. an amount of 64 Rs has to become 125 in 3 years in compound interest, find the rate of interest?

12.Length of a rectangle is increased by 50% and breadth is decreased by 25% what is the difference in the area

14.Two successive discounts of 20% and 15% is equal to a net discount of .....

16.The length of the following rectangle is '4a' and its breadth is '2a'. Radius of the two circles is 'a'. Then find the ratio of total area of the rectangle to the area not covered by the two circles with in the rectangle

17.A person starts with the speed of u/1 kmph and returns with the speed of u/2 kmph, what is his average speed.

18.A cistern will be filled in 9 hrs, but becoz of an outlet it is filled in 10 hrs, if the cistern is filled, then how much time the outlet takes to empty the cistern.

19.In a right angled triangle ABC angle B = 90 , BM is the median to AC , then AB2 + BC2 (in terms of BM)

20.Three circles with same radius r are drawn with centres as three vertices of a triangle. What is the sum of areas of the intersections of these circles with the triangle.

21.X men work for X days to produce X products, then Y men can produce Y products in days.

22.The compound interest for first and second years is 200 and 220 on a certain amount. Find the sum.

23.Marked price of a commodity is 35% above the cost price. If he gives a discount of 15%, how much he gains on the deal.

24.5 mangoes + 4 oranges = 7 mangoes + 1 orange.Find the ratio of mango to orange.

26.Food is sufficient for 100men for 60days. For how many days the food is sufficient for 500men?

27.If 8men 8hrs per day works for 8days get 45/- then how many men required if the work is 5hrs per day for10days they get 60/-?

28.A person sold an item at a profit of 12% .If he sold it at a loss of 12% then he would get Rs.6/- less. What is the cost price?

30.Avg age of X number of adults in a class is 30yrs. If 12 new adults with avg age of 32 joined with them then the avg age increases by one. Find X?

Page 50: Quantitative

31.A sphere of radius 2cms is dropped into a cylinder of radius 4 cms containing water upto certain level. The raise in the water level is (not sure)

32.find the average of reciprocals of x and y :

33.In a school there are 1000 students in the year 1999. The number of students increased by 20% in the year 2000. And it is increased by 15% in the year 2001. But it is decreased by 18% in 2002. Then what is the strength in 2002.(numbers may differ)1. 600 to be seated.10benches are less.so 2 more persons are required to be seated in each bench.so how many benches.209 36 44 602. Selling a ac gains 25% on SP.what %gain on CP.15 25 33.33 47.5

2.There is a 200 miles long tunnel. one train enters the tunnel at a speed of 200mph while the other trains enter the tunnel in the opposite direction at a speed of 1000 mph. A bee travels at a speed of 1500 mph enters the tunnel goes to and back until it reaches the train. What is the distance covered by the bee when the two train collides (the bee survives)

21.If the probability of work done by three persons are 1/3, 2/5, 5/12. Then what is the probability that the work is completed.

25.Krishna tosses a one-rupee coin and a rupee coin. He announces that one is head. But the result is not announced. What is the probability that the other coin is head?

7) Fresh Grapes contain 90% water by wt. Dried grapes contain 20% water by %age. What will b wt of dried grapes when we begin with 20 kg fresh grapes?

8. A man engaged a servant on a condition that he’ll pay Rs 90 and also give him a bag at the end of the yr. He served for 9 months and was given a turban and Rs 65. So the price of turban is Rs 10 / 19 / 0 / 55

9). The sum of six consecutive odd nos. is 888. What is the average of the nos.?i. 147 ii. 148 iii. 149 iv. 146

10. In a race from pt. X to pt Y and back, Jack averages 30miles/hr to pt Y and 10 miles/hr back to pr X.Sandy averages 20 miles/hr in both directions. If Jack and Sandy start race at same tym, who’ll finish 1st Jack/Sandy/they tie/Impossible to tell

12. Three wheels make 36, 24, 60 rev/min. Each has a black mark on it. It is aligned at the start of the qn. When does it align again for the first tym? 14/20/22/5 sec

13. If 1= (3/4)(1+ (y/x) ) theni. x=3y ii. x=y/3 iii. x=(2/3)y iv. None

1) If log 0.317=0.3332 and log 0.318=0.3364 then find log 0.319 ?Sol) log 0.317=0.3332 and log 0.318=0.3364, then

Page 51: Quantitative

log 0.319=log0.318+(log(0.318-0.317)) = 0.3396

8) Y catches 5 times more fishes than X. If total number of fishes caught by X and Y is 42, then number of fishes caught by X?Sol) Let no. of fish x catches=pno. caught by y =rr=5p.r+p=42then p=7,r=359) Three companies are working independently and receiving the savings 20%, 30%, 40%. If the companies work combinely, what will be their net savings?suppose total income is 100so amount x is getting is 80y is 70z =60total=210but total money is 300300-210=90so they are getting 90 rs less90 is 30% of 300 so they r getting 30% discount36) A person who decided to go weekend trip should not exceed 8 hours driving in a day average speed of forward journey is 40 mph due to traffic in Sundays the return journey average speed is 30 mph. How far he can select a picnic spot.

45) If Rs20/- is available to pay for typing a research report & typist A produces 42 pages and typist B produces 28 pages. How much should typist A receive?Here is the answer Find of 42 % of 20 rs with respect to 70 (i.e 28 + 42) ==> (42 * 20 )/70 ==> 12 Rs

46) An officer kept files on his table at various times in the order 1,2,3,4,5,6. Typist can take file from top whenever she has time and type it.What order she cannt type.?

49) One tank will fill in 6 minutes at the rate of 3cu ft /min, length of tank is 4 ft and the width is 1/2 of length, what is the depth of the tank? 3 ft 7.5 inches

51) Ram singh goes to his office in the city every day from his suburban house. His driver Mangaram drops him at the railway station in the morning and picks him up in the evening. Every evening Ram singh reaches the station at 5 o'clock. Mangaram also reaches at the same time. One day Ram singh started early from his office and came to the station at 4 o'clock. Not wanting to wait for the car he starts walking home. Mangaram starts at normal time, picks him up on the way and takes him back house, half an hour early. How much time did Ram singh walked?

59) There are three different boxes A, B and C. Difference between weights of A and B is 3 kgs. And between B and C is 5 kgs. Then what is the maximum sum of the differences of all possible combinations when two boxes are taken each timeA-B = 3B-c = 5a-c = 8

Page 52: Quantitative

so sum of diff = 8+3+5 = 16 kgs

65) A man, a women and a child can do a piece of work in 6 days,man can do it in 14 days, women can do it 16 days, and in how many days child can do the same work?The child does it in 24 days

66) A: 1 1 0 1 1 0 1 1B: 0 1 1 1 1 0 1 0C: 0 1 1 0 1 1 0 1Find ( (A-B) u C )==?Hint : 109A-B is {A} - {A n B}A: 1 1 0 1 1 0 1 1B: 0 1 1 1 1 0 1 0by binary sub. a-b = 01100001 (1-0=1, 1-1=0,0-0=0, n for the 1st 3 digits 110-011=011)now (a-b)uc= 01100001or      01101101gives 1101101... convert to decimal equals 102.If G(0)= -1, G(1)=1, G(N)=G(N-1)-G(N-2), then find- a) G(2)=? b) G(3)=? c) G(4)=? d) G(5)=? e) G(6)=? 3.Which is not a TRIANGLE— # a) (2m,3m,4m) b) (3m,4m,7m) c) (3m,5m,9m) # a) (30’,60’,80’) b) (40’,60’,50’) c) (60’,30’,90’) 4.Which is the exact power of 2 — Choice : a)2048 b)2068 c)2668 d)2408 11. The size of a program is N. And the memory occupied by the program is given by M = square root of 100N. If the size of the program is increased by 1% then how much memory now occupied? 12.A man, a woman, and a child can do a piece of work in 6 days. Man only can do it in 16 days. Woman can do it in 24 days and in how many days child can do the same work?

18. If A, B and C are the mechanisms used separately to reduce the wastage of fuel by 30%, 20% and 10%. What will be the fuel economy if they were used combined. 19. Which of the following are orthogonal pairs? a. 3i+2j b. i+j c. 2i-3j d. -7i+j 20.A is twice efficient than B. A and B can both work together to complete a work in 7 days. Then find in how many days A alone can complete the work? 21. Find the singularity matrix from a given set of matrices?(Hint det(A)=0) 22. A=11010101 b=10101010 c=10001001 find the decimal value of (AU(b-c)) 23. there was one pi chart and 4 qs are baised on it...easy one29. Three independent mechanisms A, B and C have been incorporated for power saving in a plant producing respectively 30%, 40% and 10% efficiency. Assuming that they operate independently, what is the net power efficiency achieved. (a) 62.2% (b) 68% (c) 61% (d) 64% (e) 56%30.COUNTERFEIT in 80 coins one coin is counterfiet what is minimum number of weighings to find out counterfiet coin

Page 53: Quantitative

31.Match the following: 1. Male - Boy ---> a. A type of 2. Square - Polygon ---> b. A part of 3. Roof - Building ---> c. Not a type of 4. Mushroom - Vegetables ---> d. A superset of 32. One year ago Pandit was three times his sister's age. He will be only twice her age next year. How old will Pandit be after five years? a) 8 b) 12 c) 11 d) 13 1. Two liquids A : B in ratio 7:5 and now 24 gallons drain out and b is added the ratio becomes 5:7 what is containr capacity ? 30, 48 , 84, none2. man has rope of length 660 mtr to fence a area , what is the max area he can fence ?3. Son is about to celebrate 10th birthday. after 11 years his age will be half the average of his parents. his mother is 17 years older to him. what is the age of the father.4. DI question where hotels project are given and no of rooms in each hotels , investment , project yr . and company namewere asked to find min cost per room is for which hotel , which avg cost per room etc.5. Avg of 6 no is 8 what 7 th no shud be added to make avg 10;6. difference  of cube of a no and its square is a perfect square , what is the no. -> 57. diff of three consecutive nos square and the nos is 214; find the nos :21) Ashok bought 16kg of wheat at the rate of Rs 11.50 per kg and 14 kg wheat at rate of Rs 14.50 per kg. He mixed the two and sold the mixture at the rate of 13.50 kg. What is his gain in the transaction ? a) Rs 16 b) Rs 18 c) Rs 24 d) Rs 28 22) What is the probability of answering a multiple choice question correctly, If you know that one of the answer if definitely wrong but you have to guess between the 3 remaining answers? a) ¼ b) ¾ c) ½ d) 1/3 24) The difference between two numbers is one-seventh of the sum of these numbers two numbers. What is the ratio of the two numbers? a) 3:4 b) 4:3 c) 2:3 d) 3:2 25) On return from a business trip Mr. Chidambaram was to be picked up from the railway station by his coachman. Someone he managed a train connection earlier and thus arrived two hours too early. Immediately on arrived he rang up home for the coach and was told that it had just left in order to be exactly in time for the train by which he was scheduled to come. To save the time he started walking homeward at 3kmph. On the way he met the coachman who brought him home an hour before schedule. How far is the Mr. Chidambaram’s house from the railway station? a) 12 Km b) 15 Km c) 18 Km d) 23 Km 26) A train travelled from Delhi to Patna and back in a certain time at the rate of 60kmph.But if the train had travelled from Delhi to Patna at rate of rate 80Kmph. And back from Patna to Delhi at the rate of 40Kmph. It would take two hours Longer. Find the distance between Delhi and Patna? a) 480Km b) 460Km c) 450Km d) 465Km 27) On a certain pasture the grass grows at an even rate. It knows that 40 cows can graze on it for 40 days before the grass is exhausted, but 30 cows can graze there as long as 60 days. How many would the pasture last if 20 Cows were to graze on it? a) 140days b) 100 days c)110 days d) 120 days 28) Most people in the United States view both neither big nor small business as particulary efficient or dynamic and regard both as providing consumers with fairly priced goods and services. However, most people consistently perceive small business as a

Page 54: Quantitative

force for good in Society, whereas bid business is perceived as socially responsible only in times of prosperity. The statement above, if true, would provide the strongest support for which one of the following hypotheses? a) Most of the people in the United states give little thought to the value of business to society. b) If small business were regarded as being more dynamic, it, too, would receive strongly favorable ratings only in times of general prosperity. c) Many people in the united states regard the social responsibility of big business as extending beyond providing consumers with fairly priced goods and services. d) Even if people did not regard big business as providing consumers with value for their money, they would still regard it as socially responsible in times of general prosperity. 29) In the first innings of the First Test match between India and West Indians at Sabina Park, Kingston. February 18 to 23 1971, the bowling honors’ in the Indians team were shared by B.S.Bedi, E.A.S. Prasanna and S.Venkataraghavan. Can you, based on the following statements, conclude the number of wickets each took? 1) Either Bedi took 5 wickets and S.Venkataraghavan took 3, or Prasanna took one more than S.Venkataraghavan. 2) Either Bedi took half the wickets or one of the players was run out. 3) The sum of Bedi’s victims and double those of prasanna is one more than three times Venkataraghavan’s. a) Bedi-2, Prasanna-4, and Venkataraghavan-3 b) Bedi-3, Prasanna-4, and Venkataraghavan-2 c) Bedi-2, Prasanna-4, and Venkataraghavan-4 d) Bedi-2, Prasanna-5, and Venkataraghavan-3

30) The Basic College Council is made up of representatives from the departments in the various divisions. From the science Division is one representative from each of the departments of chemistry, Mathematics, and Biology. The social Science Division provides a representative each History, Linguistics, and Economics. A representative from Philosophy and one from Rhetoric serve for the Humanities Division All Council Committees are made up entirely of Council members. Each Committee has exactly four members. Each Committee has at least one member from each Division. The representative from chemistry will not serve on a Committee with the representative from Biology. The representatives from Mathematics and Economics always serve on the same Committees. The representative from Rhetoric will only serve on a Committee on which the representative from Biology or the one from Economics, or both, serve. a) Chemistry, Mathematics, History, Economics b) Chemistry, Mathematics, History, Philosophy c) Biology, History, Economics, Rhetoric d) Mathematics, Biology, Economics, Rhetoric 31) A train leaves New York City at 7.15 Am and arrives in Buffalo at 2.47 that afternoon. What total length of time does the trip take? a) 6hrs, 28mins b) 7hrs, 12mins c) 7hrs, 32mins d) 8hrs, 12mins 32) After several attempts to district his young parrot from Chewing on furniture, George reluctantly took an expert’s advice and gently hit the parrot’s beak whenever the bird started to chew furniture, The bird stopped chewing furniture, but it is now afraid of hands and will sometimes bite. Since Chewing on the furniture would not have hurt the bird, George should not have hit it.

Page 55: Quantitative

When Carla’s puppy escaped from her yard, it bounded into a busy street. Although Carla does not generally approve of physical discipline, she hit the puppy sharply with her hand. Now the puppy enters the street only when accompanied by Carla, so, Carla was justified in disciplining the puppy. Which one of the following principles, if established, would justify the judgments about George’s and Carla’s actions? a) When disciplining an animal physically, a trainer should use an object such as a rolled up newspapers to avoid making the animal frightened of hands. b) When training an animal, physical discipline should be used only when such discipline is necessary to correct behavior that could result in serious harm to the animal. c) Using physical discipline to train an animal is justified only when all alternative strategies for the correcting undesirable behavior have failed. d) Physical discipline should not be used on immature animals. 33) In a Conventional Clock, how many times does the minutes hand pass the hour hand between noon and midnight? a) 8 times b)10 times c)12 times d)15 times

35) Its not easy having a mathematics professor as a friend. When she invited you to her house she says, “All the houses on my side of the street are numbered consecutively in even numbers. There are Six houses on my Side of my block and sum of their numbers is 9870. You don’t know which block I live on, and it’s a long street, but I will tell you that I live in the lowest number on my side of the block. What’s the number? Or are you just going to ring the first- numbered doorbell for twenty blocks? a) 1580 b)1640 c)1650 d)1680

36) The time to complete a standardized exam is approximately normal with a mean of 70 minutes and a standard deviation of 10 minutes. Suppose the students are given 1 hour to complete the exam. The proportion of students who don’t complete the exam is a) 0.500 b)0.159 c)0.341 d)0.841 37) Rajesh brought a TV priced at Rs 2000. He was given two Successive discounts of 10 and 5 percent. If he has to pay percent sales tax, the net amount he paid wasa) Rs 2663.40 b) Rs 2660 c) Rs 2350 d) Rs 2460 38) A drawer contains a number of Red and Blue socks. If I pull two out of random, then the chance of them being a Red pair is a half and chance of them being a blue pair is a twelfth. How many socks are in the drawer? a)120 b)140 c)150 d)160 39) A Clock strikes every hour-once at 1.00, twice at 2.00, and so on. The clock takes 6 seconds to strike 5.00 and 12 seconds to strike 9.00. the time needed to strike 1.00 is negligible. How long does the clock need for all its striking in 24hours? a)168 Seconds b)178 Seconds c)188 Seconds d) 198 Seconds 40) Grandpa was feeling generous, so he gave a total of Rs 100 to his Five grandchildren. Starting with the Youngest each got Rs 2.00 more than the next Younger one. In Other words, the youngest got one sum, the next got Rs 2.00 more, and so on. How much did the Youngest Grandchild get? a)14 b)15 c)16 d)1721.in a row of boys anand is eleventh from the left and deepak is fifteenth from the right.when anand and deepak interchange their positions, anand will be fifth from the left,which of the follwing will be deepak's position from the right? a)seventh b)seventeenth  c)eighth   d)ninth

Page 56: Quantitative

22.a watch ,which gains uniformly is 2min.show at noon on monday,and is 4min.48 seconds fast at 2pm.on the follwing monday.when was it correct? a)2 days 6hrs  b)2 days 2 hrs   c)3days 1hr d)3days 6hrs 23.on the occasion of a certain meeting each member gave shake hand to the remaining number.if the total shake were 1176,how many members were  present for the meeting. a)48 b)58 c)49  d)59 24.a multiple phtostat copier can create 120 copies in 30 seconds.how much time would it take to from 2000 copies on two such machines working simultaneously. a)4min 08sec b)4min 10sec  c)4min 25sec d)4min 30sec 5. Wind flows at 160 miles in 330 minutes, for traveling 80 miles how much time does it require?(a) 1 hour 30 minutes (b) 1 hour 45 minutes (c) 2 hours(d) 2 hours 45 minutes (e) 3 hours

6. A stationary engine has enough fuel to run 12 hours when its tank is 4/5 full. How long will it run when the tank is 1/3 full?(a) Less than 2 hours (b) 2 hours (c) 3 hours (d) 4 hours (e) 5 hours

7. If A is traveling at 72 km per hour on a highway. B is traveling at a speed of 25 meters per second on a highway. What is the difference in their speeds in meters per second?(a) 1/2 m/sec (b) 1m/sec (c) 1 1/2 m/sec (d) 2 m/sec (e) 3 m/sec

8. A salesperson by mistake multiplied a number and got the answer as 3, instead of dividing the number by 3. What is the answer he should have actually got?(a) 0 (b) 1/3 (c) 1 (d) 2 (e) 3

9. If the length of a rectangle is increased by 30% and the width is decreased by 20%, then the area is increased by...(a) 10% (b) 5% (c) 4% (d) 20% (e) 25%

11. The most economical prices among the following prices is:(a) 10 kilo for Rs.160 (b) 2 kilo for Rs.30 (c) 4 kilo for Rs.70(d) 20 kilo for Rs.340 (e) 8 kilo for Rs.130

12. A truck contains 150 small packages, some weighing 1 kg each and some weighing 2 kg each. how many packages weighing 2 kg each are in the truck if the total weight of all the packages is 264 kg?(a) 36 (b) 52 (c) 88 (d) 124 (e) 114

13. A man was arrested for exceeding the speed limit by 10 miles an hour. A second man was charged with exceeding the same limit by twice as much. The second man was driving 35 miles per hour. What was the speed limit?(a) 10 miles per hour (b) 15 miles per hour (c) 20 miles per hour(d) 25 miles per hour (e) 30 miles per hour

14. One year ago Pandit was three times his sister's age. Next year he will be only twice her age. How old will Pandit be after five years?(a) 8 (b) 12 (c) 11 (d) 13 (e) 15

15. If two pencils cost 8 cents, then how much do 5 pencils cost?

Page 57: Quantitative

(a) 18 cents (b) 20 cents (c) 22 cents (d) 23 cents (e) 24 cents

5. which will give good standard deviation1. (7,0,-7,0,7) 2. (7,-7,7,-7,7) 3. (1,0,-1,0,1)6. which is not a side of a rectangular1. (2,3,4) 2.(3,4,7) 3. (3,5,9)9. a man , woman and a boy jointly did a job in 6 days. a man alone finishes in 10 days, a women alone finish in 24 days. then how many days the boy can take to finish?

10. for temperature a function is given according to time : ((t**2)/6) + 4t +12.what is the temperature rise or fall between 4.am to 9 am

11. an aeroplane starts from a (some latitude is given according to place)at 2 am local time to b(some latitude). travailing time is 10 hours. what is the local time of b when it reaches b

22. three companies working independently and get savings 10%, 20%, 25%. if the companies work just opposite then what will be the net saving? 28. which is the perfect one among the 41. 2x +3y=4 2. x + y= -1 3. y=2x+ 3

45. The number 362 in decimal system is given by (1362) x in the X System of numbers find the value of X a} 5 b) 6 c) 7 d) 8 e) 9

4. The lowest temperature in the night in a city A is 1/3 more than 1/2 the highest during the day. Sum of the lowest temperature and the highest temperature is 100 degrees. Then what is the low temp? Ans.40 degrees6. A salesperson by mistake multiplied a number and got the answer as 3, instead of dividing the number by 3.What is the answer he should have actually got? Ans. 3

20. Number of faces, vertices and edges of a cubea. 12,8,6 b. 4,6,8 c. 6,8,12 d. 6,12,8

10. In the class of 40 students, 30 speak Hindi and 20 speak English. What is the lowest possible number of students who speak both the languages?(a) 5   (b) 20     (c) 15  (d) 10 (e) 30

12. A truck contains 150 small packages, some weighing 1 kg each and some weighing 2 kg each. how many packages weighing 2 kg each are in the truck if the total weight of all the packages is 264 kg?(a) 36 (b) 52  (c) 88 (d) 124 (e) 114

14. One year ago Pandit was three times his sister's age. Next year he will be only twice her age. How old will Pandit be after five years?(a) 8   (b) 12  (c) 11  (d) 13  (e) 15

Page 58: Quantitative

9. A MAN , WOMAN AND A BOY JOINDLY DID A JOB IN 6 DAYS. A MAN ALONE INISHES IN 10 DAYS, A WOMEN ALONE FINISH IN 24 DAYS. THEN HOW MANY DAYS THE BOY CAN TAKE TO FINSH?

10. FOR TEMPERATURE A FUNCTION IS GIVEN ACCORDING TO TIME : ((t**2)/6) + 4t +12 WHAT IS THE TEMPERATURE RISE OR FALL BETWEEN 4.AM TO 9 AM28. WHICH IS THE PERFECT ONE AMONG THE 4

1. 2x +3y=4 2. x + y= -1 3. Y=2x+ 3

1. Average of 5 innings is 20. If four innings are 32, 10, 40 and 12 find the 5th inning score.2. Area of path around a square is 256 sq.m. Find the width of path if the side of square is 30m.3. A company increases the bonus 3 times of its 50 employees. What is the % increase in bonus?4. A man swims in a river 5 min upstream and 5 min downstream. Starting at A and ending at B. What is the speed of the man?5. NESTIN is a jumbled word. What are the first and last letters of the correct word.6. There is enough material to fence 30m linear. Also there is old fence on oneside of the plot. Find the length and breadth of the plot.7. In a game of Snooker A gives B 18 points for 90, A gives C 10 points for 60. How many can C give B in a game of 70?

8. Gold is 19 times heavier than water. Copper is 9 times heavier than water. In what ratio should they be mixed so that the alloy is 15 times heavier than water?9. A sphere of radius r is placed inside a cylinder of height 2r which just fits. What is the Volume of the empty space?10. A is 3 times as old as B. B is 4 years older than C. If C is Z years old express A in terms of Z.

15. Srikanth has borrowed Rs 3000 at 10% C.I.. He has to repay that after 3 years in 3 equal installments. How much does he have to pay as an installment?16. Given a date and day, the question was to calculate the day of the date June 12, 1979.17. The weight of a Bucket full of water is 17 kg, the weight of the same bucket with half full water is 13.5 kg. What is the weight of the Bucket?18. In a party attended by boys and girls where the girls outnumbered the boys Rs 280 was distributed. Each boy was given Rs 10 and each girl Rs 20. How many boys are there?19. 10% of an army died. 10% of the were ill. 12% of the rest met with an accident. What is the total number of the original army if 7 lakhs were finally left?                                                  20. A started a business with Rs 1700 amount. B joined A after 3 months , C joined after 6 months. If the profit is Rs 1700 and they shared it in the Ratio of 2:3:5 how much did B and C invest?21. A beats B by 24m. A beats C by 20m. C beats B by 1 sec. In how many seconds does A complete the race of distance 120m.22. Which is larger Rs 35 : Rs 1.40 or Rs 48 : Rs 1.44

Page 59: Quantitative

23. A started a business with Rs 1700 amount. B joined A after 3 months , C joined after 6 months. If the profit is Rs 1700 and they shared it in the Ratio of 2:3:5 how much did B and C invest?24. In a wall clock the smaller needle is between 8 and 9 and the larger makes a right angle with it. Find the exact time.25. If the temperature increases uniformly from 9 am to 2 pm and it increases from 21 C to 36 C. What is the temperature at noon?26. A monkey climbs 3m of a greased pole in 1 minute and then slips 1m in the next. When will he reach the top of the pole of 12 m?27. Length and Breadth of a Rectangle are 100 m and 6 m. Find the area of a road around the rectangle with a width of 5m.28. Find the figure with the highest Areai)Circle with r=2.ii)Equilateral Triangle with Side = 4.iii)Square of diagonal =2.iv)Triangle with a=8, b=5 and c=4.29. A boat moves 10 km upstream and returns to the same point in 45minutes. The speed of the stream is 3km/hr. The speed of the boat is ?30. Given an Equilateral triangle of side 10m. A cow is tied to one end. The length of the rope is 7 m. The area covered by the cow is ?31. A train goes from A to B at 50 km/hr and comes back at 40 km/hr and hence takes an hour more time to return. What is the distance between A and B?Q45) D/23, 17/F, I/12, 8/M, R/51.1/4th   of  a no is thrice the 3/4th of the no. then find the no.2.a right angle triangle has base of  30cms and the opp angle is 50 degree then find length of hypotenuse.3.two unbiased coins are tossed then prob of  obtaining both heads is ¼.4.if a shop owner sells 3 diff ground nuts with prices Rs.50,Rs.30,Rs.20 and thy are in proportion 2:3:5 so that they are mixed and sold at Rs.33 then find gain.

7.an aeroplane delayed by half an hour if  it travels at a speed of  250 kmphr more speed than previous to reach destination at the same time then find the original speed. Some q,s included on profit and loss ,trains ,venn diagrams,men and work

(8) A horse covers half of t total distance at 12mph without load and the remaining distance at 4mph with load.  What is the average speed of the horse?Formula use: 2xy/(x+y)X=12mph y=4mphSo average speed = 6mph10. albert and fernandes they have two leg swimming race.   both start from opposite and of the pool.  On the first leg, the  boys pass each other at 18 mt from the deep end of the  pool.   during the II leg they pass at 10 mt from the shallow end of  the  pool.  Both go at const speed.  but one of them  is  faster.   each  boy rests for 4 sec to see at the end of the  i  leg.   what  is  the length of the pool.

4.aa person sold two articles for 80 /- each.with 20% profit on one  article and 20% loss on another article, what is the loss / prifit he will gain on both.

Page 60: Quantitative

3.in a group of 5 persons a,b,c,d,e one of the person is advogate, one is doctor, one businesss man, one shop keeper and one is professor. three of them a,c,and professor prefer playing cricket to foot ball and two of them b and businessman prefer playing foot ball to cricket. the shop keeper and b and a are friends but two of these prefer playing foot ball to cricket. the advogate is c's brother and  both play same game . the doctor and e play cricket.(a) who is advogate ?a, b, c, d(b) who is shop keeper ?a, b, c, d(c) which of the following group include persons who like playing cricketbut doesn't include professor ?ab,bc,cd, none(d) who is doctor ?a,b,c,d.{ same model problem was asked in ques paper but professions will be different such as horticulturist ,physicst,journalist,advocate and other one. instead of football and cricket they will give tea and coffee }

24) A can do a piece of work in 20 days, which B can do in 12 days. In 9 days B does ¾ of the work. How many days will A take to finish the remaining work?  25) Anand finishes a work in 7 days; Bittu finishes the same job in 8 days and Chandu in 6 days. They take turns to finish the work. Anand on the first day, Bittu on the second and Chandu on the third day and then Anand again and so on. On which day will the work get over?A) 3rd b) 6th c) 9th d) 7th  26) 3 men finish painting a wall in 8 days. Four boys do the same job in 7 days. In how many days will 2 men and 2 boys working together paint two such walls of the same size?A) 6 6/13 days B) 3 3/13 days C) 9 2/5 days D) 12 12/13 days  28)  A = 10010001       B = 01101010       C = 10010110(AuB)nC =? [(A union B) intersection C =?]

4)Joe's father will be twice his age 6 years from now. His mother was twice his age 2 years before. If Joe will be 24 two years from now, what is the difference between his father's & mother's age?a) 4 b) 6 c) 8 d) 10ALLIGATION1).In a mixture, R is 2 parts, S is 1 part. In aoder to make S to 25% of the mixture, howmuch R is to be added?AVERAGE1) gavaskar average in first 50 innings was 50 . after the 51st innings his average was 51 how many runs he made in the 51st innings4)THREE COMPANIES WORKING INDEPENDENTLY & GET SAVINGS 10%, 20%, 25%. IF THE COMPANIES WORK JUST OPPOSITE THEN WHAT WILL BE THE NET SAVING?

Page 61: Quantitative

BOATS1) If a man can swim downstream at 6kmph & upstream at 2kmph, his speed in still water is:a. 4kmph b. 2kmph c. 3kmph d. 2.5kmphCAR questions1)A car is filled with four & half gallons of oil for full round trip. Fuel is taken 1/4 gallon more in going than coming. What is the fuel consumed in coming up?Sol Before the trip, car is filled with = 4 ½ gallon of oilLet ‘X’ be the quantity of fuel consumed for the trip in one directionThe fuel consumed while going = X + ¼The fuel consumed while coming = XTherefore, the fuel consumed for the trip = (X + ¼) + X = 4 ½ð 2X + ¼ = 4 ½ => 2X = 4 ½ - ¼ => 2X = 4 ¼ => X= 2 gallons approxCHAIN RULE questions1)A can copy 50 papers in 10 hours while both A & B can copy 70 papers in 10 hours. Then for how many hours required for B to copy 26 papers?

10. In a company 30% are supervisors and 40% employees are male. If 60% of supervisors are male. What is the probability that a randomly chosen employee is a male or female?49. A 3-gallon mixture contains one part S and two parts R. In order to change it to a mixture containing 25% S, how much R should be added?(a) 1/2 gallon (b) 2/3 gallon  (c) 3/4 gallon (d) 1 gallon(e) 1 1/2 gallon

53. If A is traveling at 72 km per hour on a highway. B is traveling at a speed of 25 meters per second on a highway. What is the difference in their speeds in meters per second?(a) 1/2 m/sec (b) 1 m/sec          (c) 1 1/2 m/sec       (d) 2 m/sec   (e) 3 m/sec54. A salesperson by mistake multiplied a number and got the answer as 3, instead of dividing the number by 3. What is the answer he should have actually got?(a) 0        (b) 1/3        (c) 1        (d) 2        (e) 355. If the length of a rectangle is increased by 30% and the width is decreased by 20%, then the area is increased by... (a) 10%        (b) 5%        (c) 4%        (d) 20%        (e) 25%

57. The most economical prices among the following prices is:(a) 10 kilo for Rs.160(b) 2 kilo for Rs.30(c) 4 kilo for Rs.70(d) 20 kilo for Rs.340(e) 8 kilo for Rs.13058. A truck contains 150 small packages, some weighing 1 kg each and some weighing 2 kg each. how many packages weighing 2 kg each are in the truck if the total weight of all the packages is 264 kg?(a) 36        (b) 52        (c) 88        (d) 124        (e) 11411. A man, a woman, and a child can do a piece of work in 6 days. Man only can do it in 24 days. Woman can do it in 16 days and in how many days child can do the same work?12. In which of the system, decimal number 384 is equal to 1234?

Page 62: Quantitative

13. Find the value of the 678 to the base 7.

25. A man, a woman, and a child can do a piece of work in 6 days. Man only can do it in 24 days. Woman can do it in 16 days and in how many days child can do the same work?

29. If A, B and C are the mechanisms used separately to reduce the wastage of fuel by 30%, 20% and 10%. What will be the fuel economy if they were used combined.30. What is the power of 2? a. 2068 b.2048 c.2668

35. A can copy 50 papers in 10 hours while both A & B can copy 70 papers in 10 hours. Then for how many hours required for B to copy 26 papers? 36. A is twice efficient than B. A and B can both work together to complete a work in 7 days. Then find in how many days A alone can complete the work?37. A finish the work in 10 days. B is 60% efficient than A. So how days does B take to finish the work?38. A finishes the work in 10 days & B in 8 days individually. If A works for only 6 days then how many days should B work to complete A’s work?40. Find the singularity matrix from a given set of matrices?(Hint det(A)==0)43. (a) 2+3i (b)1+i (c) 3-2i (d) 1-7i .Find which of the above is orthogonal.44. (Momentum*Velocity)/(Acceleration * distance) find units.45. The number 362 in decimal system is given by (1362)x in the X system of numbers find the value of X a) 5 b) 6 c) 7 d) 8 e) 9

2.what is the largest prime number in 8 digit number?4.the plane question was asked.it took off from 7 degree 53' 6.1'' and landed at 8 degree 6' 43.5''.it takes 12 hours to travel between these points.then what will be local time at destination ?[just follow previous ques paper the problem was same]5.g[0]=1,g[1]=-1,g[n]=2*g[n-1]-3*g[n-2] then calculate g[4]= ?

1. 9. Two stations A & B are 110 km apart. One train starts from A at 7 am, and travels towards B at 20kmph. Another train starts from B at 8 am and travels towards A at 25kmph. At what time will they meet?a. 9 am b. 10 am c. 11 am d. 10.30 am

10. If a man can swim downstream at 6kmph and upstream at 2kmph, his speed in still water is:a. 4kmph b. 2kmph c. 3kmph d. 2.5kmph

There are seventy clerks working in a company, of which 30 are females. Also, 30 clerks are   married; 24 clerks are above 25 years of age; 19 married clerks are above 25 years, of which 7 are males; 12 males are above 25 years of age; and 15 males are married. How many bachelor girls are there and how many of these are above 25?  4. In a group of five people, what is the probability of finding two persons with the same month of birth?

Page 63: Quantitative

 5. A father and his son go out for a 'walk-and-run' every morning around a track formed by an equilateral triangle. The father's walking speed is 2 mph and his running speed is 5 mph. The son's walking and running speeds are twice that of his father. Both start together from one apex of the triangle, the son going clockwise and the father anti-clockwise. Initially the father runs and the son walks for a certain period of time.    Thereafter, as soon as the father starts walking, the son starts running. Both complete the course in 45 minutes. For how long does the father run? Where do the two cross each other?  6. The Director of Medical Services was on his annual visit to the ENT Hospital. While going through the out patients' records he came across the following data for a particular day :  " Ear consultations 45; Nose 50; Throat 70; Ear and Nose 30; Nose and Throat 20; Ear and Throat 30; Ear, Nose and Throat 10; Total patients 100." Then he came to the conclusion that the records were bogus. Was he right?  7. Amongst Ram, Sham and Gobind are a doctor, a lawyer and a police officer. They are married to Radha, Gita and Sita (not in order). Each of the wives have a profession. Gobind's wife is an artist. Ram is not married to Gita. The lawyer's wife is a teacher. Radha is married to the police officer. Sita is an expert cook. Who's who?  14. A farmer has two rectangular fields. The larger field has twice the length and 4 times the width of the smaller field. If the smaller field has area K, then the are of the larger field is greater than the area of the smaller field by what amount? (a) 6K   (b) 8K   (c) 12K  (d) 7K 15. Nine equal circles are enclosed in a square whose area is 36sq units. Find the area of each circle.  16. There are 9 cards. Arrange them in a 3*3 matrix. Cards are of 4 colors. They are red, yellow, blue, green. Conditions for arrangement: one red card must be in first row or second row. 2 green cards should be in 3rd column. Yellow cards must be in the 3 corners only. Two blue cards must be in the 2nd row. At least one green card in each row.  18. A speaks truth 70% of the time; B speaks truth 80% of the time. What is the probability that both are contradicting each other?  19. In a family 7 children don't eat spinach, 6 don't eat carrot, 5 don't eat beans, 4 don't eat spinach & carrots, 3 don't eat carrot & beans, 2 don't eat beans & spinach. One doesn't eat all 3. Find the no. of children.  20. Anna, Bena, Catherina and Diana are at their monthly business meeting. Their occupations are author, biologist, chemist and doctor, but not necessarily in that order. Diana just told the neighbour, who is a biologist that Catherina was on her way with doughnuts. Anna is sitting across from the doctor and next to the chemist. The doctor was thinking that Bena was a good name for parent's to choose, but didn't say anything.  What is each person's occupation?  nw,We hd PPt abt wipro....concentrate wen he/she tells abt six-sigma...????This round is the elimination round....so u gotta b thorough wit ur stream subjects.

Page 64: Quantitative

IT-students hv to be thorough with C, C++, data structures, ADA, OS, software engineering, Unix, logic design (MUX, DEMUX....)non-IT students shud also know the basics i.e. C, C++ (t s a MUST)

Whatever u mention in the resume b thorough with it. If u hd done ny projects n ur previous semesters....b perfect in that. The que that wer asked  for me was:

I entred d room, wishd him..shook hands... 1.  xplain abt ur project (bth 1 n 2) t took nearly 10-15 min to ans tis que..      i hd done 2 projects. I totally impressed him throu my 1st proj....wen i started with 2nd one...he said "no,don wan...! seems u kno t too." 2.  diff b/w virtual func n pure virtual func? n where is t used...??  i didn kno its appln, I said "No..!idea" 3.  diff b/w C n C++? 4.  que on poly morphism, inheritance along with examples... 5.  wht s the default access level in case of a structure? ans:public 6.  wht s d diff b/w paging & segmentation, how do they serve our purposes..? 7.  prove 5=8... 8.  Xplain bubblesort n derive its time complexity n space complexity....?? 9.  code to xchg 2 numbers without usin a 3rd var. http://www.ChetanaS.org10. Name some unix commands used by a sys admin n xplain each...???I named some 5...11. y shud v hire you....???? Be +ve, tel somthin abt d comp n abt ur performance den correlate...12. wht do u think abt ur performance in tis rnd...??? I said "I think, I did well..! n i deserve his comp....!"He replied by  laughing...."Yes Ranjan..!u deserve this...take this form n get ready for ur HR..Al de bst..!!!"

I ws felicitous.....I ws flying high in the sky. I hd tech rnd for 35 min....Aftr tech rnd, if u get a form 4m d interviewer or 4m somone....ts guranteed that u r only a few inches awzy 4m Wipro. So now, u gotta fill d form with utmost care....cos the HR ll refer to this form n ask u d Que.....Be Bold enuff. u shud remebe wht al u hv filled...

Be enthusiastic, don get outwear.....

3) Two trains travel in the same direction with 56kmph speed. The fast train crosses a person standing, in 78.9sec. Find the length of the fast train.4) In election polling, the person who got 30% of the votes was defeated with a difference of 15000 votes. What is the number of votes voted for the candidate who won?

7) A sells an article to B for a profit of 50%. Similarly,B to C,C to D,D to E, for a profit of 50%. If the difference of prices of C and E is 110, at what price C sold to D?

8) In a class, average of 10 students is 19.When 1 person is removed, and then the average increases by 1/2. Find out the age of the person who is removed. 1)14  2)14 1/2  3)15  4 4)15 1/2

9) A clock shows 1hr10min, how many degrees minute hand has to travel to meet hours

Page 65: Quantitative

hand?http://www.ChetanaS.org2) One camel need to carry 45 bananas. Its capacity is 15 bananas at a time. Distance between two cities is 15miles. It eats one banana for every mile. So, max no. of bananas which it can deliver are: http://www.ChetanaS.com

3) A no. when divided by 4,7 leaves ... remainder and when divided by 3,8 leaves .. remainder. but when divided by 5 it is divisible. No. is:

4) A problem relating to weight lifter. His weight, height are given.

5) Payment to a clerk. Rs1200 for 2 weeks,works 7 hrs daily. By mistake he takes 30min lunch as working hour. what is the net amount given to him?

6) 8 balls of different colors. 2 are taken out. . Find probability to get a blue ball and red ball such that when a ball it taken out it is again placed in the bag.

1.) two pencils cost 8 cents. then 5 pencils cost?(20 cents)

2. A work is done by the people in 24 minutes. One of them can do this workalonely in 40 minutes. How much time to do the same work for the second person?(60 minutes)

3. A car is filled with four and half gallons of fuel for a round trip. Fuel is taken 1/4 more in going then coming. What is the fuel consumed in coming up? (2 gallons)

4.Low temperature at the night in a city is 1/3 more than 1/2 high as higher temperature in a day. Sum of the low tem. and highest temp. is 100 degrees. Then what is the low temp? (40 deg.)

6. A salesperson multiplied a number and get the answer 3, instead of that number devided by 3. what is the answer he actually has to get?

1 x 3 = 3so number = 1devided by 3, the ans. is 1/3.

7. A ship started from port and moving with I miles per hour and another ship started from L and moving with H miles per hour. At which place these two ships meet?

|----|----|----|----|----|----| port G H I J K L

8. A building with height D shadow upto G. A neighbour building with what height shadows C feet.

|----|----|----|----|----|----|----| A B C D E F G H

9. A person was fined for exceeding the speed limit by 10 mph. Another person was also fined for exceeding the same speed limit by twice the same. If the second person was travelling at a speed of 35 mph, find the speed limit. (15 mph)

Page 66: Quantitative

10.A bus started from bustand at 8.00am, and after 30 minutes staying at destination, it returned back to the busstand. The destination is 27 miles from the busstand. The speed of the bus is 18mph. In return journey bus travels with 50% fast speed. At what time it returns to the busstand? (11.00am).

11.In a mixture, R is 2 parts, S is 1 part. In aoder to make S to 25% of the mixture, howmuch r is to be added?

12. wind flows 160 miles in 330 min, for 80 miles how much time required.

13. with 4/5 full tank vehicle travels 12 miles, with 1/3 full tank how much distance travels ( 5 miles )

14. two trees are there. one grows at 3/5 of the other in 4 years, total growth of trees is 8 ft. what growth will smaller tree will have in 2 years ( < 2 ft. )

15. A storm will move with a velocity of towards the centre in hours, At the same rate how much far will it move in hrs. (but the answer is 8/3 or 2 2/3)

6. A sales person multiplied a number and get the answer is 3, instead of that number divided by 3. what is th answer he actually has to get ? (1/3).

8. A building with hight D ft shadow upto G A neighbour building with what height shadow C ft is (B ft.)

!_____!_____!_____!_____!_____!_____!_____! A B C D E F G H

9. A person was fined for exceeding the speed limit by 10 mph.Another person was also fined for exceeding the same speed limit by twice the same. If the second person was travelling at a speed of 35 mph. find the speed limit (15 mph)

10. A bus started from bustand at 8.00a m and after 30 min staying at destination, it returned back to the bustand. the destination is 27 miles from the bustand. the speed of the bus 50 percent fast speed. at what time it retur4ns to the bustand (11.00)

11.in a mixture, R is 2 parts, S is 1 part. in order to make S to 25% of the mixture, howmuch R is to be added ( one part).

12. wind flows 160 miles in 330 min, for 80 miles how much time required.13. with 4/5 full tank vehicle travels 12 miles, with 1/3 full tank how much distance travels ( 5 miles).

14. two trees are there. one grows at 3/5 of the other. in 4 years, total growth of trees is 8 ft. what growth will smaller tree will have in 2 years. (<2ft)

1. My father has no brothers. he has three sisters who has two childs each.

Page 67: Quantitative

1> my grandfather has two sons (f) 2> three of my aunts have two sons(can't say) 3> my father is only child to his father(f) 4> i have six cousins from my mother side(f) 5> i have one uncle(f)

2. Ether injected into gallablader to dissolve galstones. this type oneday treatment is enough for gallstones not for calcium stones. this method is alternative to surgery for millions of people who are suffering from this disease. 1> calcium stones can be cured in oneday (f) 2> hundreds of people contains calcium stones(can't say) 3> surgery is the only treatment to calcium stones(t) 4> Eather will be injected into the gallbleder to cure the cholestrolbased gall stones(t).

3. Hacking is illigal entry into other computer. this is done mostly because of lack of knowledge of computer networking with networks one machine can access to another machine. hacking go about without knowing that each network is accredited to use network facility.

1> Hacking people never break the code of the company which theywork for (can't say).2> Hacking is the only vulnerability of the computers for the usageof the data.(f)3> Hacking is done mostly due to the lack of computer knowledge (f).

(there will be some more questions in this one )

Page 68: Quantitative

4. alphine tunnels are closed tunnels. in the past 30 yrs not even a single accident has been recorded for there is one accident in the rail road system. even in case of a fire accident it is possible to shift the passengers into adjacent wagons and even the living fire can be detected and extinguished with in the duration of 30 min.

1> no accident can occur in the closed tunnels (True)2> fire is allowed to live for 30 min. (False)3> All the care that travel in the tunnels will be carried by rail shutters.(t)4>

5. In the past helicopters are forced to ground or crash because of the formation of the ice on the rotors and engines. a new electronic device has been developed which can detect the watercontent in the atmosphere and warns the pilot if the temp.is below freezing temp. about the formation of the ice on the rotors and wings.

1> the electronic device can avoid formation of the ice on the wings (False).2> There will be the malfunction of rotor & engine because of formation of ice (t)3> The helicopters are to be crashed or down (t)4> There is only one device that warn about the formation of ice(t).

6.In the survey conducted in mumbai out of 63 newly married house wives not a single house wife felt that the husbands should take equal part in the household work asthey felt they loose their power over their husbands. inspite of their careersthey opt to do the kitchen work themselves after coming back to home. thewives get half as much leisure time as the husbands get at the week ends.

1> housewives want the husbands to take part equally in the household(f)2> wives have half as much leisure time as the husbands have(f)3> 39% of the men will work equally in the house in cleaning and washing3>

7. copernicus is the intelligent. In the days of copernicus the transport and technologydevelopment was less & it took place weeks to comunicate a message at that time. wherein we can send it through satellite with in no time ----------.even with this fast developments it has become difficult to understand each other.

1> people were not intelligent during Copernicus days (f).2> Transport facilities are very much improved in noe a days (can' say)3> Even with the fast developments of the techonology we can't live happily.

(can't say)4> We can understand the people very much with the development of

communication (f).

Q8) senior managers warned the workers that because of the intfoductors of japanese industry in the car market. There is the threat to the workers. They also said that there will be the reduction in the purchase of the sales of car in public.the interest rates of the car will be increased with the loss in demand. 1> japanese workers are taking over the jobs of indian industry.(false) 2> managers said car interests will go down after seeing the raise in interest rates.(true) 3> japanese investments are ceasing to end in the car industry.(false) 4> people are very much interested to buy the cars.(false)Q9) In the totalitariturican days,the words have very much devalued.In the present day,they are becoming domestic that is the words will be much more devalued. In that

Page 69: Quantitative

days, the words will be very much effected in political area.but at present,the words came very cheap .we can say they come free at cost. 1> totalitarian society words are devalued.(false) 2> totalitarian will have to come much about words(t) 3> The art totalitatian society the words are used for the political speeches. 4>

Q10) There should be copyright for all arts. The reele has came that all the arts has come under one copy right society,they were use the money that come from the arts for the developments . There may be a lot of money will come from the Tagore works. We have to ask the benifiters from Tagore work to help for the development of his works. 1> Tagore works are came under this copy right rule.(f) 2> People are free to go to the because of the copy right rule.(can't say) 3> People gives to theater and collect the money for development.(can't say) 4> We have ask the Tagore resedents to help for the developments of art.(can't say)

4 dis section prepare old papers sometimes ull get dem withot changing nomericals also dis section is very easy 1st prepare d qstns below den ull definitely get a gud score 4 apti AGES4)Joe's father will be twice his age 6 years from now. His mother was twice his age 2 years before. If Joe will be 24 two years from now, what is the difference between his father's and mother's age? a) 4 b) 6 c) 8 d) 10

1).In a mixture, R is 2 parts, S is 1 part. In aoder to make S to 25% of the mixture, how much R is to be added?AVERAGE1) gavaskar average in first 50 innings was 50 . after the 51st innings his average was 51 how many runs he made in the 51st innings4)THREE COMPANIES WORKING INDEPENDENTLY AND GET SAVINGS 10%, 20%, 25%. IF THE COMPANIES WORK JUST OPPOSITE THEN WHAT WILL BE THE NET SAVING?BOATS1) If a man can swim downstream at 6kmph and upstream at 2kmph, his speed in still water is:a. 4kmph b. 2kmph c. 3kmph d. 2.5kmph

CAR1)A car is filled with four and half gallons of oil for full round trip. Fuel is taken 1/4 gallon more in going than coming. What is the fuel consumed in coming up?Sol Before the trip, car is filled with = 4 ½ gallon of oilLet ‘X’ be the quantity of fuel consumed for the trip in one directionThe fuel consumed while going = X + ¼The fuel consumed while coming = XTherefore, the fuel consumed for the trip = (X + ¼) + X = 4 ½ð 2X + ¼ = 4 ½ => 2X = 4 ½ - ¼ => 2X = 4 ¼ => X= 2 gallons approxCHAIN RULE1)A can copy 50 papers in 10 hours while both A & B can copy 70 papers in 10 hours. Then for how many hours required for B to copy 26 papers? 136)Three men goes to a hotel to stay ,the clerk says $30 per room/day so all the three plans to stay in one room so each pays $10.After some time the clerk realises that he

Page 70: Quantitative

made a mistake of collecting $30 but the room cost only $25,there fore he decides to return $5 to them so he calls the roomboy and gives him $5 asking him to return. The roomboy keeps $2 with him and he returns only $3($1 for each).Now Totally all have paid $9 each($27)+rommboy $2 which is equal to $27.where did $1 go, who has made the mistake?9)Which is more economical of the followinga)2kg -- 30/- b)8kg -- 160/- c)5kg -- 80/-10)Satish earns 240 weekly.12% of big amount + earning weekly = 54011) Bhanu spends 30% of his income on petrol on scooter. ¼ of the remaining on house rent and the balance on food. If he spends Rs.300 on petrol then what is the expenditure on house rent? a) Rs.525 b) Rs.1000 c) Rs.675 d) Rs.175

12) A sporting goods store ordered an equal number of white and yellow balls. The tennis ball company delivered 45 extra white balls, making the ratio of white balls to yellow balls 1/5 : 1/6. How many white tennis balls did the store originally order for?a) 450 b) 270 c) 225 d)None of these13) There is a circular pizza with negligible thickness that is cut into 'x' pieces by 4 straight line cuts. What is the maximum and minimum value of 'x' respectively?a) 12,6 b) 11,6 c) 12,5 d) 11,514) The total expense of a boarding house are partly fixed and partly variable with the number of boarders. The charge is Rs.70 per head when there are 25 boarders and Rs.60 when there are 50 boarders. Find the charge per head when there are 100 boarders.a) 65 b) 55 c) 50 d) 45

COUNTERFEIT1)in 80 coins one coin is counterfiet what is minimum number of weighings to find out counterfiet coin

CURVES1)STUDY THE NATURE OF THE CURVES Y==LOGX, XY==C etc2)curve was given & in option equation was given , you have to mark the correct equation(curve and equation was of general logx,ex ,sinx,cosx,tanx,x curves)3)values of x &f(x) was given (like x=1000&f(x)=3) and equations was given4)WHICH SHAPE WILL BE OBTAINED BY USING THESE VALUES OF X,YX Y0 0.0000110 1.02100 1.721000 3.009999 4.72

4a)WHICH SHAPE WILL BE OBTAINED BY USING THESE VALUES OF X ,YXY00.00001101.021001.7210003.0099994.725)WHICH EQUATION THAT BEST SUITS THIS CURVE

Page 71: Quantitative

A LINE CUTS X AT -1 WHEN Y=0 AND X=0 WHEN Y=3 AND GOES UPWARDY X

1)In a well of 20feet depth, a frog jumps 5feet up in the morning and comes 4feet down in the evening,on which day the frog gets out of the well.

1) LCM of x and y is 36 and HCF of x and y is 4. If x = 4 then which of the following is definitely not true?a)Y is divisible by 3 b)Y is divisible by 6 c)Y is divisible by 9 d)X+Y is divisible by LOGARITHMS1) log .342==..., log . 343==.... then log .341==????? etc...2)value of log3.142& log3.143 was given and log3,141 was asked so you have to only see on answer and mark3)If log 0.317=……… and log 0.318=………. Then find the value of log 0.319.

1)If the vertex (5,7) is placed in the memory. First vertex (1,1) 's address is 1245 and then address of (5,7) is ----------12792)A 2D array is declared as A[9,7] and each element requires 2 byte. If A[ 1,1 ] is stored in 3000. Find the memory of A[8,5] ? 31063)one circular array is given (means the memory allocation takes place like a circular fashion ) dimension (9X7) .starting address is 3000.find the address of (2,3)..5554)Some has got the qs. As memory allocation takes place coloumnwise.u have to find out the address.5)The size of a program is N. And the memory occupied by the program is given by M = square root of 100N. If the size of the program is increased by 1% then how much memory now occupied? Ans 0. 5%( SQRT 101N)6)In a two-dimensional array, X (9, 7), with each element occupying 4 bytes of memory, with the address of the first element X (1, 1) is 3000, find the address of X (8, 5). Ans 3212

MIXTURE1)16 litre can, 7 litre can,3 litre can, the customer has to be given 11 litres of milk using all the three cans only explain?

NUMBER SYSTEM1) The number 362 in decimal system is given by (1362)x in the X system of numbers find the value of X a}5 b) 6 c) 7 d) 8 e) 92) Decimal no 319 is equal to 1234.This 1234 belongs to which Number systems3)194 base 10 = ____ base 5 (1234)7) In which of the system, decimal number 384 is equal to 1234?10)The base 5 representation of the decimal number 2048 is _____.

PARTNERSHIP2) A sum of money is divided among A, B and C such that for each rupee A gets, B gets 65paise and C gets 35paise. If C's share is Rs.560, the sum is …a)2400 b)2800 c)3200 d)3800

PERCENTAGE1) What percent of 60 is 12?Ans. 20%

Page 72: Quantitative

2) A report consists of 20 sheets each of 55 lines and each such line consists of 65 characters. This report is retyped into sheets each of 65 lines such that each line consists of 70 characters. The % reduction in the number of sheets is closest toa)20 b)5 c)30 d)353) Amal bought 5 pens, 7 pencils and 4 erasers. Rajan bought 6 pens, 8 erasers and 14 pencils for an amount which was half more than what Amal had paid. What % of the total amount paid by Amal was paid for pens?a)37.5% b)62.5% c)50% d)None of these

PERMUTATIONS1)How many 3 digits with 3 Distinct digits are there from 100-1000.

POWER OF2) 8 to the power of x is 32,what is the value of x?3)Which is a/not a power of 2 or 3.5)WHICH ONE WILL BE THE EXACT POWER OF 3(i) 2768 (ii)2678 (III) 2187

PRIME NUMBERS1)Find the highest prime number that can be stored in an 8bit computer.5)What is the largest prime number that can be stored in an 8-bit memory? 251 or 1276)Which is the largest prime number that can be stored in a 9-bit register?

PROBABILITY1)in a company 30% are supervisors and 40% employees are male if 60% of supervisors are male. what is the probability that a randomly choosen employee is a male or female?2) 12 Black sox and 12 White sox mixed in a box, a pair of sox is picked at a time, in which pick/how many pick ,to get the right pair(black & black or white & white)?3)Two coins one with HEAD IN BOTH SIDES and the other coin HEAD IN ONE SIDE AND TAIL IN THE OTHER SIDE is in a box, a coin is taken at random and FOUND HEAD IN ONE SIDE .what is the probability that THE OTHER SIDE IS HEAD?4) There are 5 distinct pairs of white socks and 5 pairs of black socks in a cupboard. In the dark, how many socks do I have to pull out to ensure that I have at least 1 correct pair of white socks? a)3 b)11 c)12 d)16 

Page 73: Quantitative

PROBLEMS ON NUMBERS4) WHAT IS THE NUMBER OF ZEROS AT THE END OF THE PRODUCT OF THE NUMBERS FROM 1 TO 1005)9/10 or 10/11 which is greater.6) (x-y/3)-(y-x/3)=?15) If the numerator of a fraction is increased by 25% and denominator decreased by 20%, the new value is 5/4. What is the original value?a) 3/5 b) 4/5 c) 7/8 d) 3/716)The value of ¾ + 5 / 36 + 7 / 144 + …….+17 / 5184 + 19 / 8100 isa)0.99 b)0.98 c)0.95 d)None of these17) A student's grade in a course is determined by 6 quizzes and one examination. If the examination counts thrice as much as each of the quizzes, what fraction of final grade is determined by the examination?a)1/6 b)1/5 c)1/3 d)¼18) When ¾ of a unit's digit is added to the ten's digit of a two number, the sum of the digits becomes 10. If ¼ of the ten's digit added to the unit's digit, then the sum of the digits is 1 less than the previous. Find the number.a)94 b)84 c)48 d)8819) A person had to multiply two numbers. Instead of multiplying by 35, he multiplied by 53and the product went up by 540. What was the raised product?a)780 b)1040 c)1590 d)1720

RECTANGULAR1) WHICH IS NOT A SIDE OF A RECTANGULAR1. (2,3,4) 2.(3,4,7) 3. (3,5,9)3) If the length of a rectangle is increased by 30% and the width is decreased by 20%, then the area is increased by...(a) 10% (b) 5% (c) 4% (d) 20% (e) 25%4) The length of a rectangle is increased by 60%. By what % would the width have to be decreased to maintain the same area? a)30% b)60% c)75% d)37.5%

TEMPERATURE2)FOR TEMPERATURE A FUNCTION IS GIVEN ACCORDING TO TIME : ((t**2)/6) + 4t +12 WHAT IS THE TEMPERATURE RISE OR FALL BETWEEN 4.AM TO 9 AM2)FOR TEMPERATURE A FUNCTION IS GIVEN ACCORDING TO TIME : ((t**2)/6) + 4t +12 WHAT IS THE TEMPERATURE RISE OR FALL BETWEEN5 PM to 8 PM3)Low temperature at the night in a city is 1/3 more than 1/2 high as higher temperature in a day. Sum of the low tem.  and highest temp. is 100 degrees. Then what is the low temp? (40 deg.)

Page 74: Quantitative

TIME AND DISTANCE3) falling height is proportional to square of the time. one object falls 64cm in 2sec than in 6sec from how much height the object will fall.4)A car has run 10000 miles using 5 tyres interchangably,To have a equal wornout by all tyres how many miles each tyre should have run. answer 4000 miles/tyre5)A person, who decided to go to weekened trip should not exceed 8 hours driving in a day. Average speed of forward journey is 40 m/h. Due to traffic in sundays, the return journey average speed is 6)A ship started from port and moving with I miles per hour and another ship started from L and moving with H miles per hour. At which place these two ships meet?|----|----|----|----|----|----|port G H I J K L7)A person was fined for exceeding the speed limit by 10mph. Another person was also fined for exceeding the same speed limit by twice the same. If the second person was traveling at a speed of 35 mph, find the speed limit. Sol: Let ‘x’ be the speed limitPerson ‘A’ was fined for exceeding the speed limit by = 10mphPerson ‘B’ was fined for exceeding the speed limit by = twice of ‘A’ = 2*10mph=20mphgiven that the second person was traveling at the speed of 35mph => 35mph – 20mph = 15mphA bus started from bustand at 8.00am, and after 30 minutes stayingat destination, it returned back to the busstand. The destinationis 27 miles from the busstand. The speed of the bus is 18mph. Inreturn journey bus travels with 50% fast speed. At what time it returns to the busstand? (11.00am).9) wind flows 160 miles in 330 min, for 80 miles how much time required.10)A storm will move with a velocity of towards the centre in hours,At the same rate how much far will it move in hrs.( but the answer is 8/3 or 2 2/3 )11)If A is traveling at 72 km per hour on a highway. B is traveling at a speed of 25 meters per second on a highway. What is the difference in their speeds in meters per second?(a) 1/2 m/sec (b) 1 m/sec (c) 1 1/2 m/sec (d) 2 m/sec (e) 3 m/sec12)A traveler walks a certain distance. Had he gone half a kilometer an hour faster \, he would have walked it in 4/5 of the time, and had he gone half a Kilometer an hour slower, he would have walked 2 ½ hr longer. What is the distance?a)10 Km b)15 Km c)20 Km d)Data Insufficient13) A ship leaves on a long voyage. When it is 18 miles from the shore, a seaplane, whose speed is 10 times that of the ship is sent to deliver mail. How far from the shore does the seaplane catch upo with the ship?a)24 miles b)25 miles c)22 miles d)20 miles14) In a circular race track of length 100 m, three persons A, B and C start together. A and B start in the same direction at speeds of 10 m/s and 8 m/s respectively. While C runs in the opposite at 15 m/s. When will all the three meet for the first time on the after the start?a)after 4s b)after 50s c)after 100s d)after 200s15) If the distance traveled (s) in time (t) by a partile is given by the formula s = 1+ 2t+3t2+4t3 , then what is the distance travelled in the 4th second of its motion?a)141m b)171m c)243 m d)313 m16) A non stop bus to Amritsar overtakes an auto also moving towards Amritsar at 10 am. The bus reaches Amritsar at 12.30 pm and starts on the return journey after 1 hr. On the way back it meets the auto at 2 pm. At what time the auto will reach Amritsar?

Page 75: Quantitative

a) 2.30pm b)3.00pm c)3.15pm d)3.30pm

TRAINS1)TWO STATIONS A & B ARE 110 KM APART. ONE TRAIN STARTS FROM A AT 7AM, AND TRAVELS TOWARDS B AT 20KMPH. ANOTHER TRAIN STARTS FROM B AT 8 AMAND TRAVELS TOWARDS A AT 25KMPH. AT WHAT TIME WILL THEY MEET?A. 9 AM B. 10 AM C. 11 AM D. 10.30 AM

TREE1) 900 M WIDE 3000 M WIDTH SOMETHING I CAN’T REMEMBER SOME VALUES ARE GIVEN BY AIR PER M Rs. 4 BY GROUND PER M Rs. 5THEN WHERE U WILL CUT2)Two trees are there. one grows at 3/5 of the other in 4 years, total growth of trees is 8 ft. what growth will smaller tree will have in 2 years ( < 2 ft. ) ie 1 ½ feet

TRIANGLES1) Given the length of the 3 sides of a triangle. Find the one that is impossible? (HINT : sum of smaller 2 sides is greater than the other one which is larger)2)3 angles or 3 sides r given. Which will form a triangle?

UNITS2)(energy * time * time )/(mass * dist) = distance3)(momentum * velocity)/(force * time) = velocity5)Find the physical quantity represented by MOMENTUM *VELOCITY] / [LENGTH * ACCELERATION]?

VENN DIAGRAM1)Venn Diagram kinda ques. Some know English, some French, some German……how many know two languages…..2)VENN DIAGROM below1. HOW MANY PERSON KNOW ENGLISH MORE THAN FRENCH2. HOW MUCH % OF PEOPLE KNOWS ALL THE 3 LANGUAGES3. HOW MUCH % OF PEOPLE THOSE WHO KNOWS FRENCH AND GERMAN AND NOT ENGLISH

WEIGHTS2)A truck contains 150 small packages, some weighing 1 kg each and some weighing 2 kg each. how many packages weighing 2 kg each are in the truck if the total weight of all the packages is 264 kg?(a) 36 (b) 52 (c) 88 (d) 124 (e) 114

 1. Fathers age 3years back is twice the son’s age.the sum of father’s and son’s age after 10 years from now is 66.what is father’s age?

2) 1 3 3 6 5 9 7 12__ __ __3) 3 8 27 112 __ ___4. There r 6 people named madhavi,pramod,pravven,asha,gopal,nisha.they r seated around the table.1.madhavi and pramod always are left to a women.

Page 76: Quantitative

2.asha and gopal are always opposite to a man.3.something condition didn’t remember.(some reasoning type). 5. If a man had travelled 3km/hr faster he wuld have reached 40 min earlier,if he wuld have travelled 2 km/hr slower he wuld have reached 40 min late.what is distance he had travelled. 6. Jhon works for 60 days.for the day he is present he is paid 7.00/- and for the day he is absent he is paid 3.00/- per day.if he is paid 170/-.how many days he is present. 7. There r 4 links in agold chian .1 has 5links,other has 4 links and the other 3 links..goldsmith takes 50.00paise to open the link and 1.00/- to attach the link.what is least cost to bake whole bracelet. 8. A cyclist moves half the speed of tonga driver and tonga driver moves half the speed of man walking .If man walks at 3kms per hour what is the time that cyclist covers 27kms. 9. There r 600 pages .there is 1 error on avg per page.what is the pbt that there are n errors per page. 10.A square is of side 1 km.A man travels first 2 sides with the 30 mph and third side with 60 kmph.what is the speed that man has to travel the fourth side if the avg speed is 60kmph 11.A Student got 78 marks in avg of 4 subjects.the avg odf 5 subjects is 80.what is the amrk he got in 5th subject. 12.How many kg of salt at 42 paise per kg must a man mix with 25kg of salt at 24 paise per kg so that he may, on selling the mixture at 40 paise per kg gain 25% on the outlay?A)15 kg         B)20 kg             C)25 kg             D)28 kg             E)none of these 13.Find the least number which when divided by 20,25,30,36 and 48 leaves the remainders 15,20,25,31,43 respectively.A)3685 B)3585             C)3595             D)3535             E)None of these

15.  For china assuming a linear growth in LMVs population, extrapolate nearly, the year beyond 2030 when the growth in population will be 108%. A)2044        B)2032             C)2050             D)2038             E)None of these    16. A sheet of metal in the form of a sector of a circle of angle 90 degrees and radius 16cm is folded to form an open conical cup. The capacity of the cup isA)64 sqrt(15)cm3             B)64 sqrt(5/3)TTcm3             C) 64 sqrt(5)TT             D) 64/3sqrt(15)cm3    E)None of these

18. Three faces of a fair die are Yellow, two faces red and one blue.The die is tossed three times. The probability that the colours, yellow,red and blue appear in the first,second and third tosses respectively is A)1/36             B)1/18             C)1/32             D)1/37             E)None of these 19. At the foot of a mountain the elevation of its summit is 45 degrees. After ascending one KM towards the mountain upon an incline of 30 degrees, the elevation changes to 60 degrees. Find the Height of the mountain? A)1.333Km       B)1.366Km       C)1.233Km       D)1.266Km      E)None of these

20. The average of a couple was 23Yrs when they were married 5Yrs ago.The avg age of the couple and a child, who was born during the interval, is 20Yrs now. How old is the child now ?A)2 Yrs B)4 Yrs C)3 Yrs D)1 Yrs E)None of these 21.  Two liters of a mixture of alcohol and water contain 10% water. This is added to 4 litres of another mixture containing 8% water and half a litre of water is then added to

Page 77: Quantitative

the whole. Find the percentage of water in the resulting mixture.A)17 3/11% B)15 3/13% C)14 12/13% D)15 9/13% E)None of these 1)A starts a bussiness inv Rs.4000 n B joins by inv Rs.6000 after 3 months .A leaves wen it ws 4 months 21 yr.wat s the ratio of their profit? 2)a circular disc hs 6 diff fruits  n nams of thos frts wer mentiond in wat fashion they shud b arranged so tat  i)mango &pineappl r 2gthr  ii)mango n grapes r nt 2gthr.....  3)a monkey tries 2 climb a pole 21m long it climbs 6m in 1sec n cms dwn by 3m in th nxt sec.in hw mny sec wil it reach the top of th pole? 4)SI  is gn as Rs.6000 rate of int s 10% anually for 4 yrs .find CI? 5)AC,BG,CE,DI,_,EG fil th series 6)sm thing rltd 2 north ,west  n all tat. lk man walks in a part drn  n taks turn sm thing lk tat.and finally v wer asked 2 find the displacement between 2 pts. 7)a bag A has 6 white,4black balls.bag B has 5 white n 4 blk balls.wen a card s drawn frm a pack if it s red spade a white ball s picked frm bag A n few conds wer gn finally v had 2 find the probb of picking blk bal frm bagB. 8)if 'x' books cost Rs.15 n 'y' cost Rs.8 wat s the avg cost of each book. 9)a paragraph ws gn n based on tat few qns wer asked. 10)teaching comp s made compulsory in schools 4 effective learning. and few stmnts wer gn v had 2 pick th sent tat weakens the stmnt 11)there r 7 tapes in which one of thm s news tape n rest r A,B,C,D,E,F are music tapes.few condns wer gn lk c must b played jst bfr F etc n v had 2 pick th right  time slot 2 play news tape. 12)3 horses n their names wer gn n their owners nams wer also gn n there ws sm problem wit every horse n v had 2 find which of th 3 horses runs fast. 2- wht is the difference between SI and CI of 1000Rs where Rate =10%,time =1 yr for SI  and Rate is 10.5 in CI, time=1yr.3-there was a ques of average.   4-series ques.5-a ques was A salary+ B salary=10000, if A spend 70% of his income, and B spends 80% of his income , the diff btw there spnd money is 500, then wht is the salary of A.3) If 10 oct 2004 was saturday. Then which day will be on 10 oct 1986........Infosys Placement Paper 28th Sep 2005 by Bharti This is so strange that infy people gave same paper on two dates 28 oct 2004 and 28 sep 2005these two dates having more than 9 month gapthat means on 28 sep 2005 some persons were writing same paper that they have written on 28oct 2004IT MAY BE POSSIBLE YOU MAY ALSO FACE THE SAME PAPER IN INFY EXAMBEST OF LUCKthe question are may not be in correct sequence   i am giving you my answers also (that may not be correct)here is that paperINFOSYS QUESTION BANKQuestion:A fly is there 1 feet below the ceiling right across a wall length is 30m at equal distance from both the ends. There is a spider 1 feet above floor right across the long wall equidistant from both the ends. If the width of the room is 12m and 12m, what distance is to be traveled by the spider to catch the fly, if it takes the shortest path?Question:Ramesh sit around a round table with some other men. He has one rupee more than his

Page 78: Quantitative

right person and this person in turn has 1 rupee more than the person to his right and so on, Ramesh decided to give 1 rupee to his right & he in turn 2 rupees to his right and 3 rupees to his right & so on. This process went on till a person has 'no money' to give to his right. At this time he has 4 times the money to his right person. How many men are there along with Ramesh and what is the money with poorest fellow.Question related to probabilities of removing the red ball from a basket, given that two balls are removed from the basket and the other ball is red. The basket contains blue, red, yellow balls.Venkat has 1boy and 2daughters.The product of these children age is 72.The sum of their ages give the door number of Venkat boy is elder of three. Can you tell the ages of all the three?Question:L: says all of my other 4 friends have moneyM: says that P said that exact one has moneyN: says that L said that precisely two have moneyO: says that M said that 3 of others have money.P: Land N said that they have money.All are liars.Who has money & who doesn't have?Postman has a data of name surname door number, pet name of 4 families. But only one is correct for each family. There are a set of statements & questions.4 couples have a party. Depending on the set of statements, find who insulted whom and who is the host of the party.5 women given some of their heights (tall, medium, short), hair(long, plaited), studs(Black or Brown), sari,2 medium, 2-short.Tall->no sari. Plaited->medium. Answer the combinations.Question:A person has to go both Northwards & Southwards in search of a job. He decides to go by the first train he encounters. There are trains for every 15 min both southwards and northwards. First train towards south is at 6:00 A.M. and that towards North is at 6:10. If the person arrives at any random time, what is the probability that he gets into a train towards North?

Page 79: Quantitative

Question:A person has his own coach&. Whenever he goes to railway station he takes his coach. One day he was supposed to reach the railway station at 5 O'clock. But he finished his work early and reached at 3 o'clock. Then he rung up his residence and asked to send the coachimmediately. He came to know that the coach has left just now to the railway station. He thought that the coach has left just now to the railway station. He thought that he should not waste his time and started moving towards his residence at the speed of 3 miles/hr. On the way, he gets the coach and reaches home at 6 o'clock. How far is his residence from railway station?Question:Radha, Geeta & Revathi went for a picnic. After a few days they forgot the date, day and month on which they went to picnic. Radha said that it was on Thursday, May 8 and Geeta said that it was Thursday May 10. Revathi said Friday Jun 8. Now one of them told all things wrongly, others one thing wrong and the last two things wrongly. If April 1st is Tuesday, what is the right day, date and month?Question:There is 66x33m rectangular area. Ram is 11/8 times faster than Krishna. Both of them started walking at opposite ends and they met at some point then, Ram said "See you in the other end" then they continued walking. After some time Ram thought he will have tea so he turned back walked back 15 meters then he changed his mind again and continued walking. How much Krishna has traveled by the time they meet?Question:There are 5 burglars and once went to a bakery to rob it obviously. The first guy ate 1/2 of the total bread and 1/2 of the bread. The second guy ate 1/2 of the remaining and 1/2 of the bread. The third guy, fourth guy and fifth guy did the same. After fifth guy there is no bread left out. How many bread are there?Question:All members belonging to D are members of A.All members belonging to E are members of D.All members belonging to C are members of both A & D.Some members of A does not belong to D.All members belonging to D are members of E. 5 questions are there.Write each statements true or false:-A. The sum of the 1st three statements and the 2nd false statementgives the true statement.B. The no. of true statements false statement.C. The sum of 2nd true statement and 1st false statement gives thefirst true statement.D. There are at most 3 false statements.E. There is no two consecutive true statements.Question:There are twelve consecutive flags at an equal interval of distance. A man passes the 8th flag in 8 seconds. How many more seconds will he take to pass the remaining 4 flags?Question:A person has to cover the fixed distance through his horses. There are five horses in the cart. They ran at the full potential for the 24 hours continuously at constant speed and then two of the horses ran away to some other direction. So he reached the destination 48 hours behind the schedule. If the five horses would have run 50 miles more, then the person would have been only 24 hours late. Find the distance of the destination?

Page 80: Quantitative

Question:A boat M leaves shore A and at the same time boat B leaves shore B. They move across the river. They met at 500 yards away from A and after that they met 300 yards away from shore B without halting at shores. Find the distance between the shore A & B.

Question:A person was going through train from Bombay to Pune. After every five minutes he finds a train coming from opposite direction. Velocity of trains are equal of either direction. If the person reached Pune in one hour then how many trains he saw in the journey?Question:Food grains are to be sent to city from godown. Owner wants to reach the food grains at 11 O' Clock in the city. If a truck travels at a speed of 30km/hr then he will reach the city one hour earlier. If the truck travels at a speed of 20km/h then he will reach the city one hour late. Find the distance between the godown to city. Also with which speed the truck should travel in order to reach at exactly 11 o'clock?Question:There are five persons A,B,C,D,E whose birthdays occur at the consecutive days. Birthday of A is some days or day before C & birthday of B is exactly the same days or day after E. D is two daysolder than E. If birthday of C is on Wednesday then find out the birthdays of other.Question:Persons say these statements.A says either Democratic or liberal wins the elections.B says Democratic wins.C says neither democratic nor liberal wins the election.Of these only one is wrong. Who wins the election?Question:Six persons A,B,C,D,E, &F went to soldier cinema. There are six consecutive seats. A sits in the first seat followed by B, followed by C and so on. If A taken on of the six seats, then B should sitadjacent to A. C should sit adjacent to A or B. D should sit adjacent to A, B or C and so on. How many possibilities are there?Question:Suppose there are four grades A, B, C, D. (A is the best and D is the worst) 4 persons Jack, Jean, Poul and Lucy wrote the final exam and made the statements like this:-1. Jack: If I will get A then Lucy will get D.2. Lucy: If I will get C then Jack will get D. Jack grade is better than Poul grade.3. Jean: If Jean doesn't get A then Jack will not get A.4. Poul: If Jack get A, then Jean will not get B, Lucy will get C, Iwon't either A or B.A bird keeper has got P pigeon, M mynas and S sparrows. The keeper goes for lunch leaving his assistant to watch the birds. Suppose p=10, m=5, s=8.a.) When the bird keeper comes back, the assistant informs that x birds have escaped. The bird keeper exclaims oh no! all my sparrows are gone. How many birds flew away?b.) When the bird keeper comes back, the assistant told him that x birds have escaped. The keeper realized that at least 2 sparrows have escaped. What is minimum no of birds that can escape?Question:Select from the five alternatives A,B,C,D,E. At the end of each question, two conditions will be given. The choices are to be filled as follows.

Page 81: Quantitative

A: If a definite conclusion can be drawn from condition 1.B: If a definite conclusion can be drawn from condition 2.C: If a definite conclusion can be drawn from condition 1 and 2.D: If a definite conclusion can be drawn from condition 1 or 2.E: No conclusion can be drawn using both conditions.Question:Person 1 says N<5Person 2 says n>5.Person 3 says 3N>20Person 4 says 3n>10Person 5 says N<8.What is the value of N?Question:There are N coins on a table. There are two players A&B. You can take 1or 2 coins at a time. The person who takes the last coin is the loser. A always starts first.  If N=7, thenA. A can always win by taking two coins in his first chance.B. B can win only if A takes two coins in his first chance.C. B can always win by proper play.D. None of the above.Question:There are N coins on a table. There are two players A&B. You can take 1or 2 coins at a time. The person who takes the last coin is the loser. A always starts first.  B can win by proper play if N is equal toA. 25B. 26C. 32D. 41E. noneQuestion:There are N coins on a table. There are two players A&B. You can take 1or 2 coins at a time. The person who takes the last coin is the loser. A always starts first.  if N<4, can A win by proper play always?Question:There are 4 parties A,B,C,D. There are 3 people x,y,z. X-says A or D will win. Y-says A will not win. Z-says B or D will not win. Only one of them is true. Which party won?

Persons A and B. Person A picks a random number from 1 to 1000. Then person B picks a random number from 1 to 1000. What is the probability of B getting number greater then what A has picked?

Page 82: Quantitative

Question:Three boys and three girls brought up together. Jim, Jane, Tom, Virgina, Dorothy, XXX. They marry among themselves to form three couples. Conditions are:-i) Sum of their ages would be the same.ii) Virgina was the oldest.iii) Jim was Dorothy's brother.iv) Sum of ages Jane+Jim and Tom+Dorothy is same.Give the three couples.Question:X^(1/3) - X^(1/9) =60. Solve for X.Question:X Z Y+X Y Z = Y Z X.Find the three digits.Question:Two boats start from opposite banks of river perpendicular to the shore. One is faster then the other. They meet at 720 yards from one of the ends. After reaching opposite ends they rest for 10mins each. After that they start back. This time on the return journey they meet at 400yards from the other end of the river. Calculate the width of the river.Question:Basketball Tournament organizers decided that two consecutive defeats will knock out the team. There are 51 teams participating. What is the maximum no. of matches that can be played.Question:The Master says to his grandmaster that me and my three cousins have ages in prime nos. only. Summation of our ages is 50. Grandmaster who knows the age of the master instantly tells the ages of the three cousins. Tell the ages of three cousins. (1 is not considered as prime no.)Question:There are two families Alens and smiths. They have two children each. There names are A,B,C,D whose ages are different and ages are less then or equal to 11. The following conditions are given:-i) A's age is three years less then his brother's age .ii) B is eldest among the four.iii) C is half the age of the eldest in Alens family.iv) The difference in sum of the ages of Alens children and smiths children is same as that of five years ago.Find the ages of all the children.Question:a,b,c,d,e are having numerical values. There are some conditions given:-a) a=c <=== b!=eb) Difference between a and c as same as difference between c and b as same as difference between a and d.c) c<a and c>d.Then find a,b,c,d,e.

Page 83: Quantitative

Question:There are six cards in which it has two king cards. All cards are turned down and two cards are opened. What is the possibility to get at least one king?Question:There are six cards in which it has two king cards. All cards are turned down and two cards are opened. What is the possibility to get two kings?

Question:There are two women, Kavitha and Shamili and two males Shyam, Aravind who are musicians. Out of these four one is a pianist, one flutist, violinist and drummer.i) Across Aravind beats pianist.ii) Across Shyam is not a flutist.iii) Kavitha's left is a pianist.iv) Shamili's left is not a drummer.v) Flutist and drummer are married.Question:When Arthur is as old as his father Hailey is now, he shall be 5 times as old as his son Clarke is now. By then, Clarke will be 8 times older than Arthur is now. The combined ages of Hailey and Arthur are 100 years. How old is Clarke?Question:The seven digits in this subtraction problem are 0, 1, 2, 3, 4, 5 and 6. Each letter represents the same digit whenever it occurs.D A D C B-  E B E G--------------------  B F E G--------------------What digit is represented by each letter?Question:The Jones has named their four boys after favorite relatives; their friends, the Smiths, have done the same thing with their three boys. One of the families has twin boys. From the following clues, can you determine the families of all seven children and their ages?i) Valentine is 4 years older than his twin brothers.ii) Winston, who is 8, and Benedict are not brothers. They are eachnamed after a grandfather.iii) Briscoe is two years younger than his brother Hamilton, But threeyears older than Dewey.iv) Decatur is 10 years old.v) Benedict is 3 years younger than Valentine; they are not related.vi) The twins are named for uncles.Question:Motorboat A leaves shore P as B leaves Q; they move across the lake at a constant speed. They meet first time 600 yards from P. Each returns from the opposite shore without halting, and they meet 200 yards from. How long is the lake?

1.420% OF 7.79 = 32.718

2 3427 / 16.53 = 202

3. 10995 /95 = 115.7365

Page 84: Quantitative

4. 43+557-247 =353

5. 3107*3.082= 9591

6. 48.7 + 24.9 - 8.7 = 64.90

7.525.0/47.8 = 118. (135-30-14)*7 - 6 +2 = 39.3/8 * 5.04=1.89

10.697 /219 =3.18

11.8/64 +64/16 = 4.1412.298*312/208 = 453.5413. 0.33 *1496 /13 = 37.98

14.0.26 + 1/8 = 0.385

15.66.17+1/3= 67.03

16. 2.84+1/4= 3.09

17. 33% OF 450 = 148.5

18. 907.54 / 0,3073= 3002

3. Two men are going along a track rail in the opposite direction. One goods train crossed the first person in 20 sec. After 10 min the train crossed the other person who is comming in opposite direction in 18 sec .After the train has passed, when thetwo persons will meet?Approx 72min check it once.

7.Thereare2diomans ,1 spadeand1 club and 1ace and also 1king ,1 jack and 1 aceare arranged in a straight line 1.The king is at third place2.Theleft of jack is a heart and itsright is king3. No two red colours arein consequtive.4.The queensareseperated by two cards. Write the orderor which suits (hearts ,clubs ) and names(jacks queensetc.)

1. 2 Marks             When Arthur is as old as his father Hailey is now, he shall be  5 times as old as his son Clarke is now. By then, Clarke will be 8 times older than Arthur is now. The combined ages of Hailey and Arthur are 100  years. How old is Clarke ?

8) Yesterday my mother asked me to buy some stamps. Stamps are available in 2 paise, 7paise, 10paise, 15paise and 20paise denominations. For three types of stamps I was asked to buy five of each. For the other two types of stamps. I was asked to buy six of each. Unfortunately I forgot which I was supposed to buy five of and which

Page 85: Quantitative

to buy six of. Luckly my mother had given me the exact money required to buy the stamps, Rs. 3. 00 and the shopkeeper was able to give me the correct stamps. Which stamps did I buy?

9) Farmer jones sold a pair of cows for Rs. 210 , On one he made a profit of ten percent and on the other he lost ten percent. Althougher he made a profit of five percent. How many did each cow originally cost him?

Question # 4-------------Find the 3 digit no. whose last digit is the square root of the first digit and second digit is the sum of the other two digits.

Question # 5-------------Meera was playing with her brother using 55 blocks. She gets bored playing and starts arranging the blocks such that the no. of blocks in each row is one lessthan that in the lower row. Find how many were there in the bottom most row?

Question # 6-------------Rahul took part in a cycling game with many others in a circular closed circuit. After pedaling for several minutes, he found that 1/5th of the cyclists ahead of him and 5/6th of the cyclists behind him together formed the total no. of participants. Howmany were participating in the race?

Question # 10-------------A company's director said during the board meeting: " The company's income from roads will be sufficient to pay 6% of the entire stock issue, but since we are paying 7. 5% interest on the preferred stock of Rs. 4, 000, 000 we are able to pay only 5% ofthe common stock". Find the value of the common stock.

Question # 1-------------A man is going to a wedding party. He travels for 2hrs when he gets a puncture. Changing tyres takes 10mins. The rest of the journey he travels at 30 miles/hr. He reaches 30mins behind schedule. He thinks to himself that if the puncture had occurred 30miles later, he would have been only 15mins late. Find the total distance traveled by the man

Page 86: Quantitative

Question # 7-------------A man was on his way to a marriage in a car with a constant speed. After 2 hours one of the tier is punctured and it took 10 minutes to replace it. After that they traveled with a speed of 30 miles/hr and reached the marriage 30 minutes late to the scheduled time. The driver told that they would be late by 15 minutes only if the 10 minutes was not waste. Find the distance between the two towns?

Question # 1-------------i participated in a race. 1/5th of those who are before me are equal to 5/6th of those behind me. what were the total number of contestants in the race?

Question # 2-------------Find the 3 digit number. Third digit is square root of first digit. Second digit is sum of first and third digits. Find the number

Question # 5-------------A girl has 55 marbles. she arranges them in n rows. the nth row consists of n marbles, the n-1th row consists of n-1 marbles and so on. what are the number of marbles in nth row?

1). A stamp collector has the habit to arrange or rearrange the stamps accordingly. while doing this he some times keeps the stamps in pairs, or in group of 3 or in 4 or in or in 6 and realises that in any case he is left with 1 stamp and when he arranges them in groups of 7 no stamps remain. what is the number of stamps he has?

3). Two wine mechants goes to a city in paris for trade. the first has 64 barrels of wine and the second one has 20. the first one pays 4barrels and 40 francs as duty at the toll gate and the second one pays by giving 2 barrels and recieves 40 francs. how much each barrel costs and how much duty one must pay for each barrel (120, 10)

4). there are 3 custoners who wants to take a hair cut and shave. there are 2 barbers who takes one quarter of an hour for a hair cut, and 5 minutes for a shave. both the barbers want to finish off and go quickly to their homes. in what time can do it.

5). we travelled to a place at the rate of 10 miles per hour and offcourse returned the same way, but owing to less traffic at the rate of 15 miles per hour. what was our relative speed.

6). there are 3 types of apples in a box. what is the number of apples we should take so that we end up with 3 apples of one kind.

8) ted while sipping coffee looking down the hall said " i think your 2 kids might have grown up. i have seen them 4 years ago. what are their ages now. " " thats not 2 but three. jane had one more in the mean time. and if we multiply their ages it would result in 96. and if we add their ages it gives your house number" said

Page 87: Quantitative

charlie. ted said" so i would not get it any way". charlie replied " why , u can have a try". what are the ages of the kids?

1. one stamp collector arranges his stamps in some order, whenever he arranges stamps in group of 2or 3or 4 or 5 or 6 one stamp left , but when he arranges in such a order that every group is having 7 number of stamps then nothing is left. How many stamps he is having. (sentence is some what changed ).

2. Two wine merchant s reached on paris gate. A is having 64 barrels wine and B is having 20 barrels . A paid 5 barrel plus 40 frank as tax and B paid 2 barrels and got 40 franks inreturn. What is the cost of one barrel and how much is tax per barrel.

3. There are two barbers both take same time for hair cut and shave. If 15 min required for hair cut and 5 min for shave. what is the possible minimum time if there are three customer in theire shop for hair cut as well as shave.

5. One race we have completed with the speed of 10 miles per hour , while returning because of smooth traffic we come back with 15miles per hour speed . So what was our avarage speed?

8. A says to B " when we met last exactly 4 years back, you were having two kids, How old they are now? "B says "Not two , three meanwhile". "The product of their ages is ninty six , and sum is equalent to your house number. "A says "You are smart" . But i am not sure tell me more. B says "Its easy you can find it "

Question :What is the sum of their ages?(6 marks).

Question # 2-------------Mr. T has a wrong weighing pan. One arm is lengthier than other. 1 kilogram on left balances 8 melons on right. 1 kilogram on right balances 2 melons on left. If all melons are equal in weight, what is the weight of a single melon?

Question # 6------------- G: I am 22 M = G + 2 H = G - 1 H: I am not the youngest. difference between M & H is 3 G is 25.

M: M is younger than G. G = 23. H = G+2. Mr. G, Mr. M, Mr. H made only one statement false. Find the ages of all three.

(I am not sure abt the statements jus chk it out with others)

Page 88: Quantitative

Question # 8-------------A cyclist got his tyre punctured when he had covered two third of the distance to be covered. Finishing on foot, He takes twice the time taken before to reach the destination. How fast does he ride than walking?

4. Aperson says that his son is 5 times as old as his daughter and his wife is 5 times older than his son and he is twice the age of his wife . The sum total of all the ages equals the age of the grand mother who celebrated her 81st birthday today. How old was his son?

5. A bargainhunter bought some plates for $ 1. 30 from asale on saturday, where price 2 was marked off at each article . On monday she went to return them at regular prices, and bought some cups and saucers from that much amount of money only. the normal price of plate were equal to the price of 'one cup and one saucer'. In total she bought 16 items more than previous. saucers were only of 3 cents hence she brought 10 saucers more tahn the cups, How many cups and saucers she bought and at what price?

6. Two marbel shooters. One gained 20 marbles more than other , but to play off he lost two-third of his marbles leaving the other with the 4 times more of the marbles the one had at the end of the play. how many marbles did each had in the beggining?

test18--------1. A boss told his employer that he spend one sixth of his age as industrialist and one twelelth of his age as spokesman, one seventh and five years as politician and matrimony. by the time his son arvind was born. now he was elected as governor four years ago in US?what was the BOss current Age?

Q4. A lady buys some plates in 130$ with 2$ off every item. she then returns the plates for the same amount next day to exchange them for some cups & saucers. Each saucer costs only 3$ each and the no. of saucers is 10 more than cups. Altogether she takes 16 more items than before. If she had to buy only cups, how many of them she could have been taken home on the first day?

Question # 1-------------A Couple decided to travel a north country side . so they decide to travel a minimum amount on car the first day and the second and sbsequent day a distance of 20 miles. If they travel a total amount of 1080 miles. Find he distance travlled on the 4th day and the 9 day.

Question # 2-------------A card boarb of 34 * 14 has to be attached to a wooden box and a total of 35 pins are to be used on the each side of the cardbox. Find the total number of pins used .

Question # 3

Page 89: Quantitative

-------------During a Pizza buffet where A eats more times 2. 4 than B, and B eats 6 times less than C. find the leat number of times all the three has to eat.

Question # 4-------------Last Year mu cousin came to my place and we played a game where the loosing one has to give one choclate to the person who won the game . At the end of the vacation, i. e the day my cousin was leaving she counted number of games that i won an she won. At last she gave me a total of 8 choclates even though she won about 12 games. Find the mumber of games that we played.

2. xyz +yxz = zxy find x, y, z this question u will get in fresher world paper

2. there are 770 peanuts. when susie takes 4 lily takes 3. when susie take 6 julie takes 7. how can u divide the chestnuts in proportion mentioned. THIS IS FROM SHANKUTALA DEVI. KINDLY BEWARE OF THE NAMES IN THE ANSWER SHEET BE ATTENTIVE

3. a hare and tortoise in a circular track of circumference 10 yards. the tortiose starts first and walks abt 1/8 of distance. when hare starts tortoise travels of 1/6 of distance. to win race at what spped hare neds to run. 4. some pigs and ducks are there. they have altogeher 60 eyes and 86 feet. calculate the no of pigs and ducks. AGAIN FROM SHANKUTALA DEVI

5. this giving details like. grocery store closes on monday. shoe store closes on thursday, bank will be open on monday, wednesday friday. boutique store closes on tuesday. on sunday all stores wil be closed. four people A B C D goes out for shopping on a day.

according to statements of four people A B C D u have to tell to which stores they are going.

6. three games played A vs B. B Vs C. A Vs C results of games are as follows. total six games goals for=goals against.

find the score of the teams in each games played.

1. A alone can do a work in 6 days B alone can do in 8 days with help of c they finished the work in 3 days. If the agreed sum is 640 what is the share of c. (refer r. s agarwal) 2. A drives a car four times a lap 10, 20 30, 60 kmph what is the average speed. 3. speed of boat in still water 10 km, if speed up stream is 24 km and speed down stream is 16 what is speed of the river. 4. If grand father age is sum all the three grand childern whos age r in equal intervalwhat is the age of the grand father? (refer r. s agarwal) test 37--------

Page 90: Quantitative

1) There is a five digit number. The fifth digit is one fourth of the third digit and one half of the fourth digit. Third digit is one half of the first digit. second digit is 5 more than the fifth digit. What is that 5 digit no.?

test 41---------

1. John had decided to divide his RS. 1000/- for his four children according to their ages. The elder child should be a RS. 20/- extra for each than his younger child . What will be the share of Mahesh who is the youngest?[3 marks]2. MR. lee told to MRS. Sheela that her cabbage patch was larger than last year. MRS. Sheela answered that “Yes, it was 231 more cabbages”, How many rise was it than the previous year?[3marks]

3. One side of the measuring arm was longer than the other side . If 3 pyramid width was placed in longer side it is equal to 2 cube width in the shorter arm. If 8 pyramid width was placed in longer arm then it is equal to 6 cube width. Let 1 pyramid width is equal to 10kg. What is the wgt of cube width?

4. A, B, C, D are four girls who have 1, 2, 3, 4 apples with them respectively. If E have apples equal to his sister, F have twice the apples as his sister, G have thrice the apples than his sister and I have four times the apples than my sister. All together we have 32 apples . A, B, C, D are the sisters of whom and whom?

7. A man wrote his “will’ accordingly the money was also shared between his car driver, 5 sons and 5 daughters. First he gave one rupee to his car driver, remaining 1/5 of the money is given to his 1st son . again he gave one to his car driver and remaining 1/5 of the money is given to his 2nd son, continuing the process until 5 sons are completed . After that he remaining money is divided between his 5 daughters. What was total amount of money?

8. 4 people identified a criminal and their statements are: # A: Eyes was blue , height was tall and he wore a hat & a vest. # B: Eyes was dark , height wa short and he wore a hat & a vest # C: Eyes was green , height was medium and he wore hat & a tie. # D: Eyes was grey , height was tall and he wore a rain coat and a hat. Everyone said only one correct identify other two was untrue . How can be the criminal identified?

10. A wall clock was slow by 10minutes . According to the wall clock, a table clock was 10 minutes ahead of it . According to the table clock an alarm clock was 5 minutes behind and according to the alarm clock wrst watch was 5 minutes fast. Atnoon all the clocks were adjusted . What will be the time at 6 P. M wrist watch?

test 42--------1). Ann, Brian, Cedric and Dave were Architect, Barber, Caretaker and Dentist notnecessarily in that order.

a)Atleast one but not all have the initials of occupation and name as same alphabet.

Page 91: Quantitative

b)The architect did not have 'r' in his namec)The dentist and barber had exactly one alphabet common in their names.

Tell the occupation of Ann and Cedric (4 marks)

2). John had a square farm. This year he had a larger square patch and hence he had 211 cabbages more than previous year. How many cabbages did he have the previous year. (4 marks)

3) An old man had 1000 rs. and he wanted to distribute them among his five sonsaccording to their ages such that a son received 20 rs more than the younger one. Watson was the youngest one. How much did watson receive. ?(4 marks)

5) An old man had some gold coins which he wanted to distribute among his 5 sonsand 5 daughters. He gives 1 gold coin to his driver and then 1/5 of remaining to his first son. Again he gives 1 coin to his driver and 1/5 of the remaining to his second son and so on till the last son. He does the same thing with his daughters. How many gold coins did the old man have initially. ?(6 marks)

7)A weighing pan was faulty as it had one arm longer than the other. 3 cubeweights on right side balanced 8 potweights on left side and 6 cubeweights on left side balanced 1 potweight on right side. One cube weight weighs one ounce then find weight of one potweight. (4 marks)

8) When the actual time passes 1hr the wall clock is 10 min behind it. when 1 hr is shown by wall clock, table clock shows 10 min ahead of it. when table clock shows 1 hr the alarm clock goes 5minbehind it, when alarm clock goes 1 hr wrist watch is 5 min ahead of it assuming that all clocks are correcrt with actual tinme at 12 noon what will be time shown by wrist watch at 5. 00 pm. (6 marks)

9) 32 marbles are to be distributed. Ann gets 1, Mary gets 2, Rose gets 3 and Lisagets 4. John Brown gets as much as his sister. Tim Smith gets 2 times as much ashis sister. Neil Johnson gets 3 times as much as his sister. Sam Paul gets 4 timesas much as his sister. Find the surnames of Ann, Mary, Rose and Lisa. (4 marks)

10) B & C initially speak English but when D joined spanish, they also took upspanish. The only common language between A, B and E is French. The only common language between C and E is Italian. Three people speak Portuguese. Most common language is spanish. One person knows all 5 languages. One person knows

test 43-------- Question # 3-------------8 Kigs and 14 Ligs can do 510 tors of work in10days. 13 Kigs and 6 Ligs can do 484 tors of work in 12 days. Then find work done by Kigs and Ligs individually in tors/hr?

Page 92: Quantitative

Question # 4-------------There is a 3 digited number. 3rd number is the square root of the 1st digit. 2nd digit is the sum of 1st and 3rd. And that number is divisible by 2, 3, 6, 7. What is that number?

Question # 5-------------A boy is playing a game. He took totally 55 blocks and kept like placing some x number on the ground, next one less than that above those blocks like that till the topmost one is one, like:

xx x

x x x x x x x x x x x x x x x x x x x x x x x x x x x x x x x x x x x x x x x x x xx x x x x x x x x x

(They didn’t gave this pictuire, but my explanation is not clear, that’s I gave u this picture) the question is how many blocks are there at the base level?

Question # 6-------------A cyclist is cycling in a circular path. he is at some point on the path, at that point 1/5th of the cyclists infront of him and 5/6th back to him gives the total number of clyclists participating in the race. What is the total number of cyclists?

Page 93: Quantitative

Question # 9-------------There are two dice painted on their sides. A and B are playing with those dice. A will win if the colour on the two faces of dice is same and b wins if color is different. First die is painted 5faces with red and 1 with blue. how is second die painted, if their chances of winning the game is even.

Question # 10-------------A manager declared like this in the meeting. Normally we give 6% interest on common stock, but the total income this time is Rs 400000 with 7 1/2 interest, we are giving only 5%interest on common stock. What is the worth of Comon stock?

test 44----------Question # 1-------------if alphabets are given position values according to their arrangement, for example A=1, and B=it's position to the power of previous alphabet's value.

i. e. , b= 2 1=2. c = 32, d= 4c.

Then find the value of (X-A)(X-b). . . . . (X-y)(X-z)?

Question # 2-------------

Two dice are rolled. If the score is calculated as a product of the number appeared. The score for the second role is six more than that of the 1st roal, the score for 3rd roll is 6 less than the 2nd roll, the score for 4th role is 11 more than the 3rd roll, the score on 5th role is five more than the 4th roll. Find the scores of 1, 2, 3, 4 rolls

Question # 3-------------A person traveled ¾ th of a distance by riding horse. He traveled remaining distance by walk. If he took twice the time to walk as he rode, how many times he rode faster than he walked?

Question # 4-------------There are diff color mushrooms

A) only and always purple is poisonous when red areB) green is safe for 6 monthsc) yellow is safe when purple are poisonousd) red is safe for 6 months.

Which coloured mushroom is safe at this time of year?Question # 5-------------

Page 94: Quantitative

The below are the statements of 3 persons. If the person's speak only two of the three statements correctly, what are their ages?A: A is 22. B is 1 yr more than A. C is 1 yr less than A.

B: I am not the youngest. There is 3 yrs difference between me and C. C is 25.

C: A is 23 B > A

Question # 6-------------A, B, C, D, E wore shirts of colours red, blue, green, yellow and orange respectively. They stood on a ladder. Yellow is not the colour at the tail end of the ladder. Yellow is between those who wore green and red. Green is between those who wore yellow and orange and who is behind blue. Which color is at the end of the ladder?

Question # 7-------------In a business, A invested 31/2 times more than B. They wanted to include C by taking an amount of Rs2500 from C. How should tha amount be divided among A and B such that three of them get the same interest?

Question # 8-------------One of the arms of a balance is longer than the other. If 1kg is kept on the left pan, it is equal to 8 water melons. If the 1kg is kept on a a right pan, it is equal to 2 water melons on right. If the weight of all the watermelons are sam, what is the weight of the water melon?

Question # 9-------------A person has a son. His son's age is fivew times his daughter's age. His wife's age is 5 times his son's age. His age is twice his wife's age. The total of all this ages is equal to his son's grandma's age who celebrated her 81st birthday this year. What is his son's age?

Question # 10-------------Tom has given some stamps to A. He asked A about them. A said " They were great. B got 3 more than ½ of what she would have got if I kept 3 more than ½ of what she got.

Page 95: Quantitative

Tom asked A "How much did u keep?" A answered "two more than what I gave B". Find the total no of stamps Tom gave A.?

test 48--------1. Analytical question. 6 poets, liking and disliking of poetry E. g. : All who like the poetry of Frost do not like the poetry of Coolidge. Some who like the poetry of Dunne like the poetry of Earl. 4 related questions.

2. Analytical question. 4 people, 4 products. E. g. : Vickie will not get a tie unless George buys a scarf. Remmie will not get a hat unless Jack buys gloves. Around 12 statements like this. Who will buy what?

3. 13 kigs and 6 libs can produce 510 tors in 10 hrs, 8 kigs and 14 libs can produce 484 tors in 12 hrs, find the rate of production of tors for kigs and libs. Express the answer in tors/hr.

4. Find the 3 digit no. whose last digit is the square root of the first digit and second digit is the sum of the other two digits.

5. Meera was playing with her brother using 55 blocks. She gets bored playing and starts arranging the blocks such that the no. of blocks in each row is one less than that in the lower row. Find how many were there in the bottom most row?

6. Rahul took part in a cycling game with many others in a circular closed circuit. After pedaling for several minutes, he found that 1/5th of the cyclists ahead of him and 5/6th of the cyclists behind him together formed the total no. of participants. How many were participating in the race?

9. Two people are playing with a pair of dies. Instead of numbers, the dies have different colors on their sides. The first person wins if the same color appearson both the dies and the second person wins if the colors are different. The odds of their winning are equal. If the first dice has 5 red sides and 1 blue side, find the color(s) on the second one.

10. A company's director said during the board meeting: " The company's income from roads will be sufficient to pay 6% of the entire stock issue, but since we are paying 7. 5% interest on the preferred stock of Rs. 4, 000, 000 we are able to pay only 5% of the common stock". Find the value of the common stock. 4. Boys are allowed to watch football at C.V.Raman auditorium subjected to conditions. The boy over age 16 can wear overcoat No boy over age 15 can wear capTo watch the football either he has to wear overcoat or cap or both A boy with an umberella or above 16 or both cannot wear sweater. Boys must either not watch football or wear sweater. What is the appearence of the boy who is watching football. 5. A bird keeper has got P pigeons, M mynas and S sparrows. The keeper goes for lunch leaving his assistant to watch the birds. Suppose p=10, m=5, s=8 when the bird keeper comes back, the assistant informs the x birds have escaped. The bird keeper exclaims: "Oh no! All my sparrows are gone."How many birds flew away? When the bird keeper comes back, the assistant told him that x birds have escaped. The

Page 96: Quantitative

keeper realised that atleast 2 sparrows have escaped. What is minimum no of birds that can escape? 7. There are N coins on a table and there are two players A & B. You can take 1 or 2 coins at a time. The person who takes the last coin is the loser. A always starts first. If N=7

(a) A can always win by taking two coins in his first chanse (b) B can win only if A takes two coins in his first chance. (c) B can always win by proper play (d) none of the above

3. B can win by proper play if N is equal to (a) 25 (b)26 (c) 32 (d) 41 (e) none

4. if N<4, can A win by proper play always?(a) Yes (b) No 11. Mrs. Jones, Mrs. Davis and Mrs. Smith are Friends. Their first names are Mary, Helen and Dorothy though not in order. All three of them go on shopping, where Marry spends twice of what Helen spent and Helen spends thrice of what Dorothy Spends. Mrs. Davis spends 3.85 more than Mrs. Smith. Identify the Families of these three women.  13. A Person gets his old book binded. But he finds that the page numbers are cut off. So he starts numbering the pages. During the process he finds that 3 is encountered 61 times. Can u tell how many pages were there in the book?  14. A child questions his father, "What is your father's age", to which the father answers "He was aged X in year X2". If the context is 20th Century, What is his Date of Birth?  15. A Clock loses exactly 24 minutes per hour. The Clock is now showing 3.00 A.M. If the clock was corrected a midnight and it stooped 1 hour ago, What is the correct time now?

6. A boy will cycle his way to his grandfather's house. On the first day he cycles 1/2 of the distance. On the second day he cycles 1/2 of he remaining distance. On the third day he cycles three quarter of the remaining. On the fourth day he cycles 10 miles. On the fourth day he cycles 3/4 of the remaining distance and on the final day he cycles the remaining 5 miles. What is total distance he traveled?  

Page 97: Quantitative

17. A rich man wanted to distribute his collection of gold coins among his ten children comprising of 5 sons and 5 daughters. He also wanted to give some coins to his car driver. He proceeds on the following basis. The first coin went to the car driver and then 1/5th of the remaining coins to his first son. He then gave another coin to the car driver and distributed 1/5th of the remaining coins to his second son. This procedure went on till all the five sons received their share. Once all the sons received their shares, he distributed the remaining coins equally among his daughters. Can u find how many gold coins the rich man had?  18. Ava, Fanny, Eartha, Cynthia, Isabelle and Rick, Smith, Vic, Steve, Willy are 5 Couples though not in order. Each were married on the different day of the week from Monday to Friday.Ava was married on Monday but not to Willy Steve was married on Thursday and Rick on Friday (but not to Isabelle) Fanny married Vic on the day after Eartha was married. Identify the Couples. 30. Sum of two consecutive nos is 55, larger one is?  31. A person goes 4/5 of his usual speed reaches 10min lateto his destinaton, time taken?  32. 80% pass in english, 70%pass in maths , 10%fail in both , 144 pass inboth . How many all appeared to the test? 33. To get a parabola if you cut a section of? 34. Bird is flying 120km/hr b/w B to R. two trians at B to R at 60 kmphThe distance trvelled by the bird before it is killed.Ans.120 15.Ten coins are distr. Among 4 people P, Q, R, S such that one of them gets a coin, another gets 2 coins,3rd gets 3 coins, and 4th gets 4 coins. It is known that Q gets more coins than P, and S gets fewer coins than R a. If the no. of coins distr. To Q is twice the no. distr. to P then which one of the following. is necessarily true?i. R gets even no. of coinsii.  R gets odd no. of coinsiii. S gets even no. of coinsiv. S gets odd no. of coinsb. If R gets at least two more coins than S which one of the following is necessarily true?i. Q gets at least 2 more coins than Sii. Q gets more coins than Piii. P gets more coins than Siv. P and Q together get at least five coinsc. If Q gets fewer coins than R, then which one of the following is not necessarily true?i.  P and Q together get at least 4 coinsii. Q and S together get at least 4 coinsiii.R and S together get at least 5 coinsiv.P and R together get at least 5 coins 

1. In 1978, a kg of paper was sold at Rs25/-. I f the paper rate increases at 1.5% more than inflation rate which is of 6.5% a year, then what wil be the cost of a kg of paper

Page 98: Quantitative

after 2 years?

a)29.12 (b) 29.72 (c) 30.12 (d) 32.65 (e) none of these

8. In a class , except 18 all are above 50 years. 15 are below 50 years of age. how many people are there

a) 30 b) 33 c) 36 d) none of these.

9. A square plot of some size , at four corners equal squares of some size are cut and is formed as open box. If this open box carries 128ml of oil. What is the size of the plate i.e. side

a.17 b.14 c.13

1) speed of boat in still water is 10kmph..if it travels 24km downstream,16km upstream in the same amount of time,what is the speed of the stream?(a)3kmph (b)3.5kmph (c)2kmph (d)...

6) if the area of a square has increased by 69%,by what % has its side increased?

7) in a class the average age is 16yrs.if the teacher who is 40 yrs of age is also included, the average becomes 17yrs,how many students were there?

10)if the 1rst day of 1999 is a sunday,what is the last day?(a)Sunday (b)Monday (c).... (d)cannot be determined.

32)which of the following uses the minimum length of cable?(a)ring (b)star (c)mesh (d)bus (e)all of the above

1. In 1978, a kg of paper was sold at Rs25/-. If the paper rate increases at 1.5% more than the inflation rate which is 6.5% a year, then what wil be the cost of a kg of paper after 2 years?

(a) 29.12 (b) 29.72 (c) 30.12 (d) 32.65 (e) none of these

4. A person has Rs 100/- in his pocket, he can as 25 pencils or 15 books. He kept 15% of the money for travelling expenses and purchased 5 pencils. So how many books he can purchase with the remaining money.

Page 99: Quantitative

6. The values of shares (in Rs).of A, B and C from January to June are as follows.

Month A B C January 30 60 80 February 35 65 85 March 45 75 65 April 40 75 82 May 55 75 85 June 50 75 80

i) During this period which share has undergone maximium fluctuation?ii) In which month it is possible to buy B and C selling A?iii) In which month the share values are very low?iv) By purchasing one share of A and 4 each of B and C in the beginning of the period, when shoudl these be sold to get maximum profit?

6. The values of shares (in Rs).of A, B and C from January to June are as follows.

Month A B C January 30 60 80 February 35 65 85 March 45 75 65 April 40 75 82 May 55 75 85 June 50 75 80

i) During this period which share has undergone maximium fluctuation?ii) In which month it is possible to buy B and C selling A?iii) In which month the share values are very low?iv) By purchasing one share of A and 4 each of B and C in the beginning of the period, when shoudl these be sold to get maximum profit?

7. In a computer institute 9 languages can be taught. The module is of 6 months duration and of the six languages only one can be taught each month. In addition to that BASIC is always taught and should be in first month itself

WORD PERFECT is to be taught in the preceeding week of WORD STAR.  FORTRAN can not be taught until COBAL is taught prior to that  BINO, FIFO can never be taught in single module  languages are BASIC, WORD STAR, WORD PERFECT, FORTRAN, COBAL, BINO, FIFO, LOTUS, Ci) If word star is in 3rd month , what could be in 6th month.ii) If COBAL is in the 2nd month and BINO in 6th month. FORTRAN will be taught in which month.9. A square plate of some size is cut at four corners. Equal squares of the same size are cut and is formed as open box.If this open box carries 128 ml of oil. What is the size of the side of the plate?

(a) 17 (b) 14 (c) 13 (d) None of these

Page 100: Quantitative

10. In a square, all the mid points are joined. The inner square is shaded. If the area of the square is A, what is the area of the shaded area?

1)Find the reminder when 333666777888999 divided by 3 or 9 or 11 ? (2 Questions like this) 6) 3 red and 4 blue balls are in a basket. A member of PPTeam is drawing balls from the basket. What is the probablity of getting the 3 red balls simultaneously? 7) Let ax2 + bx + c = 0If the sum of the equal roots is equal to the product of the same roots.Then which of the following hold true(a) a + b = 0 (b) a = 0 (c) c = 0 (d) b + c = 0 20)In a class total 34 students, 16 are have a brother, 15 are have sisters, 9 students don't have either brothers or sisters. Find the number of students having both brother and sisters. 22)The ratio of the length : breadth : height of a cuboid is 5 : 4: 3, and the volume is 7500. What will be its surface area? 24) If 38473ABC(This was not the exact no given) is divisible by 75 what is ABC in the same order.

26) In the series 0, 3, 8, 15,__ What is the next number?

28) Let ax2 + bx + c = 0If the sum of the equal roots is equal to the product of the same roots. Then which of the following hold true(a) a + b = 0 (b) a = 0 (c) c = 0 (d) a + c = 0

42). A persons salary iis decreased by steps of 20%, 15% and 10%. What will be the percentage decrease, if the salary is decreased in a single shot?

43). 3 persons started placementpapers with a capital of Rs.3000 . B invest Rs.600 less than A, C invest Rs.300 less than B. Then what is the share amount of B in a profit of Rs.886 ?

44). 178^2 -22^2(USE (A+B)(A-B))

45). f(x) = x! g(x)=x^10 h(x)= 10^x for large values of x which is greater?

46). f(x) = 1/(1-x) find(f(f(f(x)))

47). x+(1/x) = 2.5 find x^2 + (1/x^2)

48). 2^(2^n) = 256 find 2^(2n)

49). x=0 y=0 6x+7y=42 find position(lies inside/out/on) of (4,5) in the triangle

Page 101: Quantitative

50). 22 march 2004 is Sunday which year previously did 22jun became Sunday

Walk/Swim thro Coding

7. Find the reminder when 333666777888999 divided by 3 or 9 or 11 ?

8. Which is the biggest perfect square amongst the following15129, 12348, 23716, 20736

9. The greatest area of the following(a) The radius of circle is 4(b) The square of diagonal is 4(c) The square of side is 4

10. The area of the maximum size of the circle described from the 10 square inch square?

13. 3 red and 4 blue balls are in a basket. A member of PP Team is drawing balls from the basket. What is the probability of getting the 3 red balls simultaneously?

14. Let ax2 + bx + c = 0If the sum of the equal roots is equal to the product of the same roots. Then which of the following hold true(a) a + b = 0 (b) a = 0 (c) c = 0 (d) a + c = 0

20. A cylinder is inserted in a sphere d/h = 2/3. Find the surface area of the cylinder?

36. 6 women can do 75 units of work in 8 days by working 5hrs/day. In how many days can 4 women do 30 units of work by working 8hrs/day?

37. A persons salary iis decreased by steps of 20%, 15% and 10%. What will be the percentage decrease, if the salary is decreased in a single shot?

38. The ratio of the length : breadth : height of a cuboid is 5 : 4: 3, and the volume is 7500. What will be its surface area?

39. If the circumference of a circle is 100 units, Then what will the length of the arc described by an angle of 20 degree?40. 3 persons started placement papers with a capital of Rs.3000 . B invest Rs.600 less than A, C invest Rs.300 less than B. Then what is the share amount of B in a profit of Rs.886?

Find the values for the following Problem: f(X)= 2X-1 + f(X-1) if X is not equal to zero and if f(X=0)=09. Value of f(5) a)15 b)24 c)22 d)None10. Value of f(f(2))11. Value of f(16)- f(15)12. Value of f(16)+f(15)-48013. If f(f(X))=81 then the value of X=?

Page 102: Quantitative

14. If f(X)=4f(X-1) then the value of X=?15. If f(X)= f(X-1)+f(X-2) for X>1 then X=?16. If f(X)-f(X-1)=f(X-8) for X>5 then X=?  The following is a group of questions is based on a passage or a set of conditions for each question.Select the best answer choice given.

(i). If it is fobidden by law if the object of agreement is the doing of an act,that is forbidden by law the agreement is void.

(ii). If it is of the nature that,it would defeat the provision of any law is the agreement is void.if the object of agreement is such that thing got directly forbidden by law it would defeat the provision of statuary law.

(iii). If the object of agreement is fraddulent it is void.

(iv). An object of agreement is void if it involves or implies to the personnal property of another.

(v). An object of agreement is void where the constant regards as ignored.

(vi). An object of agreement is void where the constant regards is as opposed to public policy.

Q. An algorithm follws a six step process za,zb,zc,zd,ze,zf, it is governed by the following

(i) zd should follow ze(ii) the first may be za,zd or zf

(iii) zb and zc have to be performed after zd(iv) zc must be immediately after zb

Q. If za is the first set zd must be

a) 3rd b) 5th c) 2nd d) 4th

Q. If zb must follow za then za can be

a) third or fourth b) first or second c) can not be third d) fouth or fifth e) none

Page 103: Quantitative

Q. If ze is third term the number of different operations possible are

The following questions are based on the given statements Ravi plants six seperate saplings -- x,y,z,w,u,v in rows no 1 to 6, according to the following conditionsHe must plant x before y and uHe must plant y and wThe third has to be z

Q. Which of the following is acceptable

a) xuywzv b) xvzyuw c) zuyxwv d) zvxuwy e) wyzuvx

Q. Which of the following is true

a) z before v b) z before x c) w before u d) y before u e) x before w

Q. If he plants v first, then which can be planted second

a) x b) y c) z d) w e) u

Q. Which of the following describes a correct combination of sapling and row?

a) x,3 b) y,6 c) z,1 d) w,2 e) u,6

Q. If he plants b 6th which would be planted first and second

a) x and w b) x and y c)y and x d)w and z e) w and u

Q. If he plants w before u and after v he should plant w at

a) first b) second c) fourth d) fifth e) sixth

Q. At a certain moment a watch shows 2 min lag although it is running fast.If it showed a 3 min lag at that moment, but also gains by 1/2 min more a day than its current speed it would show the true time one day sooner than it usually does. How many mins does the watch gain per day.

a).2 b).5 c).6 d).4 e).75

Q. In 400m race A gives B a start of 7 sec and beats him by 24 sec. In another race A beats B by 10 sec.the speeds are in the ratio

a)8:7 b)7:6 c)10:8 d)6:8 e)12:10

To get AAABBD from BBBAAA what number should be applied:-a) 25 b) 34 c) 25 & 34 d) none

2. Similar question on functions.3. [Based on the function in the first question] For the function f( 4 a b c ) the number of terms is...?

Page 104: Quantitative

Hint f(4 a b c ) = f( 3 a c b ) + f( 1 a b c ) + f( 3 b a c ) etc.

4. What is the value of the function f ( 5 a b c ) = ?

1. r = number of flags; n = number of poles;Any number of flags can be accommodated on any single pole.

1) r=5,n=5 The no. of ways the flags can be arranged ?

Questions 2-5 are based on the above pattern

6. r = 5 n = 3 . If first pole has 2 flags, third pole has 1 flag    How many ways can the remaining be arranged?

SECTION -III:

In the following questions a 'word' is given which may not have any meaning.Find differnet possible words or palindromes for the word as per the question.

For the following find no of distinct words that can be formed.

17. TYGHHTT

a).420 b)1540 c)840 d)None

18. TYGHHTY

19. TYGHHTT

20. TYGHHTT

21. TYGHASD

22. TYGHHTY

Find the number of possible palindromes for following

23. TYGHHTY

24. TYHHHTYH.

[1]Two train travel opposite to each other from one station to another. First train leaves the station every one hour and second train leaves the station every one and half hour. If first train reaches to second station to second station after 5 hours. Then the no. of train that will cross the first train will be: marks(3)

5.A ball was dropped from a height of 180 feet. If it is to rebound 10%. how much distance it will to cover. 220

Page 105: Quantitative

1.Sabrina's boss states that she will increase Sabrina's salary from $12,000 to $14,000 per year if she enrolls in business courses at a local community college. What percent increase in salary will result from Sabrina taking the business courses? A. 15% B. 16.7% C. 17.2% D. 85% E. 117% 2. How long will Lucy have to wait before her $2,500 invested at 6% earns $600 in simple interest? A. 2 years B. 3 years C. 4 years D. 5 years E. 6 years 3. At a company fish fry, ½ in attendance are employees. Employees' spouses are 1/3 of the attendance. What is the percentage of the people in attendance who are not employees or employee spouses? A. 10.5% B. 16.7% C. 25% D. 32.3% E. 38% 4. How many years does Steven need to invest his $3,000 at 7% to earn $210 in simple interest? A. 1 year B. 2 years C. 3 years D. 4 years E. 5 years 5. If y = 3, then y3(y3-y)= A. 300 B. 459 C. 648 D. 999 E. 1099 6. Two cyclists start biking from a trail's start 3 hours apart. The second cyclist travels at 10 miles per hour and starts 3 hours after the first cyclist who is traveling at 6 miles per hour. How much time will pass before the second cyclist catches up with the first from the time the second cyclist started biking? A. 2 hours B. 4 ½ hours C. 5 ¾ hours D. 6 hours E. 7 ½ hours 7. John is traveling to a meeting that is 28 miles away. He needs to be there in 30 minutes. How fast does he need to go to make it to the meeting on time? A. 25 mph B. 37 mph C. 41 mph D. 49 mph E. 56 mph 8. If two planes leave the same airport at 1:00 PM, how many miles apart will they be at 3:00 PM if one travels directly north at 150 mph and the other travels directly west at 200 mph? A. 50 miles B. 100 miles C. 500 miles D. 700 miles E. 1,000 miles 9. Sally has three pieces of material. The first piece is 1 yd. 2 ft. 6 in. long, the second piece is 2 yd. 1 ft. 5 in long, and the third piece is 4 yd. 2ft. 8in long. How much material does Sally have? A. 7 yd. 1 ft. 8 in. B. 8 yd. 4 ft. 4 in. C. 8 yd. 11 in. D. 9 yd. 7 in. E. 10 yd. 10. Jim has 5 pieces of string. He needs to choose the piece that will be able to go around his 36-inch waist. His belt broke, and his pants are falling down. The piece needs to be at least 4 inches longer than his waist so he can tie a knot in it, but it cannot be more that 6 inches longer so that the ends will not show from under his shirt. Which of the following pieces of string will work the best? A. 3 4/5 feet B. 3 2/3 feet C. 3 3/8 feet D. 3 1/4 feet E. 2 1/2 feet 11. The last week of a month a car dealership sold 12 cars. A new sales promotion came out the first week of the next month and the sold 19 cars that week. What was the percent increase in sales from the last week of the previous month compared to the first week of the next month? A. 58% B. 119% C. 158% D. 175% E. 200%

12. During a 5-day festival, the number of visitors tripled each day. If the festival opened on a Thursday with 345 visitors, what was the attendance on that Sunday? A. 345 B. 1,035 C. 1,725 D. 3,105 E. 9,315 14. Michelle wants to expand her flowerbed by 2 ft. on each side. What will the new area of the flowerbed be?

Page 106: Quantitative

A. 2 LW B. 2 (L+W) C. 2L +2W D. (L+2) (W+2) E. LW/2 15. Which of the following fractions are correctly placed from the least in value to the greatest in value? A. 1/4, 17/25, 3/4, 11/16 B. 17/25, 1/4, 11/16, 3/4 C. 1/4, 17/25, 11/16, 3/4 D. 1/4, 17/25, 3/4, 11/16 E. 3/4, 17/25, 11/16, 1/4 16. A mattress store sells their stock for 15% off of retail. If someone pays cash, they take an additional 10% off of the discounted price. If a mattress's retail price is $750, what is the price after the store discount and the cash discount? A. $550.75 B. $562.50 C. $573.75 D. $637.50 E. $675.00 17. Which of the following letters represents the vertex in the following picture?

A. D and E B. E and H C. F and G D. G only E. H only 18. Art realized that he had 2 more quarters than he had originally thought in his pocket. If all of the change in his pocket is quarters and it totals to $8.75, how many quarters did he originally think were in his pocket? A. 27 B. 29 C. 31 D. 33 E. 35 19. Which of the following statements is false? A. In the fraction ½, one is the numerator. B. When 4.89 is rounded to the ones place, the answer is 5. C. Ten thousandths place is located 5 places to the right of the decimal D. 7/6 is described as an improper fraction.

E. 331/3 % is equivalent to 20. You need 4/5 cups of water for a recipe. You accidentally put 1/3 cups into the mixing bowl with the dry ingredients. How much more water in cups do you need to add? A. 1/3 cups B. 2/3 cups C. 1/15 cups D. 7/15 cups E. 7/16 cups

1. A family, planning a weekend trip, decides to spend not more than a total of 8 hours driving. By leaving early in the morning, they can average 40 miles per hour on the way to their destination. Due to the heavy Sunday traffic, they can average only 30 miles per hour on the return trip. What is the farthest distance from home they can plan to go?       (a) 120 miles or less (b) Between 120and 140 miles(c) 140 miles (d) Between 140 and 160 miles      (e) 160 miles or more

2. A car is filled with four and half gallons of fuel for a round trip. If the amount of fuel taken while going is 1/4 more than the amount taken for coming, what is the amount of fuel consumed while coming back?

(a) Less than 2 gallons (b) 2 gallons   (c) 2 1/2 gallons(d) 3 gallons (e) More than 3 gallons

Page 107: Quantitative

3. A 3-gallon mixture contains one part S and two parts R. In order to change it to a mixture containing 25% S, how much R should be added?

(a) 1/2 gallon (b) 2/3 gallon (c) 3/4 gallon(d) 1 gallon   (e) 1 1/2 gallon

4. A tree grows only 3/5 as fast as the one beside it. In four years the combined growth of the two trees is eight feet. How much does the shorter tree grow in two years? 

(a) Less than 2 feet  (b) 2feet                (c) 2 1/2 feet(d) 3 feet (e) more than 3 feet.

5. Wind flows at 160 miles in 330 minutes, for traveling 80 miles how much time does it require?      (a) 1 hour 30 minutes     (b) 1 hour 45 minutes     (c) 2 hours(d) 2 hours 45 minutes   (e) 3 hours

6. A stationary engine has enough fuel to run 12 hours when its tank is 4/5 full. How long will it run when the tank is 1/3 full?

(a) Less than 2 hours (b) 2 hours         (c) 3 hours(d) 4 hours (e) 5 hours

7. If A is traveling at 72 km per hour on a highway. B is traveling at a speed of 25 meters per second on a highway. What is the difference in their speeds in meters per second?

(a) 1/2 m/sec (b) 1m/sec (c) 1 1/2 m/sec (d) 2 m/sec (e) 3 m/sec

8. A salesperson by mistake multiplied a number and got the answer as 3, instead of dividing the number by 3. What is the answer he should have actually got?       (a) 0 (b) 1/3 (c) 1       (d) 2 (e) 3

9. If the length of a rectangle is increased by 30% and the width is decreased by 20%, then the area is increased by...

(a) 10% (b) 5%   (c) 4%     (d) 20% (e) 25%

10. In the class of 40 students, 30 speak Hindi and 20 speak English. What is the lowest possible number of students who speak both the languages?

(a) 5       (b) 20     (c) 15      (d) 10 (e) 30

11. The most economical prices among the following prices is:

(a) 10 kilo for Rs.160              (b) 2 kilo for Rs.30     (c) 4 kilo for Rs.70(d) 20 kilo for Rs.340               (e) 8 kilo for Rs.130

Page 108: Quantitative

12. A truck contains 150 small packages, some weighing 1 kg each and some weighing 2 kg each. how many packages weighing 2 kg each are in the truck if the total weight of all the packages is 264 kg?

(a) 36 (b) 52 (c) 88  (d) 124 (e) 114

13. A man was arrested for exceeding the speed limit by 10 miles an hour. A second man was charged with exceeding the same limit by twice as much. The second man was driving 35 miles per hour. What was the speed limit?

(a) 10 miles per hour                (b) 15 miles per hour (c) 20 miles per hour (d) 25 miles per hour                (e) 30 miles per hour

14. One year ago Pandit was three times his sister's age. Next year he will be only twice her age. How old will Pandit be after five years?

(a) 8  (b) 12   (c) 11 (d) 13 (e) 15

5.WHICH WILL GIVE GOOD STANDARD DEVIATION

1. (7,0,-7,0,7) 2. (7,-7,7,-7,7) 3. (1,0,-1,0,1)

6. WHICH IS NOT A SIDE OF A RECTANGULAR1. (2,3,4) 2.(3,4,7) 3. (3,5,9)7.WHICH SHAPE WILL BE OBTAINED BY USING THESE VALUES OF X ,Y X Y

0 0.0000110 1.02100 1.721000 3.009999 4.72

8. WHICH EQUATION THAT BEST SUITS THIS CURVE A LINE CUTS X AT -1 WHEN Y=0 AND X=0 WHEN Y=3 AND GOES UPWARD

YX

9. A MAN , WOMAN AND A BOY JOINDLY DID A JOB IN 6 DAYS. A MAN ALONE FINISHES IN 10 DAYS, A WOMEN ALONE FINISH IN 24 DAYS. THEN HOW MANY DAYS THE BOY CAN TAKE TO FINISH?

10. FOR TEMPERATURE A FUNCTION IS GIVEN ACCORDING TO TIME : ((t**2)/6) + 4t +12 WHAT IS THE TEMPERATURE RISE OR FALL BETWEEN 4.AM TO 9 AM

19. G (0) =1 G (1)= -1 IF G (N)=2* (G (N-1))- 3(G (N-2)) THEN WHAT IS THE VALUE OF G (4)? 21. TIME DEGREE

2 7 ° 6' 43.15" 4 5° 31' 4.3"

THEN WHAT WILL BE THE DEGREE WHEN TIME IS 3 O CLOCK

Page 109: Quantitative

22. THREE COMPANIES WORKING INDEPENDENTLY AND GET SAVINGS 10%, 20%, 25%. IF THE COMPANIES WORK JUST OPPOSITE THEN WHAT WILL BE THE NET SAVING?

23. WHICH ONE WILL BE THE EXACT POWER OF 3

(i) 2768 (ii)2678 (III) 2187

28. WHICH IS THE PERFECT ONE AMONG THE 4

1. 2x +3y=4 2. x + y= -1 3. Y=2x+ 3

4. Year 95 96 97 98 99 Members 100 1 70 10 50

5. In Madras, temperature at noon varies according to -t^2/2 + 8t + 3 (READ as: -t square /2 +...), where t is elapsed time. Find how much temp. More or less in 4pm to 9pm. (May be we can solve it by Definite Integration. Check any way)

6. A man, a woman, and a child can do a piece of work in 6 days. Man only can do it in 24 days. Woman can do it in 16 days and in how many days child can do the same work? (Numbers are not correct. Problem model is important)

10. The size of a program is N. And the memory occupied by the program is given by M = square root of 100N. If the size of the program is increased by 1% then how much memory now occupied?

12. The size of the bucket is N KB. The bucket fills at the rate of 0.1 KB per millisecond. A programmer sends a program to receiver. There it waits for 10 Milliseconds. And response will be back to programmer in 20 milliseconds. Based on above information one question is there.

13. A power unit is there by the bank of the river 750 mts. A cable is made from power unit to power a plant opposite to that of the river at 1500mts. The cost of the cable below water is Rs. 15/- per meter ansd cost of cable on the bank is Rs. 12/- per meter. Based on above information one question is there.

14. Match the following.i. brother - sister a. Part ofii. alsecian - dog b. Siblingiii. sentence - paragraph c. Type of

10. A train traveling at 72 kmph crosses a platform in 30 seconds and a man standing on the platform in 18 seconds. What is the length of the platform in meters?

(1) 240 meters (2) 360 meters (3) 420 meters (4) 600 meters

While solving simultaneous equations of the form x2 + y2 = 65 and x + y = 3, there is no real need to solve it theoretically, which turns out to be a laborious process. Consider perfect squares and by hit and trial see if two perfect squares add up to 65. x2 cannot be

Page 110: Quantitative

4, 9, 25, 36 as then y2 has to be 61, 56, 40, 29 which are not perfect squares. Thus the possible values for x and y are (±1, ±8) and (±4, ±7). The values that satisfy x + y = 3 are 7 and -4. The only assumption in the above is that x and y are rational. And this can be ascertained by looking at option choices or certain other data in the question or else one can simply take chances. If one does not get perfect squares adding up to 65, only then work theoretically.Consider this question:If x2 + y2 = 0.1 and |x – y| = 0.2, what is the value of |x| + |y|?1. 0.3 2. 0.4 3. 0.6 4. 0.2Consider the square of 0.1, 0.2, 0.3, 0.4 i.e. 0.01, 0.04, 0.09, 0.16 and it is obvious that 0.01 + 0.09 = 0.1. Thus x and y are 0.1 and 0.3. And these values also satisfy the second equation. Also the answer choices make it evident that one should not be thinking of multiple values or solutions. Thus answer has to be 0.4

• In simultaneous equations please remember that we need as many independent equations as the number of variables used ONLY when we have to find the unique value of all the variables. If we do not need the values of all the variables but only the value of an equation using the variables, we can possibly find the answer even when the number of equations is less than the number of variables. So do not be in a hurry to mark “Cannot be determined” as the answer in such case.Even after having as many equations as the number of variables, it is not necessary that we will be able to fid the values of all the variables. To be able t do so, all the equations should be independent. Check to see if by adding or subtracting two of the equations (including use of multiples if needed) we can derive one of the given equation. If we can do this, all the given equations are not independent.• Whenever the question deals with roots of any polynomial, axn + bxn-1 + cxn-2 + …… + px + q = 0, remember to check options such that sum of roots = -b/a and product of roots = ±q/a. Thus if a = 1 (as is usually the case or else you could make a = 1 by dividing the entire equation by a) and q is an integer and it is also known that the roots are also integers, then the roots have to be factors of q.• For a quadratic expression used in an inequality and that cannot be factorized, find the determinant and see if the quadratic expression takes only positive or negative values or takes all values:For a quadratic ax2 + bx + c with imaginary roots i.e. b2 – 4ac < 0,(ax2 + bx + c) is always positive if a is positive OR(ax2 + bx + c) is always negative if a is negative Whenever there appears any term of the type a3 + b3 + c3, do check for a + b + c being equal to zero. If a + b + c is indeed zero, then a3 + b3 + c3 = 3abc.•The series 1, 3, 6, 10, 15 should immediately be recognized as series of sum of first n natural numbers.•To form all natural numbers from 1 to N by adding any natural numbers, one would just need 1, 2, 4, 8, 16, 32, 64…..2n, where 2n is the largest power of 2 smaller than or equal to N. E.g. What is the minimum number of weights needed to be able to measure all natural numbers weights till 80, if weights can be kept only on one pan of the balance. One would need weights 1, 2, 4, 8, …64•To form all natural numbers from 1 to N by adding or subtracting any natural numbers, one would just need 1, 3, 9, 27, 81, …..3n. Be careful of the largest number needed in this case. E.g. What is the minimum number of weights needed to be able to measure all natural numbers weights till 80, if weights can be kept on both pans of the balance. One would need weights 1, 3, 9, 27, 81.

Page 111: Quantitative

•In questions of the type where certain flowers/sweets/points etc gets diminished and then increases and again diminishes and again increases……rather than forming an equation, see if one can work backwards if final quantity is given or else work with options. Also in such cases, if 1/3rd the objects are given away, work on the objects that are remaining i.e. 2/3rd to save time.If working with options, select options intelligently e.g. I pick 1/3rd of the chocolates in a bowl and then return 3, next I pick 1/5th of the chocolates and then return five, next ……What is the number of chocolates in the bowl initially? The initial number of chocolates has to be a multiple of 3. Also 2/3rd of the initial number of chocolates plus 3 should be divisible by 5. This should be enough to reduce the possible options to just about 2.•Remember that a2, b2, c2 or |a|, |b|, |c| are either zero or positive quantity. Thus solution to a2 + b2 + c2 = 0 or |a| + |b| + |c| = 0 is a = b = c =0 channelize the skill of effective reading into the area of comprehension. The question is, given a passage how do we tackle it?

1) it has 20 mixutre conatins mil and water in the ratio 3:5,replace 4 litres of mixture with 4 litres of water what is the final ratio of milk and water.

3) The equivalent compound ratio of 5:6::7:10::6:5 ( question of this type this is not exact question).

3) work can be done by 8 men and 10 women in 25 days, the same work can be done by 10 children and 5 women . in how many days 2 children and 3 men (similar to this) 

4) one man or two women or three boys can do a work in 44 days then one man, one women and one boy together can fininsh the same work in ---- dyas

5) (998-1)(998-2)(998-3)…………..(998-n)=------- when n>1000 ans is zero

6) in how many ways can a lock be opened if that lock has three digit number lock if i) the last digit is 9ii) and sum of the first two digits is less than or equal to the last digit.Numbers are from 0-9

7) if a man reduces the selling price of a fan from 400 to 380 his loss increases by 20% . cost price of fan is.

9) in pure milk if 20% replaced by water and in this again 20% is replaced by water and again 20% is replaced by water then what is the praportion of milk in that mixture

11) races and games ---- 2 questions from this chapter like (A beats B by 10 meters and B beats C by 15 metres the A beats C by ) 

3) F,G,H,J,K r some juniors and P,Q,R,S are some seniors...n there r 3 communities Cultural,Recreation n Ethics..n each contained 3 members each..n each one was a member of only one committee..

i)F,Q are in Cultural committee.

Page 112: Quantitative

ii)P is in recreation committee.iii)J wont be in a committee where H or Q are there.iv)G will be in recreation committe if S also works in recreation committee.v)there will be atleast one senior in each committee.vi)R wont be in a committee if S is there.

4)there was a community in which there were 1000 couples. In that 2/3rd of men who r taller r also heavier n 3/4th of the men who R heavier r also taller n there were 120 women who were both heavier n taller than men. So how many men r both taller n heavier than men?

7) X Y Z X Y ZA B + A B -C D E F B G Afind X,Y,Z,G

1. Find min value of fn:|-5-x| + |2-x|+|6-x|+10-x|; where x is an integer0/17/23/192. units digit in expansion os 2 raised to 51 is:

2/4/6/83. 2 men at same tym start walking towards each other from A n B 72kms apart. sp of A is 4kmph.Sp of  B is 2 kmph in 1st hr,2.5 in 2nd, 3 in rd. n so on…when will they meeti   in 7 hrs ii  at 35kms from A iii in 10 hrs iv midway5. 45 grinders brought @ 2215/-.transpot expense 2190/-.2760/- on octroi . Find SP/piece to make profit of 20%

2585/2225/2670/33256. in a 2 digit no unit’s place is halved and tens place is doubled. diff bet the nos is 37. digit in unit’s place is 2 more than tens place.

24/46/42/none7. if x-y + z = 19 , y + z =20 , x-z=3 , find d value of x+4y-5z

22/38/17/none8. Find approx value of 39.987/0.8102+1.987*18.02

72/56/86/449. If the ratio of prod of 3 diff comp’s A B & C is 4:7:5 and of overall prod last yr was 4lac tones and if each comp had an increase of 20% in prod level this yr what is the prod of Comp B this yr?

2.1L/22.1L/4.1L/none10. If 70% of a no. is subtracted from itself it reduces to 81.what is two fifth of that no.?

108/54/210/none11. If a certain sum of money at SI doubles itself in 5 yrs then what is d rate?

5%/20%/25%/14.8%12. If radius of cylinder and sphere r same and vol of sphere and cylinder r same what is d ratio betn the radius and height of the cylinderi.  R= H ii. R= (3/4)H iii. R = (4/3)H iv. R=2/3H13. Which one of the foll fractions is arranged in ascending orderi.  9/11,7/9,11/13,13/14 ii  7/8,9/11,11/13,13/14iii 9/11,11/13,7/8,13/14 iv none14. A is 4 yrs old and B is thrice A>when A is 12 yrs, how old will B be?

16/20/24/28

Page 113: Quantitative

15. Boat goes downstream from P to Q in 2hrs, upstream in 6hrs and if speed of stream was ½ of boat in still water. Find dist PQ

6/4/10/none16. Fresh Grapes contain 90% water by wt. Dried grapes contain 20% water by %age. What will b wt of dried grapes when we begin with 20 kg fresh grapes?

2kg / 2.4kg / 2.5kg /none17. How many 5 digit no. can b formed wit digits 1, 2, 3,4,5,6 which r divisible by 4 and digits not repeated

144 / 168 / 192 / none18. Asish was given Rs. 158 in denominations of Rs 1 each. He distributes these in diff bags, such that ne sum of money of denomination betn 1 and 158 can be given in bags. The min no. of such bags reqd

10 / 17 / 15 / none19.There is a rectangular Garden whose length and width are 60m X 20m.There is a walkway of uniform width around garden. Area of walkway is 516m^2. Find width of walkway

1/2/3/420. In a race from pt. X to pt Y and back, Jack averages 0 miles/hr to pt Y and 10 miles/hr back to pr X. Sandy averages 20 miles/hr in both directions. If Jack and Sandy start race at same tym, who’ll finish 1st

Jack/Sandy/they tie/Impossible to tell21. A man engaged a servant on a condn that he’ll pay Rs.90 and also give him a bag at the end of the yr. He served for 9 months and was given a turban and Rs.65. So the price of turban is

i. Rs 10 / 19 / 0 / 5522. Three wheels make 36, 24, 60 rev/min. Each has a black mark on it. It is aligned at the start of the qn. When does it align again for the first tym?

14/20/22/5 sec23. If 1= (3/4)(1+ (y/x) ) theni. x=3yii. x=y/3 iii. x=(2/3)y iv. None24. The sum of six consecutive odd nos. is 888. What is the average of the nos.?i. 147 ii. 148 iii. 149 iv. 14625. 1010/104*102=10?i. 8 ii. 6 iii. 4 iv. none

Five teams participated in Pepsi Cup. Each team played against each other. The top teams played finals. A win fetched 2 pts and a tie 1 point1) South Africa were in the finals2) India defeated SA but failed to reach the finals3) Australia lost only one match in the tournament4) The match between India and Sri Lanka was a tie5) The undefeated team in the league matches lost in the finals6) England was one of the best teams that did not qualify1. Who were the finalists?i. SA & India ii. Aus & SL iii. SA & SL iv. none1. Who won the finals?i. Aus ii. SL iii. SA iv. Can’t be determined1. How many matches did India Win?i. 0 ii. 1 iii. 2 iv. can’t be determined1. What was the outcome of the India England Match

Page 114: Quantitative

i. India won ii. England won iii. It was a tie iv. Can’t be determinedThese qns are based on situations given below:7 Uni crick players are to be honored at a special luncheon. The players will be seated on a dais along one side of a single rectangular table.A and G have to leave the luncheon early and must be seated at the extreme right end of table, which is closest to exit.B will receive Man of the Match and must be in the centre chairC and D who are bitter rivals for the position of Wicket keeper dislike one another and should be seated as far apart as possibleE and F are best friends and want to seat together.1. Which of the foll may not be seated at either end of the table?i. C ii. D iii. G iv. F1. Which of the foll pairs may not be seated together?i. E & A ii. B & D iii. C & F iv.  G & D

Page 115: Quantitative

A robot moves on a graph sheet with x-y axes. The robot is moved by feeding it with a sequence of instructions. The different instructions that can be used in moving it, and their meanings are:Instruction         MeaningGOTO(x,y)    move to pt with co-ord (x,y) no matter where u are currentlyWALKX(P)     move parallel to x-axis through a distance of p, in the +ve direction if p is +ve and in –ve if p is –veWALKY(P)     move parallel to y-axis through a distance of p, in the +ve direction if p is +ve and in –ve if p is –ve1. The robot reaches point (5,6) when a sequence of 3 instr. Is executed, the first of which is GOTO(x,y) , WALKY(2), WALKY(4). What are the values of x and y??

i. 2,4 ii. 0,0 iii. 3,2 iv. 2,31. The robot is initially at (x.y), x>0 and y<0. The min no. of Instructions needed to be executed to bring it to origin (0,0) if you are prohibited from using GOTO instr. Is:

i. 2ii. 1 iii. x + y iv. 0

Ten coins are distr. Among 4 people P, Q, R, S such that one of them gets a coin, another gets 2 coins,3rd gets 3 coins, and 4th gets 4 coins. It is known that Q gets more coins than P, and S gets fewer coins than R1. If the no. of coins distr. To Q is twice the no. distr. to P then which one of the foll. is necessarily true?i. R gets even no. of coins ii. R gets odd no. of coinsiii. S gets even no. of coins iv. S gets odd no. of coins1. If R gets at least two more coins than S which one of the foll is necessarily true?i. Q gets at least 2 more coins than S ii. Q gets more coins than Piii. P gets more coins than S iv. P and Q together get at least five coins1. If Q gets fewer coins than R, then which one of the foll. is not necessarily true?i. P and Q together get at least 4 coins ii. Q and S together get at least 4 coinsiii. R and S together get at least 5 coins iv. P and R together get at least 5 coins

Elle is 3 times older than Zaheer. Zaheer is ½ as old as Waheeda. Yogesh is elder than Zaheer.

1. What is sufficient to estimate Elle’s age?i. Zaheer is 10 yrs oldii. Yogesh and Waheeda are both older than Zaheer by the same no of yrs.iii. Both of the aboveiv. None of the above

1. Which one of the foll. statements can be inferred from the info abovei. Yogesh is elder than Waheedaii. Elle is older than Waheedaiii. Elle’s age may be less than that of Waheedaiv. None of  the above

Page 116: Quantitative

2.There is a 200 miles long tunnel. one train enters the tunnel at a speed of 200mph while the other trains enter the tunnel in the opposite direction at a speed of 1000 mph. A bee travels at a speed of   1500 mph enters the tunnel goes to and back until it reaches the train. What is the distance covered by the bee when the two train collides (the bee survives)21.If the probability of work done by three persons are 1/3, 2/5, 5/12. Then what is the probability that the work is completed.25.Krishna tosses a one-rupee coin and a rupee coin. He announces that one is head. But the result is not announced. What is the probability that the other coin is head?1. In a village in each family they give birth to children till they get a boy. IF girl child they try again. What is the ratio of boys to girls. 3. In 1000 wine bottles stack 10 are poisoned given 10 rats what is the minimum number of tries to find the poisoned one. Rat dies once it licks the poisoned wine.4. Write 1,3,6,4 using +,-,*,/ to get 24 (no repeat of numbers)8. A 5 litre jug contains 4 litres of a salt water solution that is 15 percent salt. If 1.5 litres of the solution spills out of the jug, and the jug is then filled to capacity with water,approximately what percent of the  resulting solution in the jug is salt?

9. Mohan earned twice as much as Deep. Yogesh earned rs.3/- more than half as much as deep.   If the amounts earned by Mohan, Deep, Yogesh are M,D,Y respectively, Which of the following is  the correct ordering of these amounts?   a) M < D < Y   b) M< Y < D   c) D< M < Y   d) It cann't be determined from the information given   e) D< Y < M10. Statistics indicate that men drivers are involved in more accidents than women drivers. Hence  it may be concluded that a) sufficiently information is not there to conclude anything b) Men are actually better drivers but drive more frequently c) Women Certainly drive more cautiously than Men d) Men chauvinists are wrong about women's abilties. e) Statistics sometimes present a wrong picture of things

1) Which  is greater  among  19/18 , 9/17 , 2/3 ...??2) Which  fraction  lies  b/w  5/2  and 3/5....??    etc....,,,

Ø  The total ages of three persons A, B, C is 90yrs, 10 yrs before ratio of A, B, C ages is 1:2:3.what is C’s age? Ø  A train 500m crosses a tunnel 0.5km between the time 10:05am and 10:10am what is the speed of train? Ø   M persons decide to buy a gift for D price now three persons leave the group so what is the cost of the gift now to remaining persons? Ø   A boat moves upstream with 12km/hr and downstream with 16 km/hr what is the speed of current and speed of boat in still water? Ø   There are 30 puppies in two windows a) 3/4th are male in left window b) 1/3rd are female in right window How many female puppies are there in total?

Page 117: Quantitative

Ø  Two types of alloys A and B have gold and copper in the ratio 7:5 and 6:5 if they are mixed in equal quantities ratio of gold and silver in this new mixture? Ø  A manufacturer sells goods to trader at 20% profit, trader sells to shopkeeper at 10% profit, shopkeeper to consumer at 10% profit if shopkeeper gets Rs.1, 45,2000 original cost of the goods? Ø  What annual debt will discharge a debt of Rs.7620 due in 2yrs at 2% per annum compound interest? end

1.there are 6 (p,q,r,s...)people.each one play one game ---tennis,football,hockey,cricket etc clues given like:shortest one plays hokey largest person plays ..tennis P,Q ARE NEITHER LARGEST NOR SHORTEST  R PLAYS FOOTBALLs is between t &r in height4.aa person sold two articles for 80 /- each.with 20% profit on one article and 20% loss on another article, what is the loss / prifit he will gain on both.5.five cities. city1 is west to city3.city4 is east to city 5._____etc. which is farthest on west side.2. A and B start with uniform velocities from opp. directions separated by a distance of 20000 km. A rests when B is on the move B rests when A is on the move. A’s speed is 40kmph.If A starts ,the least time and reaches the destination in 60 hours the least time B would take to reach the destination after A makes a start (B’s speed=50kmph)A. 100 B. 90 C. 80 D. 753.Two outlets of equal capacity can empty a tank of 30000 cu.m capacity in 24 hrs. If the tank is going to be filled by flood waters, which arrived at the rate of 300 cu.m/hr, what are the additional number of outlets of the same capacity that have to be opened so that the tank never overflows?A. 4 B. 5 C. 3 D.77. Chicory costs Rs.64/kg and coffee Rs .80/kg. A trader make as a profit of 25% by selling the mixture of chicory and coffee Rs.86/kg. In what proportion were chicory and coffee mixed A. 30% :70% B.70%:30% C.25%:75% D.75%:25%12. When the price of Maruti cars increased by 30% the number of cars sold decreases by 20%.the total sales revenue A. increased by 10% B. increased by 4% C. decreased by 2% D. none7. The difference between the compound interest payable half yearly and the simple interest on a certain sum lent out at 10% p.a for 1 year is Rs.25. What is the sum? (a) Rs.15000 (b) Rs.12000 c) Rs.10000 (d) None of these10. 5 men or 8 women do equal amount of work in a day. A job requires 3 me and 5 women to finish the job in 10 days. How many woman are required to finish the job in 14 days?(a) 10 (b) 7 (c) 6 (d) 1219. In a 1000 meter race, sita beats gita by 50 meter and in the same race; gita beats sarita by 50 meter. By what distance does sita beat sarita?a) 77.5 m b) 97.5 m c) 87.5 m d) 55.5 m 23. Find the number of selections and arrangements can be made by taking four letters from the word “EXPRESSION”?a) 113; 2910 b) 2190; 113 c) 113; 2190 d) 113; 2090

25. Find the length of an edge of a cube whose volume is equal to the volume of a rectangular pyramid. The base of the pyramid is a square of side 40 cm and its saint height is 25 cm.

Page 118: Quantitative

a) 15 cm b) 12 cm c) 10 cm d) 20 cm

1. A family, planning a weekend trip, decides to spend not more than a total of 8 hours driving. By leaving early in the morning, they can average 40 miles per hour on the way to their destination. Due to the heavy Sunday traffic, they can average only 30 miles per hour on the return trip. What is the farthest distance from home they can plan to go?(a) 120 miles or less (b) Between 120and 140 miles (c) 140 miles(d) Between 140 and 160 miles (e) 160 miles or more

2. A car is filled with four and half gallons of fuel for a round trip. If the amount of fuel taken while going is 1/4 more than the amount taken for coming, what is the amount of fuel consumed while coming back?(a) Less than 2 gallons (b) 2 gallons (c) 2 1/2 gallons      (d) 3 gallons  (e) More than 3 gallons

3. A 3-gallon mixture contains one part S and two parts R. In order to change it to a mixture containing 25% S, how much R should be added?(a) 1/2 gallon (b) 2/3 gallon (c) 3/4 gallon (d) 1 gallon (e) 1 1/2 gallon

4. A tree grows only 3/5 as fast as the one beside it. In four years the combined growth of the two trees is eight feet. How much does the shorter tree grow in two years?(a) Less than 2 feet (b) 2 feet (c) 2 1/2 feet (d) 3 feet (e) more than 3 feet.

5. Wind flows at 160 miles in 330 minutes, for traveling 80 miles how much time does it require?(a) 1 hour 30 minutes (b) 1 hour 45 minutes (c) 2 hours(d) 2 hours 45 minutes (e) 3 hours

6. A stationary engine has enough fuel to run 12 hours when its tank is 4/5 full. How long will it run when the tank is 1/3 full?(a) Less than 2 hours (b) 2 hours (c) 3 hours(d) 4 hours (e) 5 hours7. If A is traveling at 72 km per hour on a highway. B is traveling at a speed of 25 meters per second on a highway. What is the difference in their speeds in meters per second?(a) 1/2 m/sec (b) 1 m/sec (c) 1 1/2 m/sec(d) 2 m/sec (e) 3 m/sec

8. A salesperson by mistake multiplied a number and got the answer as 3, instead of dividing the number by 3. What is the answer he should have actually got?(a) 0 (b) 1/3 (c) 1 (d) 2 (e) 3

9. If the length of a rectangle is increased by 30% and the width is decreased by 20%, then the area is increased by...       (a) 10% (b) 5% (c) 4% (d) 20% (e) 25%

Page 119: Quantitative

10. In the class of 40 students, 30 speak Hindi and 20 speak English. What is the lowest possible number of students who speak both the languages?       (a) 5  (b) 20 (c) 15 (d) 10 (e) 30

11. The most economical prices among the following prices is:

(a) 10 kilo for Rs.160 (b) 2 kilo for Rs.30 (c) 4 kilo for Rs.70(d) 20 kilo for Rs.340 (e) 8 kilo for Rs.130

12. A truck contains 150 small packages, some weighing 1 kg each and some weighing 2 kg each. how many packages weighing 2 kg each are in the truck if the total weight of all the packages is 264 kg?

(a) 36 (b) 52 (c) 88 (d) 124 (e) 114

13. A man was arrested for exceeding the speed limit by 10 miles an hour. A second man was charged with exceeding the same limit by twice as much. The second man was driving 35 miles per hour. What was the speed limit?

(a) 10 miles per hour (b) 15 miles per hour (c) 20 miles per hour(d) 25 miles per hour (e) 30 miles per hour

14. One year ago Pandit was three times his sister's age. Next year he will be only twice her age. How old will Pandit be after five years?       (a) 8 (b) 12 (c) 11 (d) 13  (e) 15

24. If the vertex (5,7) is placed in the memory. First vertex (1,1) ‘s address is 1245 and then address of (5,7) is ----------

25. In which of the system, decimal number 384 is equal to 1234?

26. A man, a woman, and a child can do a piece of work in 6 days. Man only can do it in 24 days. Woman can do it in 16 days and in how many days child can do the same work?

27. In Madras, temperature at noon varies according to -t^2/2 + 8t + 3, where t is elapsed time. Find how much temperature more or less in 4pm to 9pm.

28. The size of the bucket is N kb. The bucket fills at the rate of 0.1 kb per millisecond. A programmer sends a program to receiver. There it waits for 10 milliseconds. And response will be back to programmer in 20 milliseconds. How much time the program takes to get a response back to the programmer, after it is sent?29. The size of a program is N. And the memory occupied by the program is given by M = square root of 100N. If the size of the program is increased by 1% then how much memory now occupied ?

Y-1. You start to work on a project with great deal of enthusiasm.N-2. You would rather plan an activity than take part in it.

Page 120: Quantitative

Y-3. You have more than once taken lead in organizing project or a group of some kind.Y-4. You like to entertain guests.N-5. Your interests change quickly from one thing to another.N-6. When you eat a meal with others, you are usually one of the last to finish.N-7. You believe in the idea that we should " eat, drink and be merry, for tomorrow we die."N-8. When you find that something you have bought is defective, you hesitate to demand an exchange or a refund.Y-9. You find it easy to find new acquaintances.N-10. You are sometimes bubbling over with energy and sometimes very sluggish.Y-11. You are happiest when you get involved in some projects that calls for rapid action.N-12. Other people think of you as being very serious minded.N-13. In being thrown by chance with a stranger, you wait for the person to introduce himself or herself.Y-14. You like to take part in many social activities. N-15. You sometimes feel "just miserable" for no good reason at all.

N-16. You are often so much " on the go" that sooner or later you may wear yourself out.

Y-17. You like parties you attend to be lively.

Y-18. If you hold an opinion that is radically different that expressed by a lecturer, you are likely to tell the person about it either during or after the lecture.

N-19. It is difficult for you to chat about things in general with people.

N-20. You give little thought to your failures after they are passed.

N-21. You often wonder where others get all the excess energy they seem to have.

Y-22. You are inclined to stop to think things over before you act.

N-23. You avoid arguing over a price with a clerk or sales person.

Y-24. You would dislike very much to work alone in some alone place.

N-25. You often find it difficult to go to sleep at night because you keep thinking of what happened during the day.

N-26. You find yourself hurrying to get to places even when there is plenty of time.

Y-27. You like work that requires considerable attention to details.

N-28. You are satisfied to let some one else take the lead in group activities.

Y-29. You enjoy getting acquainted with people.

Y-30. It takes a lot to get you emotionally stirred up or excited.

Page 121: Quantitative

N-31. You work more slowly and deliberately than most people of your sex and age.

Y-32. You are a carefree individual.

N-33. When people do not play fair you hesitate to say anything about it to them.

N-34. It bothers you to have people watch you at your work.

Y-35. You have usually been optimistic about your future.

N-36. You like to have plenty of time to stop and rest.

Y-37. You take life very seriously.

Y-38. You enjoy applying for a job in person.

Y-39. You would like to be a host or hostess for parties at club.

N-40. You often feel uncomfortable or uneasy.

Y-41. You are the kind of person who is "on the go" all the time.

Y-42. You often crave excitement.

N-43. The thought of making a speech frightens you.

Y-44. You find it easy to start conversation with strangers.

N-45. You often feel guilty without a very good reason for it.

Y-46. People think you are a very energetic person.

N-47. You sometimes make quick decisions that you later wish you hadn't made.

N-48. You find it difficult to ask people for money or other donations, even for a cause in which you are interested.

Y-49. You are so naturally friendly that people immediately feel at ease with you.

N-50. You daydream a great deal.

Y-51. You are quick in your actions.

N-52. You have a habit of starting things and then losing interest in them.

Y-53. When you were a child many of your playmates naturally expected you to be the leader. 

N-54. You sometimes avoid social contacts for fear of doing or saying the wrong thing.

Page 122: Quantitative

N-55. You have frequent ups and downs in mood, sometimes with and sometimes without apparent cause.

Y-56. You always seem to have plenty of vigour and vitality.

N-57. It is difficult for you to understand people who get very concerned about things.

Y-58. When a clerk in a store waits on others who come after you, you call his or her attention to the fact.

Y-59. You would be very unhappy if you were prevented from making numerous social contacts.

N-60. There are times when your future looks very dark.

N-61. You sometimes wish that people would slow down a bit and give you a chance to catch up.

N-62. Many of your friends think you take your work too seriously.

N-63. You hesitate to walk into a meeting when you know that everyone's eye will be upon you.

N-64. You limit your friendships mostly to members of your own sex.

Y-65. You almost always feel well and strong.

N-66. You seem to lack the drive necessary to get as much as other people do.

N-67. You make decisions on the spur of the moment.

N-68. You are rather good at bluffing when you find yourself in difficulty.

N-69. After being introduced to someone , you just cannot think of things to say to make good conversation.

N-70. You feel lonesome even when with other people.

Y-71. You are able to work for unusually long hours without feeling tired.

N-72. You often act on the first thought that comes into your head.

Y-73. At the scene of an accident, you take an active part in helping out.

N-74. You have difficulty in making new friends.

N-75. Your mood often changes from happiness to sadness or vice versa without knowing why.

Page 123: Quantitative

N-76. You talk more slowly than most people.

N-77. You like to play practical jokes upon others.

Y-78. You take the lead in putting life into a dull party.

Y-79. You would like to belong to as many clubs and social organizations as possible.

N-80. There are times when your mind seems to work very slowly and other times when it works very rapidly.

N-81. You like to do things slowly and deliberately.

N-82. You are a happy-go-lucky individual.

N-83. When you are served stale or inferior food in a restaurant, you say nothing about it.

N-84. You would rather apply for a job by writing a letter than by going through with a personal interview.

N-85. You are often in low spirits.

Y-86. You are inclined to rush from one activity to another without pausing enough for rest.

N-87. You are so concerned about the future that you do not get as much fun out of the present as you might.

Y-88. When you are attracted to a person whom you have not met earlier you make an active attempt to get - acquainted even though it may be quite difficult.

N-89. You are inclined to limit your acquaintances to select few

N-90. you seldom give your past mistakes a second thought.

N-91. You are less energetic than many people you know.

Y-92. You often stop to analyzed your thoughts and feelings.

Y-93. You speak out in meetings to oppose those whom you feel sure are wrong.

N-94. You are so shy it bothers you.

N-95. You are sometimes bothered by having a useless thought come into your mind over and over.

N-96. You get things in hurry.

Page 124: Quantitative

N-97. It is difficult for you to understand how some people can be so unconcerned about the future.

N-98. You lie to sell things (i.e. to act as a sales person)

Y-99. You are often "Life of the Party".

N-100. You find daydreaming very enjoyable.

N-101. At work or at play other people find it hard to keep up with the pace you set.

Y-102. You can listen to a lecture without feeling restless.

Y-103. You would rather work for a good boss than for yourself.

Y-104. You can express yourself more easily in speech than in writing.

Y-105. You keep in fairly uniform spirits.

Y-106. You dislike to be hurried in your work.

N-107. You sometimes find yourself "crossing bridges before you come to them".

N-108. You find it somewhat difficult to say "no" to a sales person who tries to sell you something you do not - really want.

N-109. There are only a few friends with whom you can relax and have a good time.

Y-110. You usually keep cheerful in spite of trouble.

N-111. People sometimes tell you to "slow down" or "take it easy".

N-112. You are one of those who drink or smoke more than they know they should.

Y-113. When you think you recognize people you see in a public place, you ask them whether you have met - them before.

N-114. You prefer to work alone.

N-115. Disappointment affect you so little that you seldom think about them twice.

N-116. You are slow and deliberate in movements.

N-117. You like wild enthusiasm, sometimes to a point bordering on rowdyism at a football or baseball game.

N-118. You feel self conscious in the presence of important people.

Y-119. People think of you as being a very social type of person.

Page 125: Quantitative

N-120. You have often lost sleep over your worries.

Y-121. You can turn out a large amount of work in a short time.

Y-122. You keep at a task until it is done, even after nearly everyone else has given up. 

N-123. You can think of a good excuse when you need one.

N-124. Other people say that it is difficult to get to know you well.

N-125. You daydreams are often about things that can never come true.

N-126. You often run upstairs taking two steps at a time.

Y-127. You seldom let your responsibility interfere with your having a good time.

Y-128. You like to take on important responsibilities such as organizing a new business.

N-129. You have hesitated to make or to accept "dates" because of shyness.

N-130. Your mood is very easily influenced by people around you.

Y-131. Others are often amazed by the amount of work you turn out.

N-132. You generally feel as though you haven't a care in the world.

N-133. You find it difficult to get rid of sales person whom you do not care to listen or give your time.

N-134. You are a listener rather than a talker in a social conversation.

Y-135. You almost always feel that life is very much worth living.

N-136. It irritates you to have to wait at a crossing for a long freight train to pass.

N-137. You usually say what you feel like saying at the moment.

Y-138. You like to speak in public.

Y-139. You like to be with people.

Y-140. You generally keep cool and think clearly in exciting situations.

Y-141. Other people regard you as a lively individual.

Y-142. When you get angry, if you let yourself go, you feel better.

Y-143. You seek to avoid all trouble with other people.

Page 126: Quantitative

Y-144. People seem to enjoy being with you.

N-145. You sometimes feel listless and tired for no good reason.

N-146. It is hard to understand why many people are so slow and get so little done.

N-147. You are fond of betting on horse races and games, whether you can afford it or not.

Y-148. If someone you know has been spreading untrue and bad stories about you, you see the person as - soon as possible and have a talk about it.

N-149. Shyness keep you from being as popular as you should be.

Y-150. You are generally free from worry about possible misfortunes.

There are 60 questions to be answered in 60 minutes 1) In a club there are certain no. of males and females. If 15 females are absent then no. of males will be twice that of females. If 45 males are absent then female strength will be 5 times that of males. Find no. of males actually present. 2) Three men A, B, C plays Cards. If one loses the game he have to give Rs.3. If he wins the game he will gain Rs.6. If A has won 3 Games, B loses Rs.3, C wins Rs.12.What is the total no. of games played? 3) A can swim & cross 50m(the length of swimming pool) in 2 min. B can swim & cross 50m in 2min 15sec. Every time when they meet a bell gong is struck. For 2000m how many bell sounds might be produced? 4) When I was married 10 years ago my wife is the 6th member of the family.  Today my father died and a baby born to me. The average age of my family during my marriage is same as today. What is the age of Father when he died?ans:70. 5)There are 9 balls of equal size and same weight(they look similar) except 1. How many weights required to find the dissimilar ball using a weighing balance? 6) Product of Prime no. between 1 to 20? 7) Find out the total numbers between 1 to 999 that are neither divisible by 8 nor by 12? 8) Find out the distinct numbers that can be formed by 2, 3, 7, 6(don’t now exactly) that should be divided by 4? 9) Two trains at speed 60 km/hr comes in the opposite direction. At a particular time the distance between the two trains is 18km. A shuttle flies between the trains at the speed of 80 km/hr. At the time the two trains crashes what is the distance traveled by shuttle?10) There are n urns and m balls. If we put 3 balls in each urn 3 balls will be excess. If we put 4 balls in each urn 1 urn will be excess. Find no. of Urns (or) Balls?12)  A man gets x/y of Rs.10 and y/x of Rs.10. He returns Rs.20. The Answer choices area) He may not loseb) He may losec) He always losesd) Cannot be Determined 13)  Three men A,B,C can complete a work separately in some specified days(may be6,7,8).If they do the work together by alternate days. Then how many days need to complete the work?18)There are 1997 doors in a auditorium and as many as people as the no of doors open enter the auditorium .A door was closed and as many as people as no of doors open

Page 127: Quantitative

leave the auditorium . the process was repeated till the no of doors is equal to1.find the total no of peoples enter the auditorium?19)A batsman average was 15.at last innings he took 23 runs then his average became  16.how much run he should take to make his average 18?25)if a 36 cm thread is used to wrap a book , lengthwise twice and breadthwise once, what is the size of the book? 

Page 128: Quantitative

26) 

The various degrees of the vertices are marked in the dig. Find y. 27) if 4 circles of equal radius are drawn with vertices of a square as the centre , the side of the square being 7 cm, find the area of the circles outside the square? 28) A bus has 40 seats and the passengers agree to share the total bus fare among themselves equally. If the total fair is 80.67 , find the total no of  the seats unoccupied. 29) A 4 digit no may consist of the digits 6,2,7,5 where none of the nos are repeated. Find the possible no of combinations divisible by 36? 

Page 129: Quantitative

31)

(  Diagram drawn approximate not to scale.) if all the blocks are squares and the complete fig is also a square and the area of the a is 1 cm2,b=81 cm2what is the area of i? 33) Three birds cross a point in a same st. line and of that 2 fly in opposite directions. If a triangle is formed with the position of the birds what kind of the triangle will be formed?a) isosceles  b)right angle    c)equilateral  d)right angle and isosceles.